Critical Care Nursing A Holistic Approach 11th Edition Morton Fontaine Test Bank PDF

You might also like

Download as pdf or txt
Download as pdf or txt
You are on page 1of 335

Critical Care Nursing A Holistic Approach 11th Edition Test Bank

Chapter 1 Critical Care Nursing Practice: Promoting Excellence-Caring,Collaboration


Evidence

1. A nurse is the only one in the ICU who has not achieved certification in critical care
nursing. She often will ask her fellow nurses what to do in caring for a patient because
she doubts the accuracy of her knowledge and her intuition. She loves her work but
wishes she could do it with a greater level of competence. What is the most important
effect that obtaining certification would likely have on the nurses practice?

om
A) Recognition by peers

.c
B) Increase in salary and rank

ep
C) More flexibility in seeking employment
D) Increased confidence in making decisions

pr
2. A hospital interviews two different candidates for a position in the ICU. Both candidates
st
have around 10 years of experience working in the ICU. Both have excellent
interpersonal skills and highly positive references. One, however, has certification in
te
critical care nursing. Which of the following is the most compelling and accurate reason
for the hospital to hire the candidate with certification?
ng

A) The certified nurse will have more knowledge and expertise.


si

B) The certified nurse will behave more ethically.


ur

C) The certified nurse will be more caring toward patients.


yn

D) The certified nurse will work more collaboratively with other nurses.
3. A nurse is caring for an elderly man recently admitted to the ICU following a stroke. She
.m

assesses his cognitive function using a new cognitive assessment test she learned about in
a recent article in a nursing journal. She then brings a cup of water and a straw to the
w

patient because she observes that his lips are dry. Later, she has the patient sit in a
w

wheelchair and takes him to have some blood tests performed. He objects at first, saying
that he can walk on his own, but the nurse explains that it is hospital policy to use the
w

wheelchair. That evening, she recognizes signs of an imminent stroke in the patient and
immediately pages the physician. Which action taken by the nurse is the best example of
evidence-based practice?
A) Giving the patient a cup of water
B) Transferring the patient in a wheelchair
C) Using the cognitive assessment test
D) Recognizing signs of imminent stroke and paging the physician
4. A nurse is on a committee that is trying to reduce the occurrence of hospital-acquired
infections in the ICU. Her role is to conduct research to find which interventions have
been shown to be most effective in reducing these infections. She consults many different
sources and finds conflicting information. Which of the following sources should she
consider the most authoritative?
A) AACN expert panel report
B) A meta-analysis of randomized controlled trials in the American Journal of Nursing
C) A systematic review of qualitative studies in the Journal of Advanced Nursing

om
D) A single randomized controlled trial in the American Journal of Critical Care
5. A nurse who has been recently hired to manage the nursing staff of the ICU is concerned
at the lack of evidence-based practice she sees among the staff. Which of the following

.c
would be the best step for her to take to promote incorporating evidence into clinical

ep
practice?
A) Only hire nurses certified in critical care nursing.

pr
B) Leave copies of several different nursing journals in the nurses lounge.
C) st
Demonstrate to the staff the best nursing-related search terms to use in Google or
te
Yahoo!
ng

D) Introduce the staff to the PubMed search engine and assign them topics to research
on it.
si

6. A physician visits a patient in the ICU while the nurse is out. The patient complains that
the pain medication is not effective and that he would like to receive an increased dose.
ur

The physician has the nurse paged and consults with him in the hallway regarding the
yn

patients request for stronger pain medication. The nurse explains that patient was started
on a morphine drip only 20 minutes ago and that the drug has not had time to take effect
yet. The physician agrees and tells the patient to give it just a bit more time. Which
.m

component of a healthy work environment is most evident in this scenario?


w

A) Skilled communication
w

B) Appropriate staffing
w

C) True collaboration
D) Meaningful recognition
7. A nurse in the ICU is responding to a patient who has just gone into cardiac arrest. A
moment later, the nurse is notified that another patient has just gone into anaphylactic
shock due to a drug allergy. She is conflicted as to what to do, as she is the only nurse
available at the moment to tend to both patients. Which component of a healthy work
environment is lacking in this scenario?
A) Effective decision making
B) Appropriate staffing
C) Authentic leadership
D) Meaningful recognition
8. An ICU nurse has provided excellent care for a 6-year-old girl who had been admitted to
the ICU for a head injury. The nurse was attentive not only to the needs of the patient but
also went out of her way to care for the needs of the girls family. According to research,
which of the following forms of recognition would the nurse value the most?
A) A card from the girls family

om
B) A plaque from the ICU physicians naming her as Nurse of the Year
C) A letter of commendation from the hospitals administration

.c
D) A bouquet of flowers from her supervisor

ep
9. A patient in the ICU has recently been diagnosed with diabetes mellitus. Before being
discharged, this patient will require detailed instructions on how to manage her diet, how

pr
to self-inject insulin, and how to handle future diabetic emergencies. Which nurse
competency is most needed in this situation?
A) Clinical judgment
st
te
B) Advocacy and moral agency
ng

C) Caring practices
si

D) Facilitation of learning
ur

10. An elderly patient is admitted to the ICU with stage IV lung cancer, diabetes mellitus, and
congestive heart failure. The health care team assembled to care for her is large and
yn

diverse, including an oncologist, a pulmonologist, an endocrinologist, a cardiologist, and


others. The patient is not expected to survive more than a few weeks, and her husband is
.m

overwhelmed with stress and grief. Which nurse competency or competencies are most
needed in this situation? Select all that apply.
w

A) Clinical judgment
w

B) Caring practices
w

C) Collaboration
D) Response to diversity
11. An Ethiopian man with AIDS has recently been admitted to the ICU with a case of
pneumonia. The man is new to the U.S. and has no health insurance. He would likely be
eligible for the states Medicaid coverage, but does not understand how to access this
coverage. Which competency or competencies are most needed in this situation? Select
all that apply.
A) Clinical judgment
B) Advocacy and moral agency
C) Collaboration
D) Systems thinking
E) Response to diversity
F) Clinical inquiry
12. A nurse decides to seek certification in critical care nursing. What is the most important

om
benefit for the individual nurse in becoming certified in a specialty?
A) It will result in a salary increase.

.c
B) It is required to work in critical care.

ep
C) It demonstrates the nurses personal expertise.
D) It is mandated by employers.

pr
13. The American Association of Critical-Care Nurses (AACN) sponsors certification in
st
critical care nursing for several critical care subspecialties. What is the most important
te
benefit of such certification for the profession of nursing?
A) Provides positive publicity for nursing
ng

B) Validates nurses expert knowledge and practice


si

C) Mandated by government regulations


ur

D) Demonstrates basic knowledge in the field


yn

14. A nurse has achieved certification in critical care nursing. What is the most important
effect that this certification will have on the nurses practice?
.m

A) Recognition by peers
w

B) Increase in salary and rank


w

C) More flexibility in seeking employment


w

D) Increased confidence in critical thinking


15. The nurse cites evidence-based practice as a rationale for a patient care decision. What is
the best description of evidence-based practice?
A) Decisions based on expert legal testimony
B) Use of best available research data
C) Evolution of nursing practice over time
D) Individual optimization of patient outcomes
16. The nurse caring for a critically ill patient implements several components of care. What
component is an example of the use of evidence-based practice?
A) Use of a protocol for admission of a patient to the unit
B) Application of an insulin sliding scale method from research
C) Checking the patients armband before giving a medication
D) Limiting visits to immediate family only for 2 hours a day
17. The nurse wishes to increase the use of evidence-based practice in the critical care unit
where he works. What is a significant barrier to the implementation of evidence-based

om
practice?
A) Use of computerized records by the hospital

.c
B) Health Information Privacy and Portability Act (HIPPA)

ep
C) Lack of knowledge about literature searches

pr
D) Strong collaborative relationships in the work setting

st
18. The nurse has identified an increase in medication errors in the critical care unit over the
past several months. What aspect of medication procedures should be evaluated first?
te
A) Adherence to procedures by nursing staff
ng

B) Clarity of interdisciplinary communication


si

C) Number of new employees on the unit


ur

D) Changes in administration procedures


yn

19. A critical care unit has decided to implement several measures designed to improve
intradisciplinary and interdisciplinary collaboration. In addition to an expected
improvement in patient outcomes, what is the most important effect that should result
.m

from these measures?


w

A) Identification of incompetent practitioners


w

B) Improvement in manners on the unit


w

C) Increased staff retention


D) Less discussion in front of patients and families
20. A nurse wishes to practice using the Synergy Model developed by the American
Association of Critical-Care Nurses (AACN). What nursing behavior best supports use of
this model?
A) Attending mandatory hospital-wide in-service programs
B) Self-directed study of best practice for the patients she cares for
C) Gathering demographic data on the patients admitted to the unit
D) Participating in a research study as a data collector
21. As part of the Synergy Model, the nurse has identified a patient characteristic of
resiliency. What patient behavior demonstrates resiliency?
A) Dysfunctional grieving behaviors after receiving bad news
B) Developing a list of questions for the physician
C) Denial of any possible negative outcomes for a procedure

om
D) Assigning blame to others for undesired outcomes of illness
22. A patient is admitted to the critical care unit after receiving a mechanical heart
implantation. In making a nursing assignment, the charge nurse best demonstrates

.c
application of the Synergy Model by assigning which nurse to care for this patient?

ep
A) A newly hired new graduate nurse, for the experience

pr
B) A nurse with a patient in the next room, for proximity
C)
st
The most senior nurse on the unit, for political reasons
te
D) The nurse with most experience with this device, for expertise
23. A Muslim patient has been admitted to the critical care unit with complications after
ng

childbirth. Based on the Synergy Model, which nurse would be the most inappropriate to
assign to care for this patient?
si

A) New graduate female nurse


ur

B) Most experienced female nurse


yn

C) New graduate male nurse


.m

D) Female nurse with postpartum experience


24. Todays critical care nursing environment is constantly changing. What nursing behavior
w

best illustrates awareness of current events affecting critical care nursing?


w

A) Participating in the hospitals efforts to recruit new nurses


w

B) Volunteering to serve on a disaster response planning committee


C) Adhering to content taught in basic nursing program
D) Attending hospital-mandated in-services without other education

Answer Key

1. D
2. A
3. C
4. B
5. D
6. C
7. B
8. A

om
9. D
10. B, C

.c
11. B, D, E

ep
12. C

pr
13. B
14. D st
te
15. B
ng

16. B
17. C
si

18. B
ur

19. C
yn

20. B
.m

21. B
22. D
w

23. C
w

24. B
w

Chapter 2 The Patient’s Experience With Critical Illness

1. The critical care unit environment is very stressful for patients, families, and staff. What
nursing action is directed at reducing environmental stress?
A) Constant evaluation of patient status
B) Limiting visits to immediate family
C) Bathing all patients during hours of sleep
D) Maintaining quiet during hours of sleep
2. A patient is transferred to the ICU from the Birth Center of the hospital in the middle of
the night after experiencing complications during delivery of her baby. The patients
husband is anxious and explains to the ICU nurse that he doesnt understand why his wife
has been moved to the ICU. She is going to die, isnt she? he asks the nurse. What is the
nurses best response?
A) Explain that every measure will be taken to provide his wife with the best care
possible.

om
B) Explain that the nurse is fully trained and has years of experience.
C) Offer the husband a place to relax.

.c
D) Have appropriate staff discuss his health insurance with him.

ep
3. A patient is admitted to the ICU with injuries sustained from a fall from a third-story

pr
window. The patient is conscious, his breathing is labored, and he is bleeding heavily
from the abdomen. He groans constantly and complains of severe pain, but his
st
movements are minimal. His heart rate is elevated. Which of these is a sign that he is in
the second phase of the stress response? Select all that apply.
te
A) Bleeding heavily from his abdomen
ng

B) Labored, slow breathing


si

C) Severe pain
ur

D) Elevated heart rate


yn

E) Minimal movement
4. A patient in the ICU is recovering from open-heart surgery. The nurse enters his room
.m

and observes that his daughter is performing effleurage on his arms and talking in a low
voice about an upcoming family vacation that is planned. The room is dimly lit, and she
w

hears the constant beeping of his heart monitor. From the hall she hears the cries of a
patient in pain. Which of the following are likely stressors for the patient? Select all that
w

apply.
w

A) His daughters conversation


B) His daughters effleurage
C) The beeping of the heart monitor
D) The dim lighting of the room
E) The cries of the other patient from the hall
5. A patient in the ICU is complaining that he is not sleeping well at night because of
anxiety. Which of the following would be the most helpful intervention for the nurse to
make?
A) Provide the patient with a bath immediately following his first 90-minute REM
sleep cycle.
B) Increase the patients pain medication.
C) Provide the patient with 5 minutes of effleurage and then minimize disruptions.
D) Monitor the patients brain waves by polysomnography to determine his sleep

om
pattern.
6. A nurse walks into a patients room and begins preparing a syringe to perform a blood
draw on the patient. The nurse observes that the patient is firmly gripping the side of the

.c
bed, averting her eyes, and sweating from her forehead when she sees the needle. What

ep
would be the best intervention for the nurse to make?
A) Proceed with blood draw as quickly as possible, to get it over with.

pr
B) Offer to come back later to perform the blood draw.
C) Encourage the patient to deep breathe. st
te
D) Describe briefly the blood draw procedure and explain why it is necessary.
ng

7. A 15-year-old boy is in the ICU and preparing for an appendectomy. He is clearly


anxious and fidgets with his IV constantly. He complains that he doesnt want to be there
si

and he is sick of everyone telling him what to do. What would be the best way for the
nurse to address this patients anxiety?
ur

A) Use physical restraints to keep him from pulling out his IV.
yn

B) Offer him the remote to the television.


.m

C) Lower the head of his bed so that he can rest more easily.
w

D) Explain to the patient in detail what the appendectomy will consist of.
w

8. A nurse in a burn unit observes that a patient is tensed up and frowning but silent. The
nurse asks the patient, Can you tell me what you are thinking now? The patient responds,
w

I cant take this pain any more! I feel like Im about to die. What would be the best
response for the nurse to give to the patient, considering that the patient is already
receiving the maximum amount pain medication that is safe?
A) Try to get rid of those negative thoughtsthey only make it worse.
B) Try thinking instead, This pain will go away; I can overcome it.
C) Your pain medication is already at the highest possible dose.
D) Would you like me to raise the head of your bed?
9. A patient on mechanical ventilation is experiencing severe agitation due to being on the
ventilator. Which nursing intervention would be best?
A) Performing breathing exercises with the patient
B) Offering the patient a patient-controlled analgesic device
C) Asking the physician to prescribe an antianxiety medication
D) Offering the patient the patients own MP3 player to listen to
10. A 10-year-old female patient in ICU receiving chemotherapy has requested that her dog
be allowed to visit her. She is currently sharing a room with another patient. The nurse

om
knows that the hospital does allow for pet visits with owners, but has strict guidelines.
Which of the following scenarios is most likely to be permitted?

.c
A) The girls father may bring the dog in on a leash for a 20-minute visit.

ep
B) The girls sister may bring the dog in with a shirt on (to prevent shedding) for an
overnight stay.

pr
C) The girls mother may bring the dog in on a leash for a visit as long as he has had all
his vaccinations.
D)
st
The dog may be brought in for a brief visit once the girl is moved to a private room.
te
11. The nurse understands that a patient being cared for in a critical care unit experiences an
ng

acute stress response. What nursing action best demonstrates understanding of the
physiological parts of the initial stress response?
si

A) Adequate pain control


ur

B) Intravenous sedation
yn

C) Treatment for elevated blood pressure


.m

D) Ignoring an elevated glucose level


12. A critically ill patient experiences stress and anxiety from many factors. Treatment of the
w

patient focuses on reducing stressors and providing supportive care such as nutrition,
w

oxygenation, pain management, control of anxiety, and specific care of the illness or
injury. What is the best rationale for these interventions?
w

A) Helps to support the patients immune system


B) Part of good nursing care
C) Mandated by hospital policy
D) Reassures the patient and family
13. A patient in a critical care unit has increased stress from the constant noise and light
levels. What nursing intervention best attenuates these sources of stress?
A) Need for constant observation and evaluation
B) Dimming lights during the night
C) Frequent nursing group rounds for all patients
D) Use of tile floors for ease in cleaning
14. The nurse is caring for a patient who is orally intubated and on a mechanical ventilator.
The nurse believes that the patient is experiencing excess anxiety. For this patient, what
behavior best indicates anxiety?
A) Restlessness

om
B) Verbalization
C) Increased respiratory rate

.c
D) Glasgow Coma Scale score of 3

ep
15. The critical care unit environment is very stressful for patients, families, and staff. What

pr
nursing action is directed at reducing environmental stress?
A)
st
Constant expert evaluation of patient status
te
B) Limiting visits to immediate family
C) Bathing all patients during hours of sleep
ng

D) Maintaining a quiet environment during hours of sleep


si

16. The nurse wishes to enhance sleep cycles in her critically ill patient. Research has shown
ur

that which nursing action improves sleep in critically ill patients?


yn

A) Repositioning every 2 hours


B) Hypnotic medications
.m

C) Five-minute back effleurage


w

D) Adequate pain control


w

17. The nurse is caring for a critically ill patient with a very concerned family. Given that the
w

family is under high stress, what nursing intervention will best ameliorate their stress
while preserving independence?
A) Encourage the family to participate in patient care tasks.
B) Teach the family to ask questions of the health care team.
C) Ask the family to select a family representative for communication.
D) Limit visits to immediate family members for limited times.
18. While caring for a critically ill patient, the nurse knows that fostering patient control over
the environment is a method for stress reduction. What nursing intervention gives the
patient the most environmental control while still adhering to best practice principles?
A) Ask the patient whether he or she wants to get out of bed.
B) Give the patients bath at the same time every day.
C) Explain painful procedures only after giving pain medication.
D) Choose menu items for the patient to ensure a balanced diet.
19. The nurse is using presence to reduce the anxiety of a critically ill patient. What nursing

om
behavior demonstrates an effective use of presence?
A) Staying in the patients room to complete documentation

.c
B) Having a conversation in the patients room that excludes the patient

ep
C) Maintaining eye contact with the patient during explanations

pr
D) Focusing on specific nursing care tasks while in the patients room

st
20. The nurse is caring for a critically ill patient who can speak. The nurse notices that the
patient is demonstrating behaviors indicative of anxiety but is silent. What nursing
te
strategy would give the nurse the most information about the patients feelings?
ng

A) Explain procedures to the patient and family.


B) Ask the patient to share his or her internal dialogue.
si

C) Encourage the patient to nap before visiting hours.


ur

D) Ensure that the patient has adequate pain control.


yn

21. The patient is undergoing a necessary but painful procedure that is greatly increasing her
anxiety. The nurse decides to use guided imagery to help alleviate the patients anxiety.
.m

What is a key part of this technique?


w

A) Provide the patient with an external focus point such as a picture.


w

B) Have the patient take slow, shallow breaths while staring at a focus point.
w

C) Have the patient remember tactile sensations of a pleasant experience.


D) Encourage the patient to consciously relax all of her muscles.
22. One of the strategies shown to reduce perception of stress in critically ill patients and
their families is support of spirituality. What nursing action is most clearly supportive of
the patients spirituality?
A) Referring patients to the Catholic chaplain
B) Providing prayer booklets to patients and families
C) Asking about beliefs about the universe
D) Avoiding discussing religion with those of other faiths
23. A critically ill patient tells the nurse that he is not afraid to die because he believes in
reincarnation. What is the most appropriate nursing response?
A) What if reincarnation is not real?
B) This belief gives you strength.
C) I dont believe in reincarnation.

om
D) You shouldnt base your hopes on such a belief.
24. A critically ill patient who is intubated and agitated is restrained with soft wrist restraints.
Based on research findings, what is the best nursing action?

.c
A) Maintain the restraints to protect patient safety.

ep
B) Remove the restraints periodically to check skin integrity.

pr
C) Remove the restraints periodically for range of motion.
D) st
Assess and intervene for causes of agitation.
te
ng

Answer Key
si

1. D
ur

2. A
3. C, E
yn

4. C, E
.m

5. C
w

6. D
w

7. B
w

8. B
9. D
10. D
11. A
12. A
13. B
14. A
15. D
16. C
17. B
18. B
19. C
20. B

om
21. C
22. C

.c
23. B

ep
24. D

pr
Chapter 3 The Family’s Experience With Critical Illness
st
1. A patient has just been admitted to the ICU after being in a severe auto accident and
te
losing one of her legs. Her husband has his hand over his heart and complains of a rapid
ng

heart rate. The nurse recognizes his condition as a sign of which stage of the general
adaptation syndrome to stress?
si

A) Alarm stage
ur

B) Exhaustion stage
yn

C) Resistance stage
D) Adaptation stage
.m

2. The nurse observes that an elderly woman, whose granddaughter has been admitted to the
w

ICU, is struggling to manage her two great-grandsons, who are toddlers, in the waiting
room. What is the most likely explanation for the womans inability to manage the
w

children in this situation?


w

A) She is senile.
B) She is in the exhaustion stage of the general adaptation syndrome to stress.
C) She is assuming the role of caregiver in place of the patient, a role she is not used to.
D) She has macular degeneration and cannot see well.
3. A nurse needs to communicate with a patients family regarding consent to treat an
unconscious patient in the ICU. Which member of the group should the nurse approach
first?
A) A man she recognizes as the patients brother
B) A teenage boy who approaches the nurse
C) A woman who originally escorted the patient in
D) A woman in the group whom the others look at and call over when the nurse
approaches
4. A new nurse has recently joined the ICU from a different hospital, which had a much
stricter policy regarding visiting hours. She expresses concern about the impact of open
visiting hours on patient well-being. Which of the following would be the best

om
explanation for the purpose of open visiting hours? Select all that apply.
A) To better provide rest and quiet

.c
B) To strengthen the relationship between the family and health care provider

ep
C) To control the number of visitors for a patient
D) To provide an undisturbed environment

pr
E) To decrease the patients anxiety
F)
st
To increase the satisfaction of the family with the experience
te
5. A nurse observes that a 38-year-old single father whose 11-year-old daughter is in the
ng

ICU is struggling to explain to his 6-year-old son the likelihood that the daughter will die.
The young boy asks what will happen to his sister when she dies, but the father breaks
si

down in tears and seems unable to respond. Which of the following would be the most
appropriate intervention for the nurse to make?
ur

A) Suggest that the father contact his pastor, rabbi, or other spiritual leader for counsel
yn

for him and his son


B) Sit down with the father and son and share her own religious beliefs
.m

C) Ask the patients doctor to explain to the father the odds of the daughter surviving
w

D) Leave the father and son to grieve alone


w

6. A patient is experiencing severe pain, despite receiving pain medication for the past 24
w

hours. The patients wife expresses concern about this to the nurse. Which response by the
nurse would be most empowering to the patients family?
A) Explain that the doctor is an expert on pain medication and that the current level of
medication is the best.
B) Recommend that the family members take turns massaging the patients feet to
distract from the pain.
C) Encourage the family to request that the physician evaluate the patients pain control.
D) Ask the family to wait another 24 hours to see whether the patients pain level will
go down.
7. The sister of a patient in the ICU has been at the patients bedside non-stop for 48 hours.
The nurse suggests to her that she return home to rest. Which of the following is the
proper rationale for the nurse making such a suggestion?
A) The sister is in the way of the health care providers.
B) The patient may become annoyed by her continual presence.
C) The patient will recover more easily in peace and quiet.

om
D) The sister needs to maintain her own health during this time.
8. A young man has just arrived at the ICU from out of town and received news that his

.c
girlfriend, who is admitted there, likely only has a few days left to live. Which of the
following would be the best approach for the nurse to take in caring for the needs of this

ep
young man?

pr
A) Recommending that he go home and rest
B) Giving him unrestricted visiting hours with the patient
C)
st
Suggesting that he meet with the hospital chaplain
te
D) Recommending that he ask the doctor to evaluate the patients pain control measures
ng

9. A family of a young girl who has been diagnosed with leukemia has travelled 12 hours by
si

car to admit her to the ICU and be with her during her treatment. Which aspect of the
critical care family assistance program would most likely be needed by this family
ur

initially?
yn

A) Educational materials
B) Weekly group family information sessions
.m

C) Hospitality programs
w

D) Pet therapy
w

10. A Muslim woman is admitted to the ICU after suffering severe burns over most of her
w

body. Which of the following would be the most appropriate measure for the nurse, a
woman, to take in respect for the cultural practices of this patient?
A) Insist that only a female doctor be assigned to this patient.
B) Ensure that no pork products are included in the patients diet.
C) Ensure that direct eye contact is not made with the patients husband.
D) Ask the patients husband what religious and cultural preferences should be
considered in the patients care.
11. A client has been admitted after experiencing multiple trauma and is intubated and
sedated. When the five members of the immediate family arrive, they are anxious, angry,
and very demanding. They all speak loudly at once and ask for many services and
answers. What is the best nursing response?
A) Ask the family to leave until visiting hours begin.
B) Take them to a private area for initial explanations.
C) Page security to have them removed from unit.
D) Show them to the clients bedside and leave them alone.

om
12. The client has been in the CCU for several weeks and has been very unstable. One family
member stays at the bedside constantly and even naps in a bedside chair. The nurse
understands that the family member is exhibiting which family member response to

.c
critical illness?

ep
A) Exhibiting extreme distrust of the health care team

pr
B) Seeking evidence for future legal or punitive action
C) Trying to maintain a level of control over the situation
D)
st
Experiencing extreme fatigue from constant stress
te
13. The nurse is caring for a very seriously ill patient in the CCU. The family visits
ng

sporadically, stays for only a short time, and does not ask many questions. How could the
nurse best begin to involve the family in the patients care?
si

A) Ask one family member to assist with the patients bath.


ur

B) Encourage family members to stay longer at each visit.


yn

C) Focus nursing efforts on the patients legal next of kin.


.m

D) Ask the family to complete the Critical Care Family Needs Inventory.
14. As part of the admission process, the nurse asks several questions about family
w

relationships. The nurse bases these actions on which rationale?


w

A) Assessing family relationships is an initial step in including the family in patient


w

care.
B) These questions are part of the admission assessment tool required by this CCU.
C) The nurse has a natural curiosity and wishes to know how the family members relate
for her own knowledge.
D) There is an ongoing research study to identify variant family patterns related to
disease incidence.
15. On their first visit to a critically ill patient, family members stand in the doorway of the
room, making no effort to approach the patient. What is the most appropriate nursing
action?
A) Instruct the family where the patient can be touched and what to say.
B) Engage the family in social conversation to ease them into the milieu.
C) Use visiting hours to explain to the family the general status of the patient.
D) Leave the family to adjust to the situation when they are ready.
16. A critically ill patient is not expected to survive this admission. The family asks the nurse

om
how the patient is doing. When answering this question, what should the nurse include?
A) Emphasize that the patient is young and strong and may still survive.

.c
B) Refer the family to the physician for all details and answers.

ep
C) Give specific information such as descending trends in parameters.

pr
D) Ask if the family has determined which funeral home will be called.

st
17. A patients family is exhibiting increasingly impaired coping as the patients condition
deteriorates. The nurse asks the family to state the biggest concern from their perspective.
te
What is the most important rationale for this question?
ng

A) The question indicates active listening on the part of the nurse.


B) The question is used as a way to validate the familys knowledge.
si

C) The question clarifies the nurses understanding of current family needs.


ur

D) The question promotes problem definition, which helps define the degree of family
yn

understanding.
18. The nurse recommends that the family of a critically ill patient seek help from the Critical
.m

Care Family Assistance Program. What benefit for the family does the nurse anticipate?
w

A) Reduction of health care cost


w

B) More physical comfort


w

C) Multidisciplinary support
D) Health promotion information
19. While interacting with the family of a critically ill patient, the nurse suggests that the
family must be feeling very anxious and perhaps angry. How does this nursing action
benefit the family?
A) Removes the focus of the conversation from the patient
B) Focusing on feelings helps the family avoid delayed grief and unhealthy coping
C) Gives validation of need for psychological counseling
D) Reduces family insistence for patient progress information
20. The nurse is caring for a patient from a very different cultural group. In delivering care,
how can the nurse best demonstrate cultural sensitivity?
A) Ask the family about their cultural beliefs and customs that may apply.
B) Assume that the patient and family will adjust to the hospital culture.
C) Inform the patient and family that the routines of the hospital take precedence.

om
D) Do a literature search on the patients culture to determine beliefs.

.c
Answer Key

ep
1. A

pr
2. C
3. D
4. B, E, F
st
te
5. A
ng

6. C
si

7. D
ur

8. B
yn

9. C
.m

10. D
11. B
w

12. C
w

13. D
w

14. A
15. A
16. C
17. D
18. C
19. B
20. A

Chapter 4 Patient and Family Education in Critical Care

1. An elderly man whose wife has just been admitted to the ICU following congestive heart
failure is concerned about how passage of the Affordable Care Act will impact the care of
his wife. Which of the following are expected outcomes of this legislation that it would
be appropriate for the nurse to share with the man? Select all that apply.
A) The new legislation will result in a shortage of experienced critical care nurses.

om
B) The new legislation will require health care workers to meet higher quality standards
related to patient care.
C) The new legislation will make it more challenging to meet the educational goals of

.c
patients and families.

ep
D) The new legislation will increase the length of stay of patients in the hospital.

pr
E) The new legislation will shift the payment structure for hospitals and health care
providers from a traditional fee-for-service model to an incentive model.
st
2. A young couple whose 5-year-old daughter has been admitted to the ICU approaches the
te
nurse with looks of concern on their faces. They express frustration to the nurse that they
have not been able to speak with either the physician or the surgeon and are confused as
ng

to what the next steps are for their daughters treatment. What would be the best
intervention for the nurse to make in this situation?
si

A) Offer to get ice chips for the couple to give to their daughter to empower them.
ur

B) Make sure that they have the cell phone numbers of the physician and surgeon.
yn

C) Teach the couple about the pathophysiology of the daughters disease.


.m

D) Arrange a patient care conference with the couple and the health care team.
3. A nurse sees a group of physicians who are making teaching rounds in the hall of the ICU
w

heading toward the room of one of her patients. The patient, who currently has a visitor,
w

has given approval in the past to have teaching groups visit. What should the nurse do in
this situation?
w

A) Explain to the group of physicians that the patient currently has a visitor and ask
whether they could come by later.
B) Ask the visitor to leave so that the teaching group can discuss the patients case.
C) Allow the teaching group to enter the patients room, as he has already given
approval for them to visit.
D) Instruct the physicians to give clear explanations of the medical jargon they use.
4. The nurse is working with a patient from India who is recovering from a myocardial
infarction. When the nurse asks the patient whether she has had a myocardial infarction
before, the patient seems confused and appears not to understand the nurse, although the
patient does speak English. Which of the 4 Cs of Culture should the nurse use to better
communicate with this patient?
A) Call
B) Cause
C) Cope

om
D) Concern
5. A nurse needs to obtain informed consent from a deaf patient before a spinal tap
procedure is performed. Which of the following would be the best method for the nurse to

.c
use to ensure effective communication?

ep
A) Explain the procedure verbally, speaking slowly so that the patient can read lips.

pr
B) Have a trained oral interpreter interpret for the nurse.
C) Have the patient carefully read a printed copy of the informed consent document.
D)
st
Use diagrams to explain to the patient the details of the procedure.
te
6. A nurse is explaining to a patient how radiation therapy works to kill cancer cells. She
ng

begins by explaining how there are different types of cells in the body that reproduce at
different rates. She then explains what cancer cells are and how they reproduce. Finally,
si

she explains how radiation therapy uniquely targets cancer cells. This approach takes
advantage of which domain of learning?
ur

A) Affective
yn

B) Psychomotor
.m

C) Cognitive
w

D) Demonstration/return demonstration
w

7. A young man is recovering from anaphylactic shock caused by a bee sting. The nurse is
trying to instruct the patient on how to use an epinephrine autoinjector (EpiPen), but the
w

patient seems uninterested in learning. The nurse then explains how having the EpiPen
with him at all times and knowing how to use it could not only save his life someday but
also will give him a greater sense of security and safety. Which adult learning principle is
the nurse using? Select all that apply.
A) The learners self-concept
B) The learners life experience
C) Readiness to learn
D) Motivation to learn
E) The need to know
8. In attempting to teach a patient how to clean around the surgical sutures on his abdomen
upon discharge to home, the nurse determines that applying the principle of the learners
self-concept would be most effective with this patient. Which of the following is the best
example of an application of that principle in this situation?
A) Relating to the patient a story about another patient who failed to properly clean his
sutures and the outcome

om
B) Explaining that failure to properly clean around the suture site could result in serious
infection
C) Commenting that learning to properly clean his suture site could give the man skills

.c
that would better prepare him for a career in health care

ep
D) Mentioning to the patient that, if he would prefer, he can watch a video on the
hospitals website on how to clean around a suture site

pr
9. A nurse needs to explain to a patient about the possible side effects the patient may
st
experience related to the pain medication she is now beginning while in the ICU and
which she will be continuing upon discharge. Which method would be the most effective
te
way to teach this to the patient?
ng

A) In a planned teaching session, in which the nurse covers medication-related side


effects, dietary restrictions, and activity restrictions
si

B) Via a brochure that the patient can take with her on discharge
ur

C) By briefly explaining the side effects while administering the medication to the
yn

patient
D) By relating a story about another patient who had a severe adverse reaction to this
.m

medication
10. A nurse needs to evaluate a patients understanding of how to administer an IV medication
w

at home. Which of the following would be the best method for evaluation?
w

A) The nurse explaining the procedure to the patient and family using diagrams
w

B) The nurse having the patient and family members demonstrate the procedure
themselves
C) The nurse explaining the procedure while performing it on the patient
D) The nurse referring the patient to a computer-based educational library that has an
interactive program
11. Teaching patients and families is an important part of critical care nursing. What factor in
todays critical care unit is a barrier to this education function?
A) Large numbers of inexperienced nurses
B) Serious illness of patients
C) Increased computer support
D) Use of specialty educators
12. The nurse is caring for a critically ill patient. On the previous shift, a nurse documented
unable to teach due to critical illness. What is the best nursing action by the current nurse
to address the patients teaching and learning needs?
A) Realize that the patient is too ill to accept teaching at this time.

om
B) Look for opportunities for teaching, such a procedure explanation.
C) Focus all teaching efforts on the family to the exclusion of the patient.

.c
D) Alter the plan of care to delay teaching until transfer to step-down unit.

ep
13. After several weeks, a critically ill patient has progressed well and is being transferred to

pr
a step-down unit. What normal patient response does the nurse anticipate?
A) Relief
st
te
B) Thankfulness
C) Anxiety
ng

D) Indifference
si

14. The nurse is teaching a patient and family in the patients critical care room. The critical
ur

care unit is busy and noisy. What nursing action will best enhance learning?
yn

A) Explain the material simply using simple terms.


B) Give all explanations via commercial teaching brochures.
.m

C) Close the door to the patients room.


w

D) Ask the family not to interrupt with questions.


w

15. The nurse is teaching a patient who is not a native English speaker. As the nurse gives
w

complex explanations, the patient nods and smiles. What is the best nursing intervention
to ensure patient understanding?
A) Assume that the patients nods indicate understanding.
B) Ask a family member to interpret to the patient.
C) Use pictures whenever possible in the teaching.
D) Ask the patient to restate the information conveyed.
16. The nurse is teaching a patient newly diagnosed with diabetes mellitus how to manage a
sliding-scale insulin protocol. What portion of this interaction indicates patient learning?
A) The sliding scale insulin protocol
B) Patients questions to the nurse
C) Nurses demonstration of use of the protocol
D) Patients verbalization of a flash of insight
17. When teaching a patient and family, the nurse wishes to use the affective domain of
learning. What nursing action is most likely to involve the affective domain?

om
A) Presenting facts from simple to complex
B) Giving clear directions about when to call the physician

.c
C) Using a nonthreatening approach

ep
D) Using demonstration/redemonstration approach

pr
18. The nurse is teaching a patient newly diagnosed with diabetes mellitus how to inject
st
insulin and chooses to use a demonstration/redemonstration technique. What domain of
learning is the nurse chiefly using?
te
A) Cognitive
ng

B) Affective
si

C) Psychomotor
ur

D) Educative
yn

19. The nurse is teaching a patient newly diagnosed with cardiovascular disease how to
reduce risk factors. The nurse begins by explaining why this information is important for
the patient. What principle of adult learning is the nurse applying?
.m

A) Need to know
w

B) Learners self-concept
w

C) Learners life experience


w

D) Motivation to learn
20. The nurse is teaching a group of patients newly diagnosed with cardiovascular disease.
What action demonstrates application of the adult learning principle of learners life
experience?
A) Beginning with an explanation of why the material is important
B) Creating a learning situation that is self-directed and independent
C) Using case scenarios and problem-solving exercises
D) Applying content to real-life situations and actions
21. The nurse is assessing learning by a critically ill patient and family. What is the best
method of assessment?
A) Written test
B) Specific questions
C) Open-ended questions
D) Literacy assessment

om
Answer Key

.c
1. B, E

ep
2. D

pr
3. A
4. A
5. B
st
te
6. C
ng

7. D, E
si

8. D
ur

9. C
yn

10. B
.m

11. A
12. B
w

13. C
w

14. C
w

15. D
16. D
17. C
18. C
19. A
20. C
21. C

Chapter 5 Relieving Pain and Providing Comfort

1. A patient is in the ICU after suffering multiple trauma from a car wreck that occurred 24
hours ago. The patient feels pain from many sources, including lacerations on the arm, a
fractured femur, a tension headache from the stress of the accident, lower back pain from
a sports injury 30 years ago, nasal irritation from an endotracheal tube, and joint aches
from ongoing arthritis. The nurse recognizes which of the following as the most likely
instances of chronic pain? Select all that apply.

om
A) Lacerations on arm
B) Fractured femur

.c
C) Tension headache

ep
D) Lower back pain from old sports injury

pr
E) Irritation in throat from endotracheal tube
F) Arthritis in joints
st
2. A Nepali man is in the ICU recovering from spinal surgery to remove a malignant tumor.
te
He does not speak English, and struggles to communicate with the nurse by using hand
ng

motions. His family is constantly at his bedside, speaking with him in Nepalese.
Frequently, he puts on headphones and listens to music on his MP3 player. His wife
occasionally massages his feet. Which of the following factors is most likely exacerbating
si

this patients pain?


ur

A) Inability to communicate with the nurse


yn

B) Constant presence of his family


.m

C) Listening to music
D) His wife massaging his feet
w

3. A nurse is caring for a patient in the ICU who is recovering from coronary artery bypass
w

graft surgery. Over the course of the next few days, the nurse will be responsible for
w

changing a dressing over the surgical incision site, having the patient change positions in
bed, assisting with tracheal suctioning, and assisting with drain removal. The nurse
should anticipate that the patient will most likely perceive pain resulting from these
procedures in which order, from least painful to most?
A) Changing the dressing, position change in bed, tracheal suctioning, drain removal
B) Position change in bed, changing the dressing, drain removal, tracheal suctioning
C) Drain removal, tracheal suctioning, position change in bed, changing the dressing
D) Tracheal suctioning, position change in bed, changing the dressing, drain removal
4. A patient in the ICU is receiving intravenous opioid analgesia following myocardial
infarction. Despite receiving continuous infusion of the opioid, the patient is grimacing
and asks for an increase in the medication level. Consulting the medical chart, the nurse
recognizes that this patient has a history of opioid addiction. What would be the most
appropriate intervention?
A) Immediately take the patient off of the opioid and give him Tylenol.
B) Leave the patient on the opioid at the current dose level.
C) Increase the dose of opioid to provide more effective pain relief.

om
D) Reduce the dose of opioid and offer to turn on the television as a distraction from
the pain.
5. A patient with no history of opioid abuse is receiving opioid analgesia intravenously in

.c
the ICU for severe pain from extensive third-degree burns. The patient and his family are

ep
concerned about the likelihood of developing an addiction to the medication. What would
be the best response on the part of the nurse?

pr
A) Stop the opioid infusion immediately and use only nonpharmacological means of
analgesia.
B)
st
Begin tapering off the medication gradually.
te
C) Explain that the risk for addiction is low and that opioids are necessary for the
ng

patients level of pain.


D) Explain that addiction is likely but cannot be avoided and suggest a rehabilitation
si

program following discharge.


ur

6. A nurse is working with an elderly patient with Alzheimers disease and congestive heart
yn

failure in the ICU. Which of the following methods of pain assessment would be essential
to include in this situation, according to the American Geriatric Society?
.m

A) No assessment is appropriate; the patient should be started on a standardized


protocol for analgesia for Alzheimers patients
w

B) The verbal zero to ten scale


w

C) The word descriptor scale


w

D) Assessment of behavior and family observations


7. A patient in the ICU with renal dysfunction is to begin receiving intravenous opioids and
a nonsteroidal anti-inflammatory drug (NSAID) for severe pain. The medication is
expected to be administered for at least seven days. Which NSAID would be most
appropriate for this patient? Select all that apply.
A) Ketorolac (Toradol)
B) Indomethacin (Indocin)
C) Celecoxib (Celebrex)
D) Rofecoxib (Vioxx)
8. A nurse is working with a cancer patient who has chronic pain related to her illness. The
nurse must administer fentanyl to the patient in the form of a transdermal patch. Which of
the following should the nurse do in administering this medication?
A) Use lotion to prepare the skin where the patch will be applied.
B) Wear gloves when handling the patch.
C) Apply the patch over a wound or abrasion, if possible, to maximize absorption.

om
D) Leave old patches on the patients bedside table for the doctor to examine later.
9. A patient in the ICU is receiving an intravenous opioid infusion for pain, but is

.c
experiencing anxiety due to being on mechanical ventilation. The physician has decided

ep
to prescribe a sedative for the patient. The patients health history indicates an allergy to
soy products. Which sedative should be avoided in this situation?

pr
A) Propofol
B) Midazolom st
te
C) Diazepam
ng

D) Lorazepam
10. An elderly patient in the ICU is receiving intravenous opioid analgesia for pain. The
si

nurse observes that the patients respiratory rate has decreased to 8 breaths per minute.
ur

Which nursing intervention would be most appropriate?


A) Administer naloxone intravenously very slowly.
yn

B) Administer diazepam immediately.


.m

C) Increase the dose of opioid.


w

D) Massage the patients feet to stimulate her breathing.


w

11. The nurse is caring for an otherwise healthy victim of a motor vehicle crash who is
experiencing considerable pain. What factor indicates that the patient may be
w

experiencing acute pain?


A) It is associated with an acute and severe injury.
B) It is expected to resolve as the injury heals.
C) It requires treatment with intravenous opioids.
D) No chronic illnesses have been diagnosed.
12. The nurse is caring for a critically ill patient who has experienced multiple trauma. The
patient has high levels of pain from the injury and is receiving an intravenous opioid as
treatment. In managing the patients pain, what nursing action best demonstrates
understanding of other factors that exacerbate pain in the critically ill patient?
A) Provide supportive care without discussing it with the patient.
B) Limit visits to immediate family for a few minutes at a time.
C) Minimize care tasks during normal hours of sleep.
D) Give higher doses of intravenous opioid as needed.

om
13. Before turning and repositioning a critically ill patient, the nurse ensures that a pain
medication is administered. This action demonstrates the nurses understanding of what
phenomenon?

.c
A) Research has shown that critically ill patients perceive turning as a painful

ep
procedure.

pr
B) The patient is ordered to receive pain medication every 4 to 6 hours.
C) This patient becomes very stiff when turned and the medication will be relaxing.
D)
st
The nurse is not concerned that this patient will become addicted to the medication.
te
14. The patient is scheduled for a painful procedure. In addition to premedicating the patient
ng

with an opioid drug, what other nursing action is most likely to alleviate the pain?
si

A) Give intravenous midazolam (Versed).


ur

B) Monitor vital signs during the procedure.


C) Give explanations before and during the procedure.
yn

D) Ask the family to wait outside during the procedure.


.m

15. The nurse is caring for a patient who has required increasing doses of opioids to control
pain and expresses a concern that the patient is becoming addicted to the opioid. What
w

patient behavior would best support the nurses concern about addiction?
w

A) He needs more medication to control pain.


w

B) He has withdrawal symptoms when the medication is stopped.


C) He states that he dislikes the opioid as it makes him feel drugged.
D) He asks for another dose of the opioid to relieve anxiety and get a buzz.
16. The nurse is caring for a critically ill patient who has developed tolerance to an opioid
medication used for pain control. As the dose of the opioid is increased, the nurse
observes the patient closely for respiratory depression. If the patient has developed
tolerance to the opioid, what effect on the respiratory system does the nurse expect?
A) None
B) Depression
C) Stimulation
D) Exacerbation
17. The nurse is caring for a critically ill patient with serious pain and wishes to consult
national guidelines. What is the most comprehensive source for pain management
guidelines?
A) Web-based National Guideline Clearinghouse

om
B) American Association of Critical-Care Nurses
C) Society of Critical Care Medicine

.c
D) Agency for Healthcare Research and Quality

ep
18. The nurse is developing a policy and procedure for pain management in a critical care

pr
unit. Based on national standards, what should the nurse include?
A)
st
Pain assessment in critical care must depend on vital sign monitoring as patients are
not verbal.
te
B) Continuous intravenous opioids are preferred over as-needed dosing.
ng

C) Intravenous sedation of agitated patients takes priority over pain control.


si

D) Pain control is an independent nursing function and responsibility.


ur

19. According to national standards, pain in critically ill patients should be assessed at regular
intervals using a variety of methods. What statement about pain assessment is true?
yn

A) Absence of physical signs or behaviors is equivalent to absence of pain.


.m

B) Many of the factors in critical care combine to invalidate patient self-report of pain.
C) Behavioral observation and physiological parameters should be considered along
w

with the patients report.


w

D) The family has a more accurate assessment of pain than the patient.
w

20. The nurse is administering an intravenous opioid to manage a patients pain. What criteria
can the nurse use to determine the adequacy of therapy?
A) Minute ventilation is somewhat compromised.
B) Patient rates pain below 5 on scale of 1 to 10.
C) Minute ventilation is minimally compromised.
D) Patient rates pain below own predetermined goal.
21. A severely ill critical care patient is receiving intravenous opioids for pain management.
The physician adds a nonsteroidal anti-inflammatory drug (NSAID) to the patients plan
of care. The nurse understands that the most significant advantage of adding this drug is
what?
A) NSAIDs are cheaper than opioids while providing the same pain relief.
B) Inhibition of prostaglandin and histamine at the site of injury will relieve pain
without sedation.
C) NSAIDs are available without prescription.

om
D) Addiction and physical dependence are less of a problem with NSAIDs.
22. A critically ill patient is receiving acetaminophen (Tylenol) in combination with opioids
for pain management. Under what circumstances would the nurse question the use of

.c
acetaminophen?

ep
A) Normal liver function tests

pr
B) Low platelet levels
C) Relative hypothermia
D) Reduced pain levels
st
te
23. The nurse is caring for a critically ill patient with high levels of pain. To potentiate
ng

pharmacological pain relief, the nurse uses several nonpharmacological interventions.


What nursing strategies will be helpful in this situation? Select all that apply.
si

A) Frequent turning and repositioning


ur

B) Earphones with music of the patients choice


yn

C) Limiting visits to twice a day


.m

D) Using guided imagery and distraction


E) Teaching the quieting reflex
w

F) Using therapeutic touch


w
w

Answer Key

1. D, F
2. A
3. A
4. C
5. C
6. D
7. C, D
8. B
9. A
10. A
11. B
12. C

om
13. A
14. C

.c
15. D

ep
16. A

pr
17. A
18. B st
te
19. C
ng

20. D
21. B
si

22. C
ur

23. B, D, E, F
yn

Chapter 6 Palliative Care and End-of-Life Issues in Critical Care


.m

1. A nurse is working with a patient who is near death and experiencing delirium. Which of
w

the following would be an appropriate course of action for this nurse to take in caring for
this patient? Select all that apply.
w

A) Instruct the patient in pursed-lip breathing.


w

B) Provide oxygen support for the patient.


C) Arrange supportive psychotherapy.
D) Give the patient benzodiazepines.
E) Give the patient neuroleptics.
2. The husband of a terminally ill patient is upset at the sight of his wife continuing to linger
on the brink of death. He has heard of end-of-life sedation and wonders whether it would
be appropriate for his wife. Which of the following should the nurse mention as criteria
for use of end-of-life sedation? Select all that apply.
A) The patient is experiencing unbearable and unmanageable pain.
B) The patient is only hours or days away from death.
C) The patient has requested it.
D) It is standard procedure for patients with a terminal illness.
3. A patient in the ICU has entered a coma state, and someone must make a decision about
whether to continue life-supporting measures for the patient. No living will exists for this

om
patient. Who is the person legally authorized to make this decision?
A) The patients wife, who is the executor of his estate

.c
B) The patients brother, who is his designated health care proxy

ep
C) The patients physician, who has been managing his care from admittance

pr
D) The patients nurse, who has been assigned to him on a daily basis

st
4. The mother of a patient has requested that she and the rest of the family be allowed to be
present in the patients room in the event that the patient requires resuscitation. The nurse,
te
however, is reluctant to comply with this. What is the most likely reason she does not
want the family in the room during resuscitation?
ng

A) Family members who are present during resuscitations may experience more
si

anxiety.
ur

B) Family members present during resuscitation are more likely to sue.


C) The nurse is new and is not comfortable with having the family present.
yn

D) A dedicated staff person is required to attend to the family during resuscitation.


.m

5. A teenage boy is near death in the ICU after being involved in a severe car accident. His
mother and stepfather are in his room visiting him now. His father and stepmother have
w

arrived in the waiting room and are asking to see the patient. A few days ago, the nurse
w

observed the boys father and mother arguing loudly in the patients room. She has also
observed that the boys muscles tighten and his breathing and heart rate increase when his
w

father is near his bed. Which of the following would be the best action for the nurse to
take?
A) Ask the father and stepmother to wait until the current visitors have left.
B) Ask the mother and stepfather to leave so that the father and stepmother can visit.
C) Allow the father and stepmother to go into the patients room but accompany them.
D) Tell the father and stepmother that they are not allowed to visit the patient.
6. A patient in the ICU has ovarian cancer that has metastasized to her stomach and other
organs. The physician believes the patient only has days to survive. Which of the
following would be a realistic goal for the care of the patient that the nurse could suggest
to the family?
A) Eliminate the cancer cells by starting the patient on chemotherapy.
B) Slow the rate of growth of the cancer by starting the patient on radiation therapy.
C) Keep the patient free of pain by increasing the patients pain medication.
D) Restore gastrointestinal function to the client via surgical intervention.

om
7. A 10-year-old girl has just died in the ICU. The nurse is now questioning the patients
physician to learn more details about the patients death to communicate to the family.
Which stage of notification is the nurse currently in?

.c
A) Preparation

ep
B) Inform

pr
C) Support
D) Afterwards st
te
8. The family of a patient who is near death makes the difficult decision to allow the
physician to administer a pain-relieving drug to the patient with the intention of making
ng

the patient more comfortable, despite the fact that the drug may increase the risk for
complications that could lead to the patients death. The principle involved in the familys
si

decision is known as which of the following?


ur

A) Principle of moral distress


yn

B) Principle of palliative care


C) Principle of unintended consequences
.m

D) Principle of double effect


w

9. Which of the following is the best example of moral distress?


w

A) A patients family has decided to end mechanical ventilation of the patient, but the
w

nurse has trouble implementing their decision.


B) A patients family has decided to continue mechanical ventilation for the patient, but
the nurse disagrees and discontinues the ventilation on her own initiative.
C) A patients family has decided to end mechanical ventilation of the patient, and the
nurse implements their decision but later feels guilty.
D) A patients family has decided to continue mechanical ventilation for the patient, and
the nurse implements their decision, although she disagrees with it.
10. A patient who is near the end of his life is to be extubated, according to the wishes of his
family. Which of the following would be the best action for the nurse to take to support
the patient?
A) Ask the family to leave the room during the extubation.
B) Hold a family conference to ask the family to reconsider their decision.
C) Administer opioids to the patient to reduce pain and discomfort.
D) Administer antibiotics to the patient to prevent infection.
11. The nurse is caring for a critically ill patient who is expected to die despite the best efforts

om
of the health care team. What aspect of a critical care unit may most interfere with
effective nursing care of this dying patient?

.c
A) View of patient as disease or in terms of technology

ep
B) Critical illness has a high expectation of death
C) Constant family presence at the bedside

pr
D) Multidisciplinary management of patient
st
12. The nurse cares for critically ill patients in a busy trauma unit. The nurse manager has
te
instituted a program to incorporate principles of palliative care into the care of all patients
in the unit. What is the best reason for this inclusion?
ng

A) Most of the patients in critical care will die, so palliative care will be necessary.
si

B) Primary palliative care focuses on relief of suffering and improvement of quality of


ur

life.
C) This action has been mandated by the nurse manager and must be implemented.
yn

D) Palliation is another way of managing pain control in critical care.


.m

13. A patient has been admitted to critical care for management of exacerbation of a chronic
illness. During this admission, the patients condition deteriorates and death is deemed
w

imminent. A Do Not Resuscitate (DNR) order has been written and agreed to by the
w

family. Considering the principles of palliative care, what is the most appropriate nursing
action?
w

A) Transfer the patient to the step-down unit since DNR patients are not eligible for
critical care.
B) Encourage the family to reduce their visits so that they will not have to witness the
patients deterioration.
C) Assess the patient and family for specific spiritual needs at this phase of life and
death.
D) Discontinue all supportive care to hasten the inevitable death of the patient and
reduce costs.
14. The patient has severe pulmonary edema following an acute myocardial infarction and is
receiving intravenous diuretics to ease breathing. The nurse understands that this
intervention is considered to be what?
A) Definitive
B) Curative
C) Cause-and-effectbased

om
D) Palliative
15. The nurse is caring for a patient approaching the end of life. What symptom common at

.c
the end of life would the nurse most expect?

ep
A) Diminished pain
B) Hypotension

pr
C) Tachycardia
D) Dyspnea
st
te
16. The nurse is caring for a critically ill patient who is dying and complaining of dyspnea
ng

while receiving supplemental oxygen. What is the most effective nursing intervention to
alleviate the patients symptoms?
si

A) Increasing room temperature


ur

B) Encouraging unlimited visits


yn

C) Closing blinds and drapes


D) Providing a fan blowing on face
.m

17. The nurse is caring for a patient who is dying. The patient has persistent full depression.
w

What is the most appropriate nursing action?


w

A) Understand that depression is normal at the end of life.


w

B) Facilitate use of antidepressant medications.


C) Encourage the patient to discuss life achievements.
D) Increase family visiting time and frequency.
18. A nurse is caring for a critically ill patient who has identified an unrelated friend as his
closest relative. What is the nurses most appropriate action in this situation?
A) Treat the friend as the patients family.
B) Do not allow the friend to visit as he is not family.
C) Adhere to the legal definition of next of kin.
D) Identify the next-closest-related family member.
19. The nurse is preparing for a family conference with the family of a critically ill patient.
What will best help the nurse to prepare?
A) Complete knowledge about the status of the patient
B) Knowing which members of the family will attend
C) Knowing the goal of the conference

om
D) Skill in therapeutic communication
20. The nurse had been caring for a critically ill patient for several days, and the patient has
just died. After completing end-of-life care of the patient and family, the nurse asks to

.c
take a short break. What is the best response by the charge nurse?

ep
A) Ask the nurse to wait to take a break until after she admits another patient.

pr
B) Deny the nurse a break, as the unit is particularly busy and the nurse cannot be
spared.
C)
st
Arrange for the nurse to have psychological counseling for inability to handle stress.
te
D) Ask another staff nurse to cover this nurses duties and allow her to take a break.
ng

Answer Key
si
ur

1. D, E
yn

2. A, B
3. B
.m

4. C
w

5. A
w

6. C
w

7. A
8. D
9. A
10. C
11. A
12. B
13. C
14. D
15. D
16. D
17. B
18. A
19. C

om
20. D

.c
Chapter 7 Ethical Issues in Critical Care Nursing

ep
1. The physician has just told a patient that he has stage 4 lung cancer and likely has only
months to live. After the physician leaves, the patient, who is visibly shaken, asks the

pr
nurse, Couldnt the doctor be wrong? Is it really that bad? The nurse explains to him that,
although there is no way to know for certain how the disease will progress, the stage of
st
his lung cancer is the most serious. The nurse also indicates that the cancer has spread to
other organs in his body, meaning that the odds of recovery are not good. The patient then
te
asks that the nurse not tell his wife, who is in the waiting room, about the diagnosis just
ng

yet. Later the patients wife enters the room and, seeing that he is asleep, asks the nurse if
there is any update on the patients condition. The nurse explains that the doctor talked to
the patient earlier and that the patient can provide details once he wakes up. Which
si

ethical principle or principles has the nurse exercised in this situation? Select all that
ur

apply.
yn

A) Justice
B) Fidelity
.m

C) Veracity
w

D) Nonmaleficence
w

2. Before administering a new medication to a patient, the nurse explains to her the adverse
w

effects she may experience as a result of the medication and asks whether the patient has
any questions about the medication. When the patient indicates that she understands the
risks involved, the nurse has her sign a document and proceeds with the treatment. The
nurses action is an example of which of the following?
A) Obtaining informed consent
B) Ensuring confidentiality
C) Observing the principle of nonmaleficence
D) Acting with fairness
3. A patient is at severe risk of forming life-threatening clots in his thoracic region
following surgery. The physician has prescribed an anticoagulant medication. The patient
has had a negative experience with anticoagulants in the past and refuses the medication.
Which two ethical principles are in conflict with each other in this situation?
A) Fidelity vs. justice
B) Maleficence vs. beneficence
C) Veracity vs. autonomy
D) Beneficence vs. autonomy

om
4. A patient in the ICU recently experienced a myocardial infarction that resulted in an
aneurysm of the ventricular wall. The patients cardiovascular surgeon has determined that
the aneurysm is irreparable. In this situation, an attempt to repair the rupture via

.c
laparoscopic surgery would be considered which of the following?

ep
A) Palliative

pr
B) Curative
C) Medically futile
D) Beneficent
st
te
5. The family of a patient in a coma is struggling to decide whether to remove the patient
ng

from a ventilator. What would be the most appropriate intervention?


si

A) Explain the statistical odds that the patient will regain consciousness.
ur

B) Facilitate a care conference with the family.


C) Leave the family alone to make their decision.
yn

D) Distribute copies of the American Nurses Association Code of Ethics to the family.
.m

6. A patient complains of severe pain and requests an increase in dose of her pain
medication. Her husband, however, does not want the patients pain medication increased,
w

as he is worried about her becoming dependent on the medication. Which principle(s)


w

from the ANA Code of Ethics would be most appropriate for the nurse to apply in this
situation? Select all that apply.
w

A) The nurses primary commitment is to the patient.


B) The nurse collaborates with other health professionals and the public.
C) The nurse owes the same duties to self as to others.
D) The nurse promotes, advocates for, and strives to protect the health, safety, and
rights of the patient.
7. A nurse learns that a patient in the ICU who is scheduled to undergo a liver transplant has
a history of alcoholism. Which principle or principles from the ANA Code of Ethics
would be most appropriate for the nurse to apply while caring for this patient? Select all
that apply.
A) The nurse practices with compassion and respect unrestricted by considerations of
the nature of the health problem.
B) The nurses primary commitment is to the patient.
C) The nurse promotes, advocates for, and strives to protect the health, safety, and
rights of the patient.
D) The nurse participates in establishing, maintaining, and improving health care

om
environments.
8. A nurse works in an ICU in which she routinely sees physicians neglecting the care of
patients who do not have health insurance. She feels angry about the unfairness of this

.c
situation but also powerless to do anything because she fears the loss of her job. Which of

ep
the following best describes what the nurse is experiencing?
A) Medical futility

pr
B) Nonmaleficence
C) Moral distress st
te
D) Paternalism
ng

9. An elderly patient who is about to have open-heart surgery has verbally instructed the
nurse that she does not want her life to be extended by a ventilator after the surgery. The
si

patients advance directive, however, indicates that she would prefer to receive all life-
support measures. The nurse suspects that the patient has dementia. Which of the
ur

following questions would be most appropriate for the nurse to ask in the assessment
yn

stage of ethical decision making?


A) What are the ethical issues related to continuing or removing life-support measures
.m

for this patient?


B) Who should be involved in making the decision regarding life-support measures for
w

this patient?
w

C) Is this patient mentally competent to make decisions about use of life support?
w

D) What educational changes can be made in the ICU to resolve similar ethical
challenges related to life support measures in the future?
10. A patient who had provided an advance directive indicating that he should not be
resuscitated was allowed to die when his heart stopped beating shortly after his admission
to the ICU. The family of the patient has disputed the authenticity of the advance
directive and filed a complaint with the hospital. Which intervention would be most
appropriate to address this ethical dilemma?
A) Ethics rounds involving the medical staff involved in the patients care
B) Review by the hospitals institutional ethics committee
C) An individual patient ethics conference with the family
D) Review of the ANA Code of Ethics by the patients nurse
11. The nurse has learned about the application of ethical principles in nursing. How does this
knowledge facilitate the practice of nursing for the individual nurse?
A) It is mandated by accreditation agencies.
B) It is a component of bioethical theory.

om
C) It assists in defining good nursing care.
D) It assists in determining the legal basis of decisions.

.c
12. The family wishes to continue all aspects of care for a terminally and critically ill patient
in the interests of justice and nonmaleficence. The health care team believes that further

ep
sophisticated and technical interventions are futile and that the focus of care should be
shifted to provision of comfort, dignity, and palliative care. Neither the family nor the

pr
health care team is willing to compromise any portion of their stances. The patient is
unresponsive. This situation is an example of what kind of problem?
A) Ethical dilemma
st
te
B) Recalcitrant family
ng

C) Biotechnology effects
si

D) Paternalism
ur

13. The nurse consistently applies guidelines for safe administration of medications during
patient care. This practice is an example of the application of what ethical principle?
yn

A) Justice
.m

B) Veracity
w

C) Beneficence
w

D) Paternalism
w

14. In determining which patient is to be transferred from the CCU to make room for a new
admission, the nurse considers the relative benefit of continued CCU care related to
eventual outcomes for each patient and compares them. The nurse does not consider
factors such as ethnicity or socioeconomic status. This situation is an example of
application of which ethical principle?
A) Paternalism
B) Veracity
C) Beneficence
D) Justice
15. The health care team is considering a change in the treatment plan for a critically ill
patient. As this change is considered, the dangers and possible outcomes for continuing
with the same plan of care as well as those for the proposed change are thoroughly
discussed. The beliefs and values of the patient and family are included. What is this
situation an example of?
A) Inability to make independent decisions
B) Positive leadership action by the nurse

om
C) Application of informed consent
D) Riskbenefit analysis

.c
16. A patient is admitted to the CCU with active advance directives that include refusal of
enteral feeding by tubes. When the physician writes an order for the insertion of a feeding

ep
tube, the nurse refuses to comply. This refusal is an example of adherence to what ethical
principle?

pr
A) Nursing Practice Act
B) Patient Bill of Rights st
te
C) Patient autonomy
ng

D) Patient advocacy
si

17. While caring for a critically ill patient, the nurse identifies an ethical dilemma and seeks
the advice of the agencys Ethics Committee. This scenario is an example of what kind of
ur

nursing action?
yn

A) Inability to make independent decisions


B) Behavior mandated by protocols
.m

C) Appropriate use of resources


w

D) Fear of autonomous action


w

18. The family of a critically ill patient has said that the patient would not have wanted
w

prolonged life support. The patient has been ventilator-dependent for several weeks and is
not expected to improve. The physician states that, as the expert in health care, he knows
best, and the patient will remain on the ventilator. What behavior is the physician
exercising?
A) Routine medical care
B) Respect for autonomy
C) Expert paternalism
D) Biomedical ethics
19. In delivering patient care, the nurse bases a decision upon the ANA Code of Ethics for
Nurses. This is an example of what?
A) Inability to make independent decisions
B) Behavior mandated by protocols
C) Appropriate use of resources
D) Fear of autonomous action
20. While caring for a critically ill patient, the nurse observes the family discussing funeral
arrangements across the bed of the patient, who is intubated and sedated but awake. The

om
family signs Do Not Resuscitate (DNR) request paperwork and leaves. The patient
manages to communicate to the nurse that he does not want to die. What is the most
appropriate nursing action?

.c
A) Adhere to the wishes of the family, since the patient is probably incompetent.

ep
B) Call a multidisciplinary conference with the family to resolve this conflict.

pr
C) Destroy the Do Not Resuscitate paperwork, since the patient wants to live.
D) st
Apply the ethical principles of nonmaleficence and paternalism to the situation.
te
21. Before a surgical procedure, the patient signs a document called an informed consent.
What ethical principle is central to the use of informed consent?
ng

A) Autonomy
si

B) Fidelity
ur

C) Nonmaleficence
yn

D) Beneficence
.m

Answer Key
w

1. B, C
w

2. A
w

3. D
4. C
5. B
6. A, D
7. A, B, C
8. C
9. C
10. B
11. C
12. A
13. C
14. D
15. D

om
16. D
17. C

.c
18. C

ep
19. C

pr
20. B
21. A st
te
Chapter 8 Legal Issues in Critical Care Nursing
ng

1. A patients physician writes a medication order to administer 20 mEq of potassium


si

chloride to a patient. The ICU nurse misreads the order as 200 mEq of potassium and
administers this amount of medication to the patient. As a result, the patient dies. This
ur

situation is an example of which of the following?


yn

A) A questionable medical order


B) Negligent supervision
.m

C) Ordinary negligence
w

D) Gross negligence
w

2. A family member of a patient files a complaint against an ICU nurse, claiming that the
w

nurse improperly transferred the patient from a wheelchair to the hospital bed, resulting in
a back injury to the patient. Assuming that the nurses due process rights are observed,
which of the following actions would be most advisable for the nurse?
A) Plan to appeal to a court if the State Board of Nursing rules against her.
B) Sue the State Board of Nursing if her license is suspended before she is found guilty.
C) Acquire legal counsel and have him or her question the boards witnesses.
D) Continue nursing practice if her license is suspended, as such a suspension violates
her constitutional rights.
3. The ex-husband of a patient asks the ICU nurse a question about the medical history of
the patient. The nurse retrieves the patients file, opens it, and relates some of the patients
personal information to the ex-husband. When the ex-husband asks to see the patients
file, the nurse hands it to him, although he is not authorized by the patient to see this
information. What is the most accurate characterization of the nurses action?
A) A criminal act against the patient
B) A breaking of the rule of personal liability
C) A violation of the HIPAA Privacy Rule

om
D) A case of respondeat superior
4. A nurse is accused of professional negligence because she improperly operated a
ventilator, resulting in a patients death. The prosecution presents an expert witness to

.c
show that operating such a ventilator is within the standard of care for a reasonable nurse.

ep
Which aspect of negligence is the prosecution attempting to establish?
A) Duty

pr
B) Breach of duty
C) Causation st
te
D) Damages
ng

5. A jury involved in a malpractice suit must decide how much to compensate a patient for
the emotional distress caused her by incomplete anesthesia during surgery. To which
si

element of malpractice is this decision related?


ur

A) Duty
yn

B) Breach of duty
C) Causation
.m

D) Damages
w

6. A nurse is going to volunteer her nursing skills and time at a hospital other than the one in
w

which she is employed. Which of the following is true regarding potential liability related
to the nurses actions while volunteering? Select all that apply.
w

A) The nurse should carry her own malpractice insurance.


B) The hospital at which she is volunteering will protect her with its own professional
liability insurance.
C) The hospital at which she is employed will protect her with its professional liability
insurance even when she is working off the job.
D) There is no liability protection available for a nurse who is volunteering on her own
time.
E) The doctrine of respondeat superior does not apply in this situation.
F) The captain of the ship doctrine applies in this situation.
7. A nurse is charged with medical malpractice because she neglected to attend to a patient
in the ICU whose heart stopped and whose heart monitor alarm was clearly heard by the
nurse. The jury finds that the hospital was liable because of understaffing the ICU and
that the nurse was liable because she should have responded to the patient whose heart
stopped, regardless of the fact that she had been ordered by a physician to tend to another
patient. Which types of liability did the jury uphold? Select all that apply.
A) Captain of the ship doctrine

om
B) Rule of personal liability
C) Corporate liability

.c
D) Criminal liability

ep
8. A physician has written a medication order for a patient that includes penicillin. When the

pr
nurse receives the order, she refuses to administer the medicine because she knows that
the patient is allergic to it. What is the most accurate way to describe this situation?
A) A questionable medication order st
te
B) A medication error
ng

C) A case of respondeat superior


si

D) A case of negligent supervision


ur

9. A patient in the ICU dies as a result of a defective defibrillator. The nurse did not become
aware of the defect until after the patients death. Which of the following is true in this
yn

situation?
A) Defibrillators are not currently regulated in the United States.
.m

B) The nurse is permitted but not required to report the malfunction to the FDA.
w

C) The nurse is required to report the malfunction to the FDA.


w

D) The nurse is likely liable in the patients death.


w

10. A 10-year-old girl has been admitted to the ICU after she suffered a severe laceration on
her arm. Due to heavy hemorrhaging, the child needs a blood transfusion. Her parents,
however, who are Jehovahs Witnesses, do not believe in blood transfusions and refuse the
treatment. What would be the best action for the nurse to take?
A) Assist the physician in performing a blood transfusion for the girl, against her
parents wishes.
B) Refuse to assist the physician in performing a blood transfusion, to respect the
parents wishes.
C) Consult the hospitals risk management department before acting.
D) Schedule a conference with the parents to explain why an infusion is medically
necessary.
11. While caring for patients, the nurse completely documents the care given. This nursing
action is defined by what kind of law?
A) Administrative
B) Civil
C) Tort

om
D) Criminal
12. Nursing practice is defined by the Nursing Practice Act, administered by the State Board

.c
of Nursing. Which nursing behavior would be least likely to be addressed by the State

ep
Board of Nursing?
A) Qualifications for licensure

pr
B) Investigation of complaints
C) Nursing excellence awards
st
te
D) Protection of patient safety
ng

13. The physician has ordered a medication that is contraindicated for the patient. The nurse,
after discussion with the physician and the pharmacy, refuses to administer the
si

medication. Upon what legal principle does the nurse base this refusal?
ur

A) Respondeat superior
yn

B) Captain of the ship


C) Corporate liability
.m

D) Rule of personal liability


w

14. The nurse caring for a sedated patient leaves the side rails down when leaving the room
w

and the patient falls, breaking her hip. Why is this considered to be malpractice by
negligence?
w

A) The patient had requested the side rails be left down.


B) The nurse assumed that the family would watch the patient.
C) This is breach of duty resulting in patient harm.
D) There will probably be a suit brought by the family.
15. A nurse has been called before the State Board of Nursing for accusations of malpractice.
As part of his defense, the nurse states that he was unaware of several provisions of the
Nursing Practice Act. Why is this defense statement valid or invalid?
A) Invalid: The nurse has a legal duty to know the contents of the Nursing Practice Act.
B) Invalid: Nursing Practice Acts vary widely in content from version to version and
state to state.
C) Valid: The nurse did not know he was violating the Nursing Practice Act at the time
of his actions.

om
D) Valid: The nurse was under the supervision of his charge nurse at the time of his
actions
16. The nurse is caring for a critically ill patient. Which action by the nurse could most likely

.c
be considered malpractice?

ep
A) Assessing for allergies before administering medications

pr
B) Consistent failure to complete adequate hand hygiene
C) Strict adherence to the visiting hours policies of the unit
D)
st
Encouraging the patient to ask questions of the physician
te
17. The nurse has been accused of negligence resulting in patient harm. She was not informed
ng

about the complaint, and her employment was terminated before a hearing before the
State Board of Nursing. What best legal recourse does the nurse have?
si

A) Safe harbor provision in state law


ur

B) Fifth Amendment of the U.S. Constitution


yn

C) Civil and tort law allows the nurse to sue the accuser.
.m

D) Automatic dismissal of case by the State Board of Nursing


18. A nurse who commonly works on a general medical-surgical floor is asked to work
w

briefly in a critical care unit. The charge nurse, based on the legal principle of vicarious
w

liability, should give this nurse which patient assignment?


w

A) Admit day-of-surgery patient after open heart surgery


B) Three stable patients awaiting transfer to medical units
C) Patient with hemodynamic monitoring and unstable vital signs
D) Patient whose care requires frequent titration of vasoactive medications
19. After receiving an injury as a result of negligent care by a physician, the patient brings
suit against the hospital, the physician, and all of the members of the health care team
who participated in his care. Based on the principle of respondeat superior, the hospital is
liable for the actions of whom?
A) Nurse working in her usual assignment at hospital
B) Non-employee physician practicing in the hospital
C) Nurse on temporary assignment from an agency
D) Nurse doing volunteer work at another agency
20. A patient in CCU is critically and terminally ill. The family has made a written request
that no cardiopulmonary resuscitation (CPR) take place. The patient is well known to the
physician, who does not agree with the familys assessment of the situation. While the
physician is discussing the situation with the family and before agreement, the patient

om
experiences a cardiopulmonary arrest. Legally, what is the most appropriate nursing
action?
A) Proceed as though a DNR order has been written, since such an order is anticipated.

.c
B) Take no action and page the patients physician to ask for an immediate decision

ep
over the telephone.

pr
C) Initiate CPR and page the patients physician to come to the care unit.
D) Take no action and page the patients family to return to the care unit immediately.
st
21. A patient has been critically ill for several weeks and a Do Not Resuscitate (DNR) order
te
was written several days ago. The patients status has improved, and he is now eligible for
ng

transfer to a general nursing unit and is awake and alert. What is the most appropriate
nursing action?
si

A) Initiate risk-reduction teaching to prevent recurrence of instability.


ur

B) Ask the patient, family, and physician to review the DNR order.
yn

C) Increase the frequency of monitoring of vital signs to ensure safety.


D) Encourage the patient to get more rest in preparation for transfer.
.m

22. A terminally ill patient is admitted to the CCU for stabilization and palliative procedures.
w

The nurse encourages this awake and alert patient to complete paperwork for advance
directives. What is the best rationale for this nursing action?
w

A) Required by the admission protocol for the hospital and the unit
w

B) Allows the patient to express his or her choices for some treatment decisions
C) Prevents estranged family from interfering with competent patient decisions
D) Allows the physician to apply his or her best judgment for care decisions
23. A critical care nurse has administered an intravenous sedative in an excessive dose to a
patient. The patient experiences a cardiopulmonary arrest, and the family complains to
the State Board of Nursing. After investigation and a hearing, the Board finds that the
nurse was guilty of malpractice and disciplines the nurse. This situation is an example of
what?
A) Medication errors are the most common reason for State Board of Nursing action.
B) An unusual and rare incidence of lack of judgment by a nurse in a stressful situation
C) Absence of the application of due process for the nurse by the State Board of
Nursing
D) A family looking for an opportunity for litigation

om
Answer Key

.c
1. C

ep
2. C
3. C

pr
4. A
5. D st
te
6. A, E
ng

7. B, C
si

8. A
ur

9. C
10. C
yn

11. A
.m

12. C
w

13. D
w

14. C
w

15. A
16. B
17. B
18. B
19. A
20. C
21. B
22. B
23. A

Chapter 9 Building a Professional Practice Model for Excellence in Critical Care Nursing

1. A nurse in the ICU not only is excellent in providing care for her patients but has also
earned the credential of CCRN and is tireless in lobbying for federal legislation that
protects patients rights. She is also the first nurse in the ICU to recognize problems with

om
process and to develop solutions. However, this nurses husband has begun to complain
that he never sees her, and she feels like she has lost touch with her teenage daughter.
Which professional practice attribute or attributes does this nurse demonstrate? Select all

.c
that apply.

ep
A) Passion
B) Mastery

pr
C) Balance
D) Action st
te
2. A nurse in the ICU is known for the personal care and attention he gives to each of his
ng

patients. He takes time to learn his patients names and the names of their families. He is
especially gentle when drawing blood from children or administering injections to older
patients. He is also good at leaving his work concerns at work and focusing on his family
si

when at home. However, his lack of interest in pursuing further credentials and his lack of
ur

initiative in tackling problems that arise in the ICU have limited his career opportunities.
Which professional practice attribute or attributes does this nurse demonstrate? Select all
yn

that apply.
.m

A) Values
B) Mastery
w

C) Balance
w

D) Action
w

3. A nurse in the ICU has a natural rapport with her patients, communicates well with her
colleagues, always strives to improve her practices by learning from more experienced
nurses, and always shows up for her shift on time. Which characteristic of this nurse is
the hallmark of excellence?
A) Natural rapport with patients
B) Effective communication with colleagues
C) Always striving to improve her practices
D) Punctuality
4. A nurse in the ICU is known for working hard to improve the care that she and others
provide. When asked what motivates her to work so hard, she explains that she has a
picture of what an ideal ICU looks likea place where patients feel well cared for, where
health professionals work seamlessly together, and where nurses derive deep satisfaction
from their work. Which element of practice excellence best characterizes this nurses
perspective?
A) Values
B) Action

om
C) Mastery
D) Vision

.c
5. A nurse in the ICU loves her job and is known to volunteer her time for fundraising for

ep
health care charities and for promoting public health initiatives in her community. She
also loves to share her knowledge of physiology and medicine with her patients.

pr
Unfortunately, however, she often shares information that is outdated or not supported by
evidence. Which element is this nurse most lacking?
A) Mastery
st
te
B) Passion
ng

C) Action
si

D) Vision
ur

6. A nurse in the ICU has earned several advanced credentials in nursing and is working on
her masters degree in nursing. Although she loves gaining knowledge, she often finds
yn

herself unmotivated in her day-to-day duties as a nurse. Which element of excellence in


practice is this nurse most lacking?
.m

A) Mastery
w

B) Values
w

C) Passion
w

D) Balance
7. An ICU has recently won the Beacon Award for Critical Care Excellence. Which element
of excellence most contributed to the ICU being recognized with this award?
A) Values
B) Vision
C) Mastery
D) Action
8. A nurse in the ICU has recently undertaken an initiative to improve oral care in the
critically ill patient. This is an example of which of the following?
A) Practice alert
B) Specialty certification
C) Personal mastery
D) Nursing professionalism
9. A nurse in the ICU is extremely efficient in performing her tasks but often realizes at the
end of the day that she has not taken the time to make eye contact with her patients.

om
Which element of practice excellence is this nurse lacking?
A) Action

.c
B) Balance

ep
C) Mastery

pr
D) Vision

st
10. A nurse takes a genuine interest in her patients and is highly attentive to their needs. She
consistently checks on her patients and takes time to listen to their concerns. Which
te
value, central to nursing, is this nurse demonstrating?
ng

A) Caring
B) Patience
si

C) Honesty
ur

D) Justice
yn

11. A critical care nurse defines herself as a professional and her practice as a profession.
What is the best justification for this perception?
.m

A) Perceived as such by the general public and by peers


w

B) Defined as such by the Nursing Practice Act


w

C) Requires a specialized body of knowledge and competent skills


w

D) The nurse has completed a baccalaureate degree in nursing.


12. Members of a profession are expected to engage in lifelong learning and continual
improvement. Which nursing behavior most clearly demonstrates these behaviors?
A) Attending mandatory annual skills validation at work
B) Organizing a fundraiser for diabetes mellitus research
C) Maintaining a subscription to several professional journals
D) Volunteering to take blood pressures at a health screening event
13. One of the behaviors necessary in the practice of critical care nursing is the ability to
think independently and to apply the principles of critical thinking. Which personal
characteristic or action is most likely to foster development of independent and critical
thinking?
A) Continuing ones education voluntarily
B) Seeking promotion within administrative structures
C) Adhering absolutely to policy and procedure rules

om
D) Delivering care based solely on physician orders
14. The nurse manager of an ICU is designing a career ladder and wants to include

.c
demonstration of professional development as criteria. Which nursing behavior most
clearly demonstrates independent professional development?

ep
A) Arrives to work on time and always accepts a difficult patient care assignment

pr
B) Volunteers to help others when the unit is particularly busy
C) st
Belongs to more than one professional organization and is an officer in one
te
D) Maintains current certification in basic life support for health care professionals
ng

15. A critical care nurse has a personal goal to become an excellent critical care nurse. What
does the nurse need to understand about the development of excellence?
si

A) Consists of strict attention to provision of patient and family safety


ur

B) Measuring criteria are dynamic and change as the nurse develops.


yn

C) Requires merely hard work and continued education


D) Mandated by employer rather than being a personal goal
.m

16. A nurse in a critical care unit observes that communication in the unit is not fully honest
w

and straightforward, in that people do not confront others who are not meeting expected
care standards. The nurse proposes an improvement strategy for this problem. What
w

outcome is most likely?


w

A) The less-than-competent staff members will be encouraged to leave.


B) Improved collaborative communication will result in improved patient safety.
C) Staff will have less need to check on each others work.
D) Patients families will make fewer complaints to the patient representative.
17. Part of establishing and fostering excellence is practicing in congruence with ones values.
What value central and unique to nursing is believed to be a primary drive toward
excellence in the profession of nursing?
A) Caring
B) Justice
C) Paternalism
D) Duty of the individual
18. A group of nurses is complaining about working conditions in their hospital and in
nursing in general. What strategy could these nurses use to improve the situation?
A) Articulate a vision of the future to the administration and policymakers.

om
B) Continue to complain in the hopes that the administration will do something.
C) Refuse to serve on planning committees since conditions are so bad.

.c
D) Seek employment elsewhere to avoid current conditions.

ep
19. The critical care nurse is unable to read a physicians writing. From prior experience, the
nurse knows that this physician is likely to express anger and use inappropriate language

pr
when called with questions about his orders. What does this situation illustrate about the
development of excellence in critical care nursing?
A)
st
A common stumbling block for nurses is inappropriate and abusive treatment by
te
others.
ng

B) Part of the development of excellence is mastery of skilled communication under


stress.
si

C) Physicians are not part of the development of excellence in critical care nursing
ur

practice.
D) This situation demonstrates the need to adhere to protocols and not the development
yn

of excellence.
.m

20. One of the goals of an ICU is that all nurses be certified in a specialty. What is the most
significant benefit that specialty certification has been shown to have?
w

A) Usually results in a pay raise in forward-thinking agencies


w

B) Can take the place of continued education and is less expensive


w

C) Validates and recognizes development of personal excellence


D) Adds a cachet to ones rsum and achievement records

Answer Key

1. A, B, D
2. A, C
3. C
4. D
5. A
6. C
7. D
8. A
9. B

om
10. A
11. C

.c
12. C

ep
13. A

pr
14. C
15. B st
te
16. B
ng

17. A
18. A
si

19. B
ur

20. C
yn

Chapter 10 The Critically Ill Pediatric Patient


.m

1. A child in the ICU exhibits tachycardia, tachypnea, hypertension, and low pulse pressure
w

in the extremities. Which of these signs is the best indicator of inadequate perfusion of
blood?
w

A) Tachycardia
w

B) Tachypnea
C) Hypertension
D) Low pulse pressure
2. A 12-month-old child is admitted to the ICU after being found abandoned in a hot car.
The child is responsive, but the nurse notices that the anterior fontanelle of the child is
sunken. Which of the following conditions should the nurse suspect in this situation?
A) Dehydration
B) Increased intracranial pressure
C) Fluid overload
D) Hypertension
3. An ICU nurse, in testing the reflexes of a 3-month-old baby girl, strokes the lateral aspect
of the sole of the foot to test for possible nerve damage. In response, the child fans her
toes and dorsiflexes her big toe. What is the name of this reflex, and is the childs response
normal?
A) Moro reflex; abnormal

om
B) Rooting reflex; normal
C) Grasp reflex; abnormal

.c
D) Babinski reflex; normal

ep
4. A nurse in the ICU observes paradoxical irritability, meningeal irritability, and nuchal
rigidity in a 5-year-old girl. She suspects meningitis. What other finding or findings

pr
would support this belief? Select all that apply.
A) Positive Babinski reflex st
te
B) Positive Moro reflex
ng

C) Positive Brudzinskis sign


D) Positive Kernigs sign
si

5. A newborn baby involved in an auto accident has been admitted to the ICU and is
ur

currently being assessed by a nurse in a drafty, cool room. The nurse notes mottling on
the infants skin, a progression of coolness toward the trunk, peripheral cyanosis, normal
yn

blood pressure, low pulse pressure, and tachycardia. Given the circumstances, which of
the following are reliable signs that the infant is in shock? Select all that apply.
.m

A) Mottled skin
w

B) Progression of coolness toward the trunk


w

C) Peripheral cyanosis
w

D) Normal blood pressure


E) Low pulse pressure
F) Tachycardia
6. A nurse working with an infant patient observes that the child, who has a nasogastric
tube, is grunting on expiration, wheezing, and seesaw breathing. The nurse recognizes
that the infant is most likely experiencing which condition?
A) Respiratory distress due to obstructed nasal passages
B) Respiratory arrest due to obstruction of the glottis
C) Epileptic seizure due to overstimulation
D) Shock due to loss of blood
7. A 2-year-old is admitted to the ICU due to severe dehydration. The mother of the child is
distraught and feels guilty that she took a long walk with her child in the sun earlier in the
day. She said that she and the rest of the family had no issues with dehydration. What
would be the best response for the nurse to give the mother?
A) Children his age should have limited sun exposure and should be covered with

om
sunscreen.
B) The mother should regularly test the specific gravity of the childs urine to determine
whether he is adequately hydrated.

.c
C) Children are at increased risk for dehydration due to a higher percentage of total-

ep
body water.

pr
D) The mother should look for taut skin and edema as signs of dehydration in her child.
8. A 6-year-old girl who is in the ICU for an unrelated reason confides in the nurse that her
st
father sometimes hits her. Which of the following findings would most likely indicate
te
abuse in this situation?
ng

A) A bruise on her knee


B) A bruise on the bottom of her foot
si

C) A bruise on her abdomen


ur

D) A bruise on her face


yn

9. A 12-year-old boy is experiencing severe pain following emergency surgery in the ICU.
He is visibly frustrated and restless. His parents would like to achieve a steady state of
.m

pain relief for their son and to give him a sense of some control over the pain. Which pain
relief method would be most appropriate in this situation?
w

A) Distraction by means of a video game


w

B) Massage
w

C) Opioids
D) PCA
10. The nurse observes that a child in the ICU who has been admitted following an auto
accident is visibly stressed and crying. The child has a broken arm, which has been
splinted, and is receiving analgesics. No other injuries have been found. When the nurse
performs a pain assessment test with the child, the child refuses to cooperate. The patients
mother is hysterical and is frantically pacing next to the bed. What is the most likely
cause of the childs distress?
A) Fear of further medical interventions
B) The mothers communicated anxiety
C) Pain from the broken arm
D) Shock
11. The nurse is caring for a critically ill child. The child has been experiencing tachypnea for
several hours despite appropriate interventions. Suddenly, the child becomes bradypneic.
What is the most appropriate nursing action?
A) Continue the current plan of care, as this change indicates improvement.

om
B) Initiate positive-pressure ventilation with a manual bag-mask device.
C) Increase intravenous sedation and narcotic dosages to continue improvement.

.c
D) Ask the family to leave the room in anticipation of further action.

ep
12. A child is admitted to the ICU after being in a motor vehicle crash. The nurse assesses a

pr
sudden decrease in the pulse rate. The nurse knows that this change in pulse rate is most
likely related to what pathology?
A) Anxiety
st
te
B) Hyperventilation
ng

C) Shock
si

D) Hypoxemia
ur

13. A critically ill child who is experiencing internal bleeding from a motor vehicle accident
demonstrates a sudden significant drop in blood pressure. What intervention should the
yn

nurse implement first?


A) Administer 20 mL/kg intravenous crystalloid fluid.
.m

B) Assess for oversedation with benzodiazepines or narcotics.


w

C) Increase frequency of vital sign assessment to every 15 minutes.


w

D) Reassess mental status and level of consciousness.


w

14. A 6-month-old infant, who was admitted for gastroenteritis, has depressed fontanelle at
rest and while in a supine position. What additional assessments should the nurse perform
or facilitate?
A) Denver Developmental Test
B) Deep tendon reflexes
C) Number of wet diapers per day
D) Measure the frontal occipital circumference
15. An infant with a history of diarrhea and poor feeding for 2 days is admitted to a CCU.
The infant has progression of coolness toward the trunk, prolonged capillary refill, and
moderate tachycardia and is somewhat difficult to arouse. What additional assessment
data should the nurse collect first?
A) Cardiac monitor pattern
B) Serial blood pressure
C) Deep tendon reflexes
D) Status of anterior fontanelle

om
16. The nurse notices that a 5-month-old patient is having difficulty breathing, with nasal
flaring, audible grunting, and sternal and intercostal retractions. The infants respiratory
rate is rapid, and the infant is restless and obtunded. What intervention should the nurse

.c
implement first?

ep
A) Position the baby with head in a neutral midline position and apply the jaw-thrust
maneuver.

pr
B) Facilitate administration of an inhaled bronchodilator.
C) st
Administer intravenous sedation or pain medication.
te
D) Ask the parent to use comforting measures for the infant.
ng

17. A young child has repeated upper respiratory infections. The mother is distressed and
states that she is sure she is taking care of her child properly and does not understand why
si

her child is constantly ill. Nursing assessment validates that the mother is providing a
balanced diet, the child eats well, the mother is careful with hand hygiene, and the child
ur

has no genetic or chronic diseases. The child does attend day care while the mother
yn

works. On what rationale should the nurse base her reassurance to the mother?
A) All small children are sick all the time, and they outgrow this behavior in a few
.m

years.
B) The day care center is probably the cause of the constant illness, and the child
w

should be moved.
w

C) Small children have an immature immune system and are more susceptible to viral
w

illness.
D) The mother should be referred for counseling as she is exhibiting a pathological lack
of coping.
18. The nurse enters the room of a critically ill child who is newly unresponsive. What is the
next nursing action?
A) Open the airway.
B) Place on side.
C) Give abdominal thrusts.
D) Start rescue breathing.
19. A child experiences a respiratory arrest and is intubated. What is the best nursing
assessment parameter to immediately verify proper placement of the endotracheal tube?
A) Symmetrical rise and fall of the chest
B) Chest x-ray shows right bronchial intubation.
C) Tidal volume is within limits for age.

om
D) Absence of sternal retractions and grunting
20. The nurse is preparing to administer a medication to a child. The medication is ordered as
10 mg/kg. When the nurse looks up the childs weight on the vital sign record, no unit of

.c
measure is documented. What is the best nursing action?

ep
A) Assume that the weight was measured in kilograms, since that is unit policy.

pr
B) Call the staff member who weighed the child and verify the unit of measure.
C)
st
Use arithmetic analysis to determine the probable unit of measure used.
te
D) Reweigh the child to ensure accuracy of weight determination for the day.
21. Since most medications in pediatrics are calculated based on the childs weight,
ng

considerable nursing time is required to verify correct dosages. This time constraint can
be a problem in emergent situations. What mechanism can be used to facilitate accuracy
si

of dosing during emergencies?


ur

A) Precalculate doses of emergency drugs for each child admitted.


yn

B) Estimate childs weight and drug doses in emergencies to save time.


C) Use the standard doses from the Pediatric Advanced Life Support recommendations.
.m

D) Add a pharmacist to the emergency response team for pediatrics.


w

22. The nurse is caring for a 1-year-old child who has had major surgery and suspects that the
w

child is in pain. What method should the nurse use to assess the child for pain?
w

A) A one-dimensional system such as the Faces scale


B) A multidimensional system such as COMFORT
C) Ask the parents to decide if the child is in pain.
D) Assess vital signs, restlessness, crying, and diaphoresis.
23. A young child is experiencing pain secondary to surgical repair of a fractured femur. The
child is on supplemental oxygen and is breathing spontaneously and crying. The nurse
wishes to administer the most effective pain relief with the least risk of adverse effects.
Which combination of relief measures is most likely to meet this goal?
A) Antipsychotic medication and sedative
B) Opioid and distraction strategies
C) Sedative and opioid
D) Muscle relaxant and massage
24. A pediatric critical care unit wishes to ensure family-centered care. What nursing actions
will best support family-centered care?

om
A) Asking parents to leave during painful procedures for the child
B) Assuming that the childs illness is secondary to parental ignorance

.c
C) Encouraging a family member to be constantly present at the childs side

ep
D) Restricting visits by family to ensure that the child can rest

pr
Answer Key st
te
1. D
ng

2. A
si

3. D
ur

4. C, D
5. B, E
yn

6. A
.m

7. C
w

8. C
w

9. D
w

10. B
11. B
12. D
13. A
14. C
15. D
16. A
17. C
18. A
19. A
20. D
21. A
22. B

om
23. B
24. C

.c
ep
Chapter 11 The Critically Ill Pregnant Woman

pr
1. A pregnant woman in her third trimester is in the ICU for minor trauma to her right arm
and leg as a result of a car wreck. Her injuries appear to be superficial, but her nose
st
begins bleeding profusely immediately after the nurse places a nasotracheal tube. What is
the most likely explanation for the bleeding?
te
A) Internal hemorrhaging from trauma
ng

B) Hemophilia
si

C) Rupture of the mucosa, which are extra fragile during pregnancy


ur

D) Improper placement of tube


yn

2. An obese woman was admitted to the ICU for diabetic coma caused by advanced diabetic
ketoacidosis. On recovering, the patient mentions to the nurse that she missed her last
.m

period and has felt nauseated in recent weeks. In reviewing her chart, the nurse compares
her current test results with those from her last admission to the hospital, about 3 months
ago. She notes that the womans red blood cell count has increased by 20%, her systolic
w

blood pressure has increased by 10 mm Hg, her resting heart rate has increased from 85 to
w

105 bpm, and her creatinine clearance has increased from 90 to 130 mL/min. Which of
these changes is consistent with those that normally accompany pregnancy? Select all that
w

apply.
A) 20% increase in red blood cells
B) 10 mm Hg increase in systolic blood pressure
C) Increase in resting heart rate from 85 to 105 bpm
D) Increase in creatinine clearance from 90 to 130 mL/min
3. A woman in her ninth month of pregnancy is in the ICU, recovering from surgery to
remove a stage 1 tumor from her brain. She is still under the effects of general anesthesia.
As the nurse is taking her vital signs, the woman regurgitates some fluid from her mouth.
What is the most likely cause of the regurgitation?
A) Esophageal cancer
B) Displacement of the esophageal sphincter
C) Airway obstruction
D) Congenital defect of the esophagus
4. Blood work results of a pregnant woman in her third trimester in the ICU have recently
been delivered, and the nurse is reviewing them. The woman has a hematocrit of 35%, a

om
white blood cell count of 4,000/mm3, and a fibrinogen level of 500 mg/dL. She also has a
blood pressure reading of 118/70 mm Hg. Which of these is in the normal range for a
pregnant woman? Select all that apply.

.c
A) Hematocrit of 35%

ep
B) White blood cell count of 4,000/mm3

pr
C) Fibrinogen level of 500 mg/dL
D) Blood pressure of 118/70 mm Hg st
te
5. A pregnant woman has arrived at the ICU with a blood pressure reading of 170/115,
pronounced edema in her hands and feet, and proteinuria. Which condition does the
ng

woman most likely have?


si

A) Severe preeclampsia
ur

B) Preeclampsia
C) Disseminated intravascular coagulation
yn

D) HELLP syndrome
.m

6. A patient who experienced severe preeclampsia during her pregnancy has just delivered
her baby in the ICU. She has been receiving magnesium sulfate therapy to prevent
w

seizures. Which of the following would be the correct nursing intervention in this
w

situation?
w

A) Discontinue the magnesium sulfate therapy immediately.


B) Continue the magnesium sulfate therapy for 2 hours after delivery.
C) Continue the magnesium sulfate therapy for 24 hours after delivery.
D) Explain to the patient that she will have to remain on magnesium sulfate therapy for
the rest of her life.
7. A patient with severe preeclampsia in the ICU has recently had a seizure. She is currently
receiving magnesium sulfate therapy. What intervention can the nurse make to reduce the
risk for future seizures in this patient?
A) Turn off the overhead lights in the room.
B) Stop the magnesium sulfate therapy.
C) Have the physician prescribe hydralazine to her.
D) Place a wedge under the womans right hip.
8. A woman in her seventh month of pregnancy presents to the ICU with hemolysis,
elevated liver enzymes, and a low platelet count. Which condition does this woman most
likely have, and what is a proper intervention?
A) Severe preeclampsia; magnesium sulfate therapy

om
B) Disseminated intravascular coagulation; broad-spectrum antibiotics
C) Amniotic fluid embolism; intubation and ventilation with 100% oxygen

.c
D) HELLP syndrome; antihypertensive agents

ep
9. The nurse is working with a patient with severe preeclampsia and acute respiratory

pr
distress syndrome (ARDS). The patient is receiving mechanical ventilation using positive
end-expiratory pressure. Which nursing intervention would be most appropriate in this
situation? st
te
A) Position the patient on her side.
ng

B) Turn off the overhead light.


C) Evaluate the patient for neurological symptoms of seizure.
si

D) Perform CPR.
ur

10. A pregnant woman has been admitted to the ICU with disseminated intravascular
yn

coagulation (DIC). She exhibits tachycardia, tachypnea, temperature instability, increased


cardiac output, and decreased peripheral resistance. What is the most likely underlying
.m

cause of DIC in this situation and what is the best intervention?


A) Preeclampsia; antihypertensive agents
w

B) Sepsis; broad-spectrum antibiotics


w

C) Amniotic fluid embolism; intubation and ventilation with 100% oxygen


w

D) Abruptio placentae; prompt delivery of the fetus


11. A pregnant woman who is due to give birth in 2 weeks is shot in the shoulder during a
robbery. Estimated blood loss at the scene was 500 mL, and she lost another 500 mL
during surgical repair. The patient is admitted to the CCU and is in a supine position.
Which initial assessment findings would the nurse anticipate?
A) Vital signs consistent with compensated shock
B) Slightly high pulse and blood pressure, alert and oriented, skin warm
C) Severe and constant lower abdominal pain
D) Severe tachycardia and hypotension, obtunded, skin cool and pale
12. A pregnant woman was admitted to the CCU for monitoring after an elective surgical
procedure. Arterial blood gases are as follows: pH 7.48, PaCO2 27, HCO3 ion 21, PaO2
108, SaO2100%. Respiratory rate is slightly elevated. The patient is receiving oxygen by
mask at 40% and maintenance intravenous fluids. What is the most important nursing
action?
A) Reduce inhaled oxygen because the PaO2 and SaO2 are too high.

om
B) Medicate for pain because the acidbase balance indicates respiratory alkalosis.
C) Consider intubation and mechanical ventilation to support gas exchange.

.c
D) Continue the current plan of care, as results are within normal limits.

ep
13. A pregnant woman is admitted to a CCU after a motor vehicle crash. She is
endotracheally intubated and mechanically ventilated. Respiratory assessment reveals

pr
diminished lung sounds at posterior bases, decreased functional residual capacity, and
increased spontaneous tidal volume. What is the highest-priority nursing action?
A) st
Obtain orders for ventilator setting adjustments to compensate for the abnormal
te
findings.
ng

B) Suction the patient via the endotracheal tube to stimulate cough.


C) Continue the present plan of care, as the findings are expected.
si

D) Obtain orders for an inhaled bronchodilator for bronchospasm.


ur

14. A pregnant woman is admitted to the CCU after a traumatic incident. The nurse is
yn

concerned that the patient may be having significant and unidentified blood loss. Which
assessment findings could indicate significant blood loss?
.m

A) Slightly decreased hemoglobin and hematocrit


w

B) Urine output less than 40 mL/hr


w

C) Elevated creatinine and urea clearance rates


w

D) Slightly low serum creatinine and blood urea nitrogen


15. A woman who is 8 months pregnant is being assessed on a routine clinic visit. Which
assessment findings require further assessment and treatment?
A) Blood pressure increased by 10 points from previous visit
B) Blood pressure 150/95 mm Hg
C) 1+ edema of lower extremities
D) Urine output 750 mL in 24 hours
16. A pregnant woman at 30 weeks gestation is in the CCU with severe eclampsia. She is
experiencing pulmonary edema, severe oliguria, and hypotension and continues to have
seizures. She is endotracheally intubated, supported by a mechanical ventilator, and is
receiving intravenous magnesium sulfate at 3 g/hr; the serum magnesium level is 8
mg/dL. She is beginning spontaneous early labor. What is the most important
intervention?
A) Administer medication to delay the onset of labor, as the baby is preterm.
B) Increase the dose of magnesium sulfate to control seizures.
C) Facilitate delivery of the baby, as eclampsia is worsening.

om
D) Administer high-dose diuretics for the pulmonary edema.
17. The nurse is caring for a pregnant woman with severe preeclampsia. Based on the

.c
pathophysiology of severe preeclampsia, the nurse expects what changes in assessment

ep
data?
A) Pulmonary edema from capillary leakage or diminished left ventricular function

pr
B) Increased urine output from glomerular capillary damage
C) st
Decreased serum creatinine from increased urine output
te
D) Decreased serum aspartate aminotransferase (AST) from liver damage
ng

18. A pregnant woman with severe preeclampsia is receiving intravenous magnesium sulfate
at 3 g/hr. Her serum magnesium level is 7 mg/dL. What is the most important nursing
si

assessment?
ur

A) Decreased incidence and frequency of seizures


yn

B) Decreased respiratory rate and hyporeflexia


C) Increased urine output greater than 40 mL/hr
.m

D) Flushing of skin and diaphoresis


w

19. A pregnant woman with severe preeclampsia is being managed with intravenous
w

magnesium sulfate. Which nursing assessment would indicate that the dose of magnesium
sulfate would need to be increased?
w

A) Increased frequency of seizures


B) Hyporeflexia and diaphoresis
C) Serum magnesium level of 16 mg/dL
D) Increased drowsiness and flushed skin
20. A woman is admitted to CCU after delivery of a retained demised fetus from abruptio
placentae. The patient develops disseminated intravascular coagulation (DIC). What
findings are present in DIC?
A) Diminished platelets and elevated partial thromboplastin time
B) Elevated white blood cell count and elevated immature neutrophils
C) Diminished clotting time and prothrombin time
D) Elevated fibrinogen and diminished fibrinogen split products
21. The nurse is monitoring a pregnant woman who is at high risk for the development of
disseminated intravascular coagulation (DIC). What physical assessment findings would
indicate DIC?
A) Urine clear without sediment at 40 mL/hr

om
B) Oozing blood from all intravenous sites
C) All vital signs within expected parameters for pregnancy

.c
D) Absence of elevated protein in urine or serum

ep
22. During a patients cesarean section delivery of twin girls, the patient had a sudden onset of

pr
dyspnea and cyanosis with a drop in blood pressure, and then developed cardiac arrest.
Based on these clinical events, the nurse suspects what complication?
A) Disseminated intravascular coagulation
st
te
B) Amniotic fluid embolism
ng

C) Uterine prolapse
si

D) Massive myocardial infarction


ur

23. A pregnant woman who is close to term is admitted to the emergency room after being
involved in a motor vehicle crash. Within moments, she experiences a cardiopulmonary
yn

arrest. What is a nursing priority in the care of this woman?


A) Initiate cardiopulmonary resuscitation as for any patient.
.m

B) Monitor fetal heart tones continuously.


w

C) During care, place a 2-inch wedge under the patients right hip.
w

D) Administer oxygen by nasal cannula at 2 L/min.


w

24. A pregnant woman is admitted to the emergency room after a motor vehicle crash. She is
unresponsive and covered with blood. A cervical spine collar and back board are in place.
What should the nurse assess first?
A) Airway, breathing, circulation, and level of consciousness
B) Vital signs and urine output and protein
C) Fetal heart sounds, movement, and position
D) External and internal bleeding symptoms
25. A pregnant woman at term who is being cared for in the CCU after a traumatic injury
goes into labor. She is critically ill and somewhat unstable. What is the best nursing
action to support this patient?
A) Assign a nurse to be her labor coach and stay with the patient.
B) Contact the patients planned labor coach to be with the patient.
C) Sedate the patient so that the labor will not interfere with other care.
D) Immediately prepare for an emergency cesarean section.

om
Answer Key

.c
1. C

ep
2. A, C, D
3. B

pr
4. A, C, D
5. A st
te
6. C
ng

7. A
si

8. D
ur

9. A
10. B
yn

11. B
.m

12. D
w

13. C
w

14. B
w

15. B
16. C
17. A
18. B
19. A
20. A
21. B
22. B
23. C
24. A
25. B

Chapter 12 The Critically Ill Older Patient

om
1. The nurse is working with a patient in the ICU who has presbycusis. Which of the
following would the nurse likely find with this client? Select all that apply.

.c
A) Impairment of discrimination ability

ep
B) Dementia
C) A gray shadow in the center of the visual area

pr
D) Balance problems during transfer
st
2. An elderly patient is recovering in the ICU after surgery. When the nurse notices that the
te
patient has not touched his food, she asks him why he is not eating. He complains that the
food is too bland and that his mouth is dry. Which of the following interventions should
ng

the nurse make to help the patient enjoy his meal? Select all that apply.
A) Use liberal seasonings in his food.
si

B) Vary his position in the bed.


ur

C) Measure his creatinine clearance.


yn

D) Offer him sugar-free candies before his meal.


.m

3. A nurse is bathing an elderly patient in the ICU. The patient has fine wrinkling,
looseness, and sagging in her skin. Which of the following should the nurse do to best
w

care for the patients skin? Select all that apply.


w

A) Scrub the patients skin vigorously with a loofah to exfoliate.


w

B) Lather the patient with plenty of soap.


C) Immerse the patient for 10 minutes daily.
D) Use an emollient after bathing.
4. An elderly patient in the ICU has been on bed rest for the past 48 hours, has had
decreased intake of food and fluids, and is taking multiple medications. Which of the
following conditions is the patient especially prone to, given her situation? Select all that
apply.
A) Constipation
B) Dehydration
C) Muscle atrophy
D) Decreased glomerular filtration rate
E) Malignant lesions
F) Presbycusis
5. An elderly patient in the ICU broke her foot when she attempted to stand on it after

om
having been on bed rest for 2 weeks. The nurse, who was assisting the patient as she
stood, was surprised at how easily the woman sustained the injury. Which of the
following factors most likely predisposed this patient to the injury she sustained?

.c
A) Lack of balance

ep
B) Mental confusion

pr
C) Decreased bone mineral density
D) Pernicious anemia
st
te
6. An elderly patient in the ICU presents with atrial tachycardia. The nurse also observes
that the patient appears anorexic. Which of the following conditions would best explain
ng

these symptoms?
A) Diabetes mellitus
si

B) Arthritis
ur

C) Pernicious anemia
yn

D) Hyperthyroidism
.m

7. An older adult patient in the ICU has diabetes. Which intervention could the nurse
perform that would best prevent complications related to this disease?
w

A) Have the patient rise from his bed and bear weight.
w

B) Offer the patient sugar-free candy.


w

C) Bathe the patients feet daily and apply emollient.


D) Reposition the patient often in bed.
8. An elderly patient in the ICU has hyperthyroidism. As the nurse is talking with her, she
suddenly becomes confused and starts babbling incoherently. The nurse takes her vital
signs and finds tachycardia, a temperature of 98 F, and an elevated blood pressure
reading. Which of the following is the most reliable indication that the patient has an
infection?
A) Delirium
B) Temperature
C) Blood pressure
D) Tachycardia
9. An older patient in the ICU has been determined to have early dementia. As the nurse is
taking her vital signs, the patient expresses frustration that she cannot remember what
happened earlier in the day. What can the nurse do to help calm the patient?
A) Perform the Mini-Cog on the patient.

om
B) Remind the patient of what actually happened earlier in the day.
C) Ask the patient to recount some memories from her childhood.

.c
D) Offer the patient sugar-free candy.

ep
10. An elderly patient in the ICU is taking erythromycin to treat a bacterial infection. The

pr
patient is diabetic and has been on bed rest for 3 days. She has hypoglycemia and nausea.
After being on erythromycin for 24 hours, the patient is still showing signs of infection,
st
including fever and delirium. What is the most likely reason for the medication not taking
effect?
te
A) Lack of movement due to bed rest
ng

B) Delayed stomach emptying time


si

C) Drug interaction between erythromycin and insulin


ur

D) Drug resistance
yn

11. An older patient is admitted to the CCU after a motor vehicle crash. What normal
physiological change of aging will most affect the patients ability to adjust to the CCU
.m

milieu?
A) Onset of dementia
w

B) Impaired hearing
w

C) Altered stress response


w

D) Increased lung compliance


12. The nurse is caring for an elderly patient in the CCU who has very wrinkled skin, very
uneven skin tone, and several irregularly shaped white blotches on the skin. What is
probably the cause of these skin changes?
A) Adverse reaction or allergy to a medication
B) Secondary to stress of CCU admission
C) Overexposure to sunlight
D) Individual variation in skin tone
13. An elderly patient is admitted to the CCU with a critical illness. To support the
developmental goals and tasks of this patient, what is an appropriate intervention?
A) Restrict visiting to very immediate family members to prevent fatigue.
B) Contact the social worker to help place pets in the animal shelter.
C) Call the patient by a nickname such as Grandma or Grandpa.

om
D) Ask the patient about his life accomplishments and feelings of satisfaction.
14. An elderly patient with presbycusis is being cared for in the CCU after major surgery.
Which alteration in nursing approach will best facilitate this patients care?

.c
A) Keep the light on in the room at all times.

ep
B) Increase the dose of sedation at night.

pr
C) Introduce yourself each time you enter the room.
D) st
Speak clearly and face the patient when talking.
te
15. An elderly patient with presbyopia is being discharged from the CCU after an invasive
cardiac procedure. The nurse is giving discharge instructions to him that include
ng

instructions on medications. What accommodation should the nurse plan to use while
teaching this patient?
si

A) Speak loudly, clearly, and directly to the patient.


ur

B) Give all instructions to the family only.


yn

C) Teach the patient to get out of bed slowly.


.m

D) Use a larger font for all printed materials.


16. An elderly patient is being cared for in the CCU after a surgical procedure. The nurse
w

finds snoring, frequent awakening, nocturnal sweating, and awakening with anxiety. The
w

family states that the patient had these behaviors at home before the surgery. What is the
most appropriate nursing action?
w

A) Continue routine assessment, as these behaviors indicate normal aging changes.


B) Make a referral for sleep disorder evaluation, as these behaviors are abnormal.
C) Tell the family to discuss these behaviors with the physician after discharge from the
CCU.
D) Increase bedtime sedation to ensure that the patient gets adequate rest.
17. The nurse is caring for an elderly patient in the CCU. Which intervention indicates
understanding of the normal aging changes in the skin for this patient?
A) Use very hot water and strong soap for bathing.
B) Increase the frequency of turning and repositioning.
C) Bathe every other day and avoid use of lotions.
D) Ask the patients preference for time of bath.
18. An elderly patient in the CCU is getting out of bed for the first time after being on bed
rest for several days. What is the highest-priority nursing intervention?

om
A) Provide adequate physical help so that the patient does minimal weight bearing.
B) Ensure that the patients call light is always within reach.

.c
C) Apply a vest restraint while the patient is up to prevent falling.

ep
D) Have the patient sit on the side of the bed for a few minutes before standing.

pr
19. The nurse is caring for an elderly patient with pneumonia. What normal aging change
st
increases the patients risk for the development of pneumonia?
te
A) Decreased compliance
B) Reduced functional epithelial cells
ng

C) Increased functional reserve capacity


si

D) Damage from smoking


ur

20. The nurse is caring for an elderly patient who is receiving a number of medications.
yn

Taking into consideration normal aging, what should the nurse monitor most carefully?
A) Vital signs
.m

B) Creatinine clearance
w

C) Muscle strength
w

D) Cardiac function
w

21. The nurse is caring for a critically ill elderly patient. The patient is confused and agitated.
The family states that the patient had not exhibited confusion at home. What is the most
important nursing action?
A) Apply restraints to prevent patient injury.
B) Sedate the patient to prevent injury.
C) Assess vital signs and laboratory results for abnormal values.
D) Apply oxygen by nonrebreather mask at 10 L/min.
22. An elderly patient is exhibiting inability to remember locations and routines, does not
recognize close family members, and expresses constant anger, with frequent physical
attacks on others. The family states that the patient has had worsening behavior for
several months. The patient was clean and well-dressed on admission. There are no
bruises or lesions on the arms or head. What is probably causing this behavior?
A) Dementia
B) Delirium
C) Elder abuse

om
D) Personality disorder
23. The nurse is caring for an elderly patient. As visiting hours approach, the patient becomes
increasingly anxious and states she is worried that her son will be angry with her for

.c
being sick and in the CCU. What is the most appropriate nursing intervention?

ep
A) Assess the patient for possible abuse.

pr
B) Assure the patient that her son loves her.
C) Tell the son that his mother thinks he is angry with her.
D)
st
Give the son extra time to visit his mother.
te
24. The nurse notices that a physicians order for a medication for an elderly patient seems
ng

very low. What is the best nursing action?


si

A) Verify that the low dose is deliberate to compensate for aging.


ur

B) Ask the pharmacist for a rationale for the dose.


C) Refuse to administer the medication as the dose is too low.
yn

D) Ask the physician to increase the dose of the medication.


.m

Answer Key
w
w

1. A, D
w

2. A, D
3. C, D
4. A, B, C
5. C
6. D
7. C
8. A
9. C
10. B
11. C
12. C
13. D
14. D

om
15. D
16. B

.c
17. B

ep
18. D

pr
19. B
20. B st
te
21. C
ng

22. B
23. A
si

24. A
ur
yn

Chapter 13 The Postanesthesia Patient

1. A patient is receiving moderate IV sedation during a procedure. Which of the following


.m

statements accurately describes the expected condition of the patient in this situation?
Select all that apply.
w

A) The patient can respond verbally to questions.


w

B) The patient can maintain a patent airway.


w

C) The patient has increased anxiety.


D) The patients vital signs fluctuate dramatically.
E) The patient will remember all details of the experience.
F) The patient will require longer than usual for postoperative recovery.
2. An elderly patient in the ICU is receiving an anesthetic that is expected to allow him to
maintain his own patent airway and to respond to verbal commands during a procedure.
However, the patient has a history of complications related to anesthesia, and thus the
anesthesiologist will be observing the patient during the procedure in case the patient
develops apnea or an obstructed airway or the level of anesthesia needs to be changed to
meet the patients needs. This type of anesthesia is known as which of the following?
A) Moderate sedation
B) Monitored anesthesia care
C) Deep sedation
D) General anesthesia
3. A nurse is entering the room of a patient who has just come from surgery and is

om
recovering from general anesthesia. What method should the nurse use to determine
changes in cognitive function of the patient?

.c
A) Interview with the surgeon

ep
B) Patients response to the stir-up regimen
C) Interview with the patients family

pr
D) Mini-Cog test
st
4. A patient is transported to the PACU following a surgical procedure in which he received
te
nitrous oxide for general anesthesia. What should the patient be receiving to prevent
diffusion hypoxia?
ng

A) Moderate IV sedation
si

B) Reversal agent
ur

C) Blood products
yn

D) Supplemental oxygen
5. A patient with chronic obstructive pulmonary disease arrives in the ICU following
.m

surgery with general anesthesia. This patient will likely require ventilator support for
which purpose?
w

A) To prevent hypoventilation
w

B) To reverse hypotension
w

C) To suppress the patients urge to vomit


D) To ease postoperative pain
6. A patient arrives in the ICU following open-heart surgery. The patient is recovering from
general anesthesia and is receiving supplemental oxygen. The nurse learns from the
anesthesia providers report that the patient lost much blood during the surgery. The nurse
knows as well that the patient had fasted before the surgery, per standard procedures.
Which complication is this patient most likely to experience following surgery?
A) Hypothermia
B) Hypoventilation
C) Hypotension
D) Hypoxemia
7. A neonate arrives at the ICU following surgery for repair of a cleft palate. The nurse
learns from the anesthesiologists report that the patient received general anesthesia and
that little blood was lost during the surgery. What is the most likely postsurgical
complication for this patient, and what intervention should the nurse immediately

om
implement?
A) Hypothermia; heated blankets and warm IV fluids

.c
B) Hypotension; intravenous fluids

ep
C) Hypoxemia; supplemental oxygen
D) Hypoventilation; stir-up regimen

pr
8. A patient arrives in the ICU following surgery to her eyes. She received opioids during
st
the surgery. What complication is this patient most likely to experience, and what is the
correct intervention for the nurse to perform?
te
A) Postoperative nausea and vomiting; perform the stir-up regimen
ng

B) Hypothermia; cover the patient with warm blankets


si

C) Hypothermia; give the patient a cold IV solution


ur

D) Postoperative nausea and vomiting; position the patient on her side, with head and
neck extended
yn

9. A patient in the ICU received opioid analgesics during surgery and has just been given
.m

naloxone, an opioid antagonist, following surgery. What complication is this patient most
likely to experience, and what is the correct nursing intervention?
w

A) Severe pain; administer an analgesic drug immediately


w

B) Severe pain; administer an analgesic drug after 30 minutes


w

C) Severe hypotension; administer IV fluids immediately


D) Severe hypotension; raise the head of the patients bed to 60 degrees
10. A patient is brought to the ICU following surgery. This patient has a history of peptic
ulcers. As the patient wakes up from general anesthesia, he complains of severe pain and
asks for pain medication as soon as possible. What would be the best analgesic drug for
this patient in this situation?
A) Toradol
B) Patient-controlled analgesia
C) Intravenous opioid drugs
D) Epidural opioid drugs
11. A patient returns to the ICU following surgery. The nurse notes that the patient received
the anesthetic agent pipecuronium bromide before surgery. What dysrhythmia should the
nurse expect to find in this patient?
A) Bradycardia
B) Tachycardia

om
C) Atrial fibrillation
D) AV dissociation

.c
12. A patient who is scheduled for a surgical procedure with regional anesthesia asks the

ep
nurse to explain the anesthesia. What nursing statement is most accurate?

pr
A) It involves a deep coma assisted by inhaled medications.
B)
st
It is an induced state of calm or sleep at varying levels.
te
C) It produces analgesia and paresthesia in a specific body part.
D) The anesthetic is injected into the lumbar intrathecal space.
ng

13. A patient who will undergo correction of a dislocated joint in the emergency department
si

asks the nurse what type of anesthesia will be used. What is the most appropriate nursing
response?
ur

A) General anesthesia
yn

B) Deep sedation
.m

C) Regional anesthesia
D) Peripheral nerve block
w
w

14. A patient is to undergo a surgical procedure and needs to be unconscious for the
procedure. What anesthetic agent is most appropriate?
w

A) Inhaled halothane
B) Subcutaneous lidocaine
C) Intravenous midazolam
D) Intravenous succinylcholine
15. A patient needs to have a small laceration sutured. What is the most appropriate analgesia
for this procedure?
A) Inhaled nitrous oxide and oxygen
B) Intravenous atropine
C) Subcutaneous lidocaine
D) Intravenous propofol
16. A patient has just had a procedure with moderate sedation as the anesthesia used. During
the immediate recovery period, what is the most important nursing action?
A) Begin range-of-motion exercises.

om
B) Initiate incentive spirometry.
C) Monitor vital signs.

.c
D) Insert nasogastric tube.

ep
17. The nurse is caring for a patient recovering from general anesthesia for repair of a cardiac
valve. When the patient arrives from the operating room, what nursing assessment is most

pr
important?
A) Cardiac monitor pattern
st
te
B) Wound site assessment
C) Airway security
ng

D) Patient position
si

18. Immediately after general anesthesia, one of the nursing interventions is to elevate the
ur

head of the bed. What is the best nursing rationale for this procedure?
yn

A) Provides patient comfort


B) Reassures the family
.m

C) Hastens anesthesia recovery


w

D) Contributes to gas exchange


w

19. A patient is recovering from spinal anesthesia. During the immediate recovery period, the
w

nurse notices an increase in pulse and respiration rates and disorientation and increasing
agitation. What nursing assessment should be performed first?
A) Sensation of feet and legs
B) Whether a stir-up procedure has been performed
C) History of adverse reactions to medications
D) Current pulse oximetry value
20. The patient is recovering from general anesthesia and has a pulse oximeter reading of
82%. The nurse implements the stir-up regimen. What does this regimen include?
A) Assess vital signs every 15 minutes.
B) Administer a narcotic antagonist.
C) Elevate the patients legs 45 degrees.
D) Encourage the patient to turn, cough, and deep breathe.
21. A patient in the postanesthesia recovery period is to be extubated. Why does the nurse
suction the oral pharynx before extubation?

om
A) Reduces the risk of laryngospasm
B) Increases mucosal moisture

.c
C) Increases patient comfort

ep
D) Decreases nausea and vomiting

pr
22. A patient with a history of muscular dystrophy is in the immediate postanesthesia period
st
and begins to experience muscle rigidity, hypercarbia, tachycardia, sweating, and labile
blood pressure. What vital sign should the nurse assess first?
te
A) Temperature
ng

B) Apical pulse
si

C) Respiratory rate
ur

D) Pulse pressure
yn

23. A patient in the immediate postoperative period is experiencing significant hypotension.


What are the most likely causes? Select all that apply.
.m

A) Blood or fluid loss


B) Reaction to opioids
w
w

C) Intraoperative myocardial infarction


w

D) Acute renal failure


E) Severe hypothermia
F) Early hypoxemia

Answer Key

1. A, B
2. B
3. B
4. D
5. A
6. C
7. A
8. D

om
9. B
10. C

.c
11. C

ep
12. C

pr
13. B
14. A st
te
15. C
ng

16. C
17. C
si

18. D
ur

19. D
yn

20. D
.m

21. A
22. A
w

23. A, B, C, E
w
w

Chapter 14 Rapid Response Teams and Transport of the Critically Ill Patient

1. A nurse working in the ICU of a rural hospital is trying to make the case to the
administration to create an RRT for the hospital. There have been a number of
preventable deaths at the hospital recently. Which case provides the best support for the
nurses argument that the hospital needs such a team?
A) A patient died of cardiac arrest because medical staff did not respond quickly
enough to his condition.
B) A patient died of an overdose of medication due to a medication error.
C) A patient died of an infection after a surgeon failed to scrub in properly before
surgery.
D) A patient died from head trauma after slipping on a wet surface in her room in the
ICU.
2. An RRT has just been created at a hospital. An RN from the ICU has been appointed as
the leader and is planning a training session for other members of the RRT. Which topics
would be appropriate to include in the training? Select all that apply.
A) The benefits of early rescue

om
B) Triage skills and advanced cardiac life support
C) Equipment needed for air transport of a patient between facilities

.c
D) Expectations of RRT members when responding to a call

ep
E) EMTALA guidelines for requirements for evaluating a patients suitability for
transfer

pr
F) Guided imagery to assist patients dealing with severe pain
st
3. A patient must be transferred from one hospital to another due to lack resources at the
sending hospital needed by the patient. Which of the following conditions would incline
te
the transport team of the hospital to choose ground transport as opposed to air? Select all
ng

that apply.
A) Heavy rainfall is in the area.
si

B) The hospitals are 20 miles apart.


ur

C) The patient needs a time-sensitive intervention.


yn

D) Traffic on the ground is minimal.


.m

4. A patient with a spine injury is being transferred to another hospital. What special
intervention should the nurse make to prepare the patient for air transport?
w

A) Stabilize the patients back with a back board.


w

B) Send the patients medical chart with him.


w

C) Contact the receiving hospital to confirm that they have a bed for him.
D) Conduct preflight screen for patient fear or anxiety related to flying.
5. The medical director of hospital has received several requests for patient transfers to other
facilities. Based on the Emergency Medical Transfer Active Labor Act, which of the
following cases would justify interfacility transfer? Select all that apply.
A) A patient requires emergency coronary angioplasty and the sending hospital is not
equipped to perform this service.
B) The family of a patient severely injured in a car wreck in another state requests that
the patient be transferred to a facility where they live.
C) The administration of the hospital requests that a patient who lacks health insurance
be transferred to a facility that can better absorb the cost of his care.
D) A surgeon at the receiving hospital has unique experience performing a procedure
that the patient requires.
6. A Medicare patient who has been in the ICU for 6 weeks is being transferred to a larger
facility that can better support the costs of her care. The medical director at the sending
hospital weighs the pros and cons of ground versus air transport and opts for air transport

om
because of the long distance between the two hospitals. The receiving hospital is
contacted, and approval is given to transfer the patient. The nurse attending the patient
works with the transport team to make sure that all medical devices needed by the patient

.c
are available during the flight. The physician at the receiving hospital is waiting for the
patient as soon as she arrives. Which part of the transfer plan failed in this situation?

ep
A) Determination of the appropriateness of the transfer

pr
B) Selection of mode of transport
C) st
Communication between the sending and receiving hospitals
te
D) Collaboration among health care team members at the sending hospital
ng

7. The medical director of a hospital must decide whether to transfer a heart transplant
patient by ground or air transport to another facility that can provide the care the patient
si

needs. Which of the transfer is this decision made in?


ur

A) Phase one: Notification and acceptance by receiving facility


yn

B) Phase two: Preparation of the patient by the transport team


C) Phase three: The transport process
.m

D) Phase four: Turnover of the patient to the receiving facility


w

8. A patient is being prepared for transfer to another facility via air transport (phase two of
transfer). Which of the following is a role of the transport nurse at this time?
w
w

A) Suggest ground transport for the patient, as the receiving hospital is not far away.
B) Screen the patient for fear or anxiety of flying.
C) Call in an updated report on the patients status to the RN on the receiving unit.
D) Delivery of patient belongings to the receiving staff.
9. A nurse in the ICU participates in an evaluation of a recent patient transport. In which
phase of patient transfer does this occur?
A) Phase two: Preparation of the patient by the transport team
B) Phase three: The transport process
C) Phase four: Turnover of the patient to the receiving facility
D) Phase five: Post-transport continuous quality improvement monitoring
10. The transport nurse is accompanying her patient in transit by helicopter from one facility
to another. Which of the following is an intervention the nurse would most likely be
making at this time?
A) Contacting the RN on the receiving unit to note a change in the patients vital signs
B) Administering antiemetic medication to the patient to prevent air sickness

om
C) Endotracheal intubation
D) Communication with the patients family regarding details of the transport

.c
11. The nurse is on a committee establishing a rapid response team for the hospital. What is

ep
the best rationale for establishing a rapid response team?

pr
A) Mandated by accreditation standards
B)
st
Provides early treatment of hemodynamic instability
te
C) Relieves general floor staff of need to assess patients
D) Takes the place of cardiopulmonary resuscitation teams
ng

12. The nurse is assisting in the establishment of a rapid response team for the hospital.
si

Effective use of an RRT requires clear understanding of the role of the team. What is the
major function of an RRT?
ur

A) Perform bedside invasive procedures on general nursing units.


yn

B) Assess all hospital patients for initial signs of hemodynamic instability.


.m

C) Provide critical care expertise on general nursing units when needed.


D) Act as the transport team when patients require transfers to other facilities.
w
w

13. The nurse is on a committee working toward establishing a rapid response team. What is
the most important consideration in establishing such a team?
w

A) Number of cardiopulmonary arrests in the hospital yearly


B) Gaining support of senior hospital leadership
C) Availability of nurse practitioner to lead the team
D) Identifying possible negative effects on patient outcomes
14. Based on family request, a critically ill patient is being transferred from one facility to
another. What is the most important criterion the nurse should determine before the
transfer?
A) How stable the patient is
B) If the receiving facility has a bed available
C) Who the attending physician is
D) If the benefits of transfer outweigh the risks
15. A critically ill patient is being transferred from one agency to another. Under what
circumstances should the transfer most clearly be done by air as opposed to ground
transport?
A) Current weather includes limited visibility

om
B) Minimal ground traffic
C) Ground transfer takes too much time

.c
D) Agencies are in close proximity

ep
16. The nurse is preparing a critically ill patient for interagency transfer by air transport.

pr
What nursing action most clearly illustrates the physiologic changes expected during air
transport?
A)
st
Ensuring that copies of medical records and diagnostics go with the patient
te
B) Giving the family clear directions to the receiving agency
ng

C) Inserting a nasogastric tube before the transfer


si

D) Ensuring that the patient is heavily sedated during transport


ur

17. The nurse is preparing a critically ill patient for transfer to another facility. Based on the
criteria established by the Emergency Medical Transfer Active Labor Act (EMTALA),
yn

what patient situation would prevent transfer?


A) The sole reason for transfer is the patients uninsured status.
.m

B) The patient has been stabilized before the transfer.


w

C) The receiving agency has the expertise to care for the patient.
w

D) Transfer records are complete and sent with the patient.


w

18. Two agencies are discussing implementing a collaborative interfacility transfer plan.
What is the most important component of such a plan?
A) One agency has more sophisticated medical capabilities than the other.
B) Each agency has a clear understanding of its roles and responsibilities.
C) Transfers are accomplished with maximum speed and efficiency.
D) The two agencies are separated by distance necessitating air transport.
19. The nurse is initiating the transfer of a critically ill patient from one agency to another.
During the first phase of this transfer, what is the most important nursing responsibility?
A) Ensure that appropriate care equipment is available in the transfer vehicle.
B) Determine the willingness of the receiving agency to accept the patient.
C) Give the family detailed directions to the receiving agency.
D) Copy the patients medical records to accompany him or her.
20. A critically ill patient needs an x-ray procedure that cannot be done at the bedside. What
precautions should the nurse implement to protect the patient during transfer to the

om
radiology department?
A) Arrange for the usual patient transport technicians to transfer the patient.

.c
B) Ensure that a critical care nurse and one other person accompany the patient.

ep
C) Assume that all necessary equipment will be ready in the radiology department.

pr
D) Keep the patients medical record in the critical care unit to preserve privacy.

st
21. A patient who fell off a horse and may have a cervical spine fracture is being transferred
to another agency for more definitive care. During phase two of the transfer, the presence
te
of what equipment would be most important for this patient?
ng

A) Two large-bore intravenous lines


B) A cardiac defibrillator
si

C) A cervical collar
ur

D) Supplemental oxygen therapy


yn

22. A patient with severe blood loss from a motor vehicle crash is being transported by air
transport for definitive care. During phase two of the transfer, what is the most important
.m

nursing action?
w

A) Determine the route and method for transport.


w

B) Evaluate the patient outcome after transport.


w

C) Verify that the receiving facility will accept the patient.


D) Start at least two large-bore intravenous lines.

Answer Key

1. A
2. A, B, D
3. A, B, D
4. D
5. A, B, D
6. A
7. A
8. B
9. D

om
10. A
11. B

.c
12. C

ep
13. B

pr
14. D
15. C st
te
16. C
ng

17. A
18. B
si

19. B
ur

20. B
yn

21. C
.m

22. D
w

Chapter 15 Disaster Management: Implications for the Critical Care Nurse


w

1. A nurse from the ICU is participating in the hospitals disaster response preparedness
w

team. One issue that proves difficult for the team to agree on is a statement regarding the
standard of medical care observed during a disaster. Which of the following do you think
the nurse should recommend to the team?
A) The goal should be to provide the highest care possible, with limited resources and
equipment.
B) The lack of resources should not diminish the standard of care that the hospital
provides.
C) The medical staff should tend to the needs of the most critically ill first.
D) If electrical power should be lost to the facility, patients on life support should be
given lowest priority.
2. A nurse learns that local law enforcement officials have informed the hospital that an
imminent terrorist attack has been threatened in a building just down the street from the
hospital. Which of the following are appropriate responses? Select all that apply.
A) Explain to the patients in the ICU that a terrorist attack is expected and that their
care may be interrupted.
B) Begin making preparations to move all ICU patients to other hospital facilities in the
area in the event of an attack.

om
C) Review the hospitals disaster plan and make sure that it is distributed to the rest of
the medical staff.

.c
D) Determine what her specific role in the disaster plan is.

ep
E) Not be concerned because federal deployable medical teams will likely be sent to
the hospital in the event of an attack.

pr
F) Check the number of ventilators available in the ICU to determine whether more
st
would be needed in the event of an attack.
te
3. The nurse is at the bedside of her 90-year-old patient, Ruth, who is comatose and on life
support in the ICU, when she begins to feel the room shaking violently. The power
ng

suddenly fails and emergency generators have not started yet. The nurse provides bag-
mask resuscitation for Ruth while she waits for the power to be restored. Moments later,
si

the ICU is inundated with patients injured by the collapse of a nearby apartment building
as a result of the earthquake. The nurse is called to help. Kevin has been impaled by a
ur

metal rod through the chest, is in a state of shock, and will die without immediate
intervention. Gwyneth has compound fractures of the femur and a dislocated shoulder, is
yn

in pain, but is responsive. Mason is unconscious and unresponsive. Based on the START
triage categories, whom should the nurse assist first in this situation?
.m

A) Ruth
w

B) Kevin
w

C) Gwyneth
w

D) Mason
4. A patient arrives at the ICU after being injured by car bomb that exploded 20 feet away.
The patient sustained only primary blast injuries. Which of the following are injuries he
might have sustained? Select all that apply.
A) Perforated eardrum caused by a sudden change in atmospheric pressure
B) Laceration from a shard of glass that struck the patient
C) Concussion as a result of his body being thrown against a brick wall
D) Hemorrhagic contusion of the lungs
E) Gastrointestinal hemorrhage
F) Blunt force trauma from a piece of metal shrapnel that struck the patients head
5. A group of patients, colleagues from the same office, arrive at the ICU with symptoms of
nausea, vomiting, and diarrhea. It is determined that they are suffering from radiation
exposure as a result of an inconspicuous device placed in the office that leaked radiation
over a period of days. The nurse suspects that which of the following was used in this
terrorist attack?

om
A) Radiological dispersal device
B) Improvised nuclear device

.c
C) Nuclear weapon

ep
D) Simple radiological device
6. A patient arrives at the ICU with symptoms of radiation exposure. While in the ICU, he

pr
begins gasping for air and clutching his throat with his hand. What intervention should
take priority at this point?
A)
st
Perform CPR on the patient to restore normal respiratory function.
te
B) Evaluate the degree of radiation exposure in the patient using a Geiger counter.
ng

C) Undress the patient and have him shower and wash with soap to decontaminate
si

himself.
ur

D) Administer potassium iodide.


7. A woman arrives at the ICU after being the victim of a terrorist attack in a nearby
yn

shopping center involving a gas that spewed from a metal canister. The woman complains
of a burning sensation in her eyes, mouth, and throat, and the nurse observes blistering on
.m

her face and arms. What chemical agent should the nurse suspect, and what intervention
should she implement?
w

A) Vesicant agent; apply lotion


w

B) Nerve agent; administer benzodiazepine


w

C) Vesicant agent; soap and water blot


D) Nerve agent; provide ventilatory support
8. A worker at a local chemical plant arrives at the ICU following an industrial accident
involving a gas leak. The patient shows signs of pulmonary edema and bronchospasm.
What chemical should the nurse suspect is involved, and what intervention would be most
appropriate?
A) Nitrogen mustard; soap and water blot
B) Chlorine; airway management and ventilatory support
C) Tear gas; irrigation of eyes with water
D) Cyanide; sodium nitrate
9. A man presents to the ICU with a severe case of H1N1 viral infection. What would be the
most appropriate intervention for this patient?
A) Administer an H1N1 vaccine.
B) Administer an antibiotic.

om
C) Administer an antiviral agent.
D) Administer potassium iodide.

.c
10. A young man arrives at the ICU after being held hostage while a passenger on a
commercial airplane. He sustained a bullet wound in his chest and has undergone surgery

ep
to repair his lung. He is now receiving an analgesic for his pain. The nurse observes that
the patient frequently complains that he is sick to his stomach and has no appetite. His

pr
sleep is regularly interrupted by nightmares, and he is prone to outbursts of anger and
grief. What is the most likely cause of these symptoms?
A) Adverse reaction to pain medication
st
te
B) Reaction to severe pain
ng

C) Post-traumatic stress disorder


si

D) Delirium resulting from an infection


ur

11. The nurse is caring for a group of patients from the same workplace who have similar
symptoms of cough, respiratory distress, and nausea and vomiting. What should the nurse
yn

suspect?
.m

A) Food poisoning at local restaurant


B) Mass exposure to an inhaled agent
w

C) Allergic response to a new cleaning agent


w

D) Acute asthma exacerbation syndrome


w

12. The nurse is part of a planning committee developing a disaster response plan for a
hospital that serves as the major trauma center for the local area. The committee has
identified that the most likely cause of a disaster in the community is a devastating
tornado that significantly damages most major structures, including hospitals of all sizes,
in the community. What component of the plan is least likely to be realistic?
A) Transfer all noncritical patients to smaller hospitals in the community.
B) Provide for secure storage of emergency supplies and equipment.
C) Agree with other hospitals in town to share supplies and equipment.
D) Interface with the city disaster management plan and command center.
13. The nurse is preparing to assist with triage of victims from a large mass casualty incident.
The patients are sorted into categories at the door of the facility. What category will
receive care last?
A) Minimal
B) Delayed
C) Immediate

om
D) Expectant
14. The nurse is assisting with triage in the emergency department. A large group of patients

.c
from a mass casualty incident arrive. These patients have been classified at the scene as

ep
minimal, delayed, immediate, and expectant. What is the best nursing approach?
A) Use the classifications from the scene to determine the order of treatment.

pr
B) Reassess and reclassify patients quickly to determine the order of treatment.
C)
st
Treat the patients in the order they arrive at the emergency department.
te
D) After the first 20 patients, refer all others to another emergency care facility.
ng

15. The nurse is caring for a patient who was exposed to a high dose of external radiation.
What is the least likely nursing action?
si

A) Assess for and facilitate treatment of life-threatening injuries.


ur

B) Assess radiological measurements with a Geiger counter.


yn

C) Remove clothing and shower patient as soon as possible.


.m

D) Assist in removing penetrating radioactive materials.


16. The nurse is caring for a patient who has been exposed to radiation. The patient has been
w

increasingly unstable, with decreasing lymphocytes, leukocytes, thrombocytes, and


w

erythrocytes, along with shock, diarrhea, and altered level of consciousness for some
weeks. Today, there is clear evidence of worsening shock, subnormal body temperature,
w

and increased intracranial pressure. The family asks what the patients prognosis is. The
best nursing response is based on what rationale?
A) Increased intracranial pressure following other symptoms is a sign of impending
death.
B) The symptoms listed are typical of the latent phase of recovery, which lasts several
weeks.
C) Full recovery from radiation exposure can take many weeks to months.
D) The absence of fever indicates the patient has entered the latent phase of recovery.
17. The nurse is caring for a patient with a suspected cyanide exposure. The patient is
anxious and hyperventilating. What is the nursing priority of care?
A) Give antiseizure medications.
B) Send toxicology screen.
C) Give cyanide antidote.
D) Obtain history of exposure.

om
18. The nurse is assisting with the initial care and assessment of a group of patients with a
massive topical toxic chemical exposure. What is the best way to decontaminate these
patients?

.c
A) Use an antidote to the chemical of exposure.

ep
B) Use an alkaline substance for an acid contaminant.

pr
C) Soap-and-water shower first for most chemicals
D)
st
Administer 100% oxygen under positive pressure.
te
19. A worker in a tanning factory comes to the emergency department with itchy, papular
lesions on his hands and arms. Some of the lesions have black eschar in the center and
ng

some are vesicular. What biological exposure is most likely?


A) Cutaneous anthrax
si

B) Pulmonary anthrax
ur

C) Cutaneous smallpox
yn

D) Pneumonic plague
.m

20. A patient and his family have been exposed to probable anthrax spores through
contaminated postal contents within the past 8 hours. Upon arrival at the emergency
w

department, all members of the family are anxious and say they are afraid of dying. The
best nursing response is based on what rationale?
w

A) There is no treatment for anthrax exposure and their fears are realistic.
w

B) A vaccine given in the first 3 days after exposure will stop the disease.
C) Antibiotic therapy is very effective for cutaneous anthrax.
D) A toxicology screen is necessary to determine whether there really was an exposure.

Answer Key

1. A
2. C, D, F
3. B
4. A, D, E
5. D
6. A
7. C
8. B

om
9. C
10. C

.c
11. B

ep
12. A

pr
13. D
14. B st
te
15. D
ng

16. A
17. C
si

18. C
ur

19. A
yn

20. C
.m

Chapter 16 Anatomy and Physiology of the Cardiovascular System


w

1. The nurse is explaining to a patient with dysrhythmia how the heart conducts impulses
w

that cause myocardial cells to contract simultaneously. The patient asks what structure
allows this conduction from cell to cell. Which structure should the nurse mention?
w

A) Intercalated disks
B) Nuclei
C) Mitochondria
D) Ribosomes
2. A nurse is explaining to a cardiac patient how mechanical contraction takes place in the
myocardial cells in the heart. The nurse should explain that which of the following is
needed for contraction? Select all that apply.
A) ATP
B) Calcium
C) ADP
D) Phosphorus
E) Iron
F) Zinc
3. Which of the following statements is true of cell membrane depolarization? Select all that

om
apply.
A) In the resting myocardial cell, there are more potassium ions outside than inside the
cell.

.c
B) In the depolarized state, the cell membrane is positively charged.

ep
C) Excitability is the term used to describe the ability of a cell to depolarize in response

pr
to a given stimulus.
D)
st
Pumps move sodium out of the cell and potassium into the cell.
te
4. What is the role of the atrioventricular node in a person with normal heart rate (60 to 100
bpm) and why?
ng

A) Acts as primary pacemaker, to establish the heart rate


si

B) Accelerates conduction of atrial action potentials so that all ventricular cells contract
almost simultaneously
ur

C) Delays conduction of atrial action potentials so that all ventricular cells contract
yn

almost simultaneously
D) Pumps sodium out of the myocardial cell and potassium into it to cause cell
.m

membrane depolarization
w

5. A nurse is working with a cardiac patient who is anxious about his upcoming surgery. At
his request, the nurse hands him his MP3 player. After the patient listens to his MP3
w

player for a few moments, the nurse notices on the patients heart rate monitor that his
w

heart rate has decreased somewhat. What physiological response would explain this
decrease in heart rate?
A) Stimulation of the sympathetic nervous system and the subsequent release of
norepinephrine
B) Release of epinephrine from the adrenal medulla
C) Release of norepinephrine from the adrenal medulla
D) Stimulation of the parasympathetic nervous system and the subsequent release of
acetylcholine
6. The demand for oxygen in heart cells as a result of inotropic action is known as which of
the following?
A) Preload
B) Afterload
C) Contractility
D) Automaticity
7. Which of the following statements regarding coronary circulation is correct?

om
A) In about 90% of the population, the right coronary artery is source of blood flow to
the atrioventricular node.
B) In about 100% of the population, the left circumflex artery is the source of blood

.c
flow to the sinoatrial node.

ep
C) The right coronary artery is the source of blood flow to the bundle branches.

pr
D) The left anterior descending artery is the source of blood to the left atrium.

st
8. A patient has inadequate perfusion to her extremities. Which of the following could
explain this phenomenon?
te
A) Increased diastolic pressure
ng

B) Arteriole dilation
si

C) Decreased vascular resistance


ur

D) Decreased blood viscosity


yn

9. A patient has an irregular heartbeat. What is the most likely cause of this?
A) Excitability in cardiac tissue
.m

B) Contractility in cardiac muscle


w

C) Automaticity in cardiac muscle cells


w

D) Conductivity in the AV node


w

10. The double-layered sac that surrounds the heart is known as which of the following?
A) Endocardium
B) Pericardium
C) Myocardium
D) Epicardium
11. A patient has been prescribed calcium blockers. The nurse should expect these
medications to have what effect on the patient?
A) Decreased blood volume
B) Irregular heartbeat
C) Decreased arterial pressure
D) Tachycardia
12. In evaluating the cardiac function of a patient, the nurse understands that cardiac muscle
cells have unique properties that are not found in skeletal muscle. What is the best
example of such a unique property?
A) Automaticity

om
B) Responsiveness
C) Contractility

.c
D) Elasticity

ep
13. While teaching a patient, the nurse explains that cardiac functioning has both a

pr
mechanical and an electrical component. What is the best way to assess the mechanical
function of the heart?
A) Blood pressure
st
te
B) Pulse
ng

C) Membrane potential
si

D) Cardiac monitor pattern


ur

14. Part of cardiac function involves the movement of ions in and out of cells, resulting in
changes in the polarity of the cell membrane. As part of a teaching plan, the nurse
yn

explains that the term depolarized means that the cardiac cell membrane is what?
A) In transition
.m

B) Electrically neutral
w

C) Negatively charged
w

D) Positively charged
w

15. Part of understanding cardiac electrical function is understanding the normal conduction
system of the heart. List the following parts in normal functional order.

A) Bundle of His

B) Intranodal pathways

C) Atrioventricular node
D) Left bundle branches

E) Sinoatrial node

F) Purkinje fibers
16. A patient has compromised cardiac output. What parameters would the nurse assess to
monitor cardiac output most fully?
A) Stroke volume and blood pressure
B) Heart rate and stroke volume

om
C) Stroke volume and pulse pressure
D) Heart rate and contractility

.c
17. The patient is receiving a beta-receptor antagonist medication. What is the most

ep
significant effect on cardiac function that the nurse should be monitoring for?

pr
A) Decreased cardiac output
B) Supraventricular tachycardia
C) Increased stroke volume
st
te
D) Increased blood pressure
ng

18. The nurse is evaluating a patients cardiac output. What situation would be most indicative
si

of increased preload?
ur

A) Sinus tachycardia
B) Elevated central venous pressure
yn

C) Elevated blood pressure


.m

D) Evidence of myocardial infarction


w

19. The nurse is caring for a patient in cardiac failure who has very high afterload. What is
the most direct adverse effect on cardiac function that the nurse would anticipate?
w

A) Increased cardiac output


w

B) Decreased heart rate


C) Decreased stroke volume
D) Increased heart rate
20. The patient has been diagnosed with a severe blockage of the proximal left anterior
descending artery. What cardiac disturbance does the nurse anticipate as most likely?
A) Arrhythmias from the sinoatrial node
B) Arrhythmias from the atrioventricular node
C) Atrial muscle dysfunction and failure
D) Disturbance in bundle branch conduction
21. A patients heart rate is 45 bpm and regular. What part of the conduction system is most
likely initiating the rhythm?
A) Sinoatrial node
B) Atrial muscle tissue

om
C) Atrioventricular node
D) Purkinje fibers

.c
22. The patient has a heart rate of 80 bpm, blood pressure of 140/80 mmHg, and stroke
volume of 100 mL/beat. The nurse calculates the patients cardiac output as ____ L/min?

ep
23. The patient has a peripheral capillary refill time of less than 2 seconds. What relationship

pr
between blood volume and peripheral resistance would the nurse expect to find?
A)
st
Elevated preload and blood pressure measurements
te
B) Normal preload and blood pressure measurements
C) Diminished preload and blood pressure measurements
ng

D) No relationship between preload and blood pressure


si
ur

Answer Key
yn

1. A
.m

2. A, B
3. C, D
w

4. C
w

5. D
w

6. C
7. A
8. A
9. C
10. B
11. C
12. A
13. B
14. C
15. E, B, C, A, D, F
16. B
17. A
18. B

om
19. C
20. D

.c
21. C

ep
22. 8 L/min

pr
23. B

Chapter 17 Patient Assessment: Cardiovascular System


st
te
1. The patient is complaining of chest pain. The nurse asks several questions to determine
ng

the likelihood that this pain is cardiac in origin. What patient response is most likely to
indicate cardiac pain?
si

A) Started suddenly and was at maximum intensity from the start


ur

B) Accompanied by no other symptoms, such as nausea


yn

C) Painful area is the size of a clenched fist and substernal.


.m

D) Described as stabbing and knifelike pain


2. As part of the general physical assessment of a patient with cardiac disease, the nurse
w

assesses central and peripheral pulses. What is the most accurate statement about pulse
w

assessment?
w

A) All pulses should be assessed bilaterally at the same time.


B) A pulse that is weak and thready would be scaled +4.
C) Pulsus alternans varies in intensity with every other beat.
D) Absence of posterior tibial pulses is a normal variant.
3. The nurse is auscultating a patients heart sounds. What best describes a normal finding?
A) S2 is heard best at the second intercostal space to the right of the sternum.
B) S2 is heard equally at all areas across the precordium and apex.
C) S1 usually disappears after 25 to 35 years of age.
D) S3 is caused by retrograde blood flow though an incompetent valve.
4. A patient has been determined to have a new heart murmur. The best description of the
cause of a cardiac murmur is what?
A) The patient had rheumatic heart disease as a child.
B) A murmur is caused by the flow of blood through a narrow or incompetent valve or
septum.
C) A murmur indicates a ruptured ventricular septum from myocardial disease or

om
infarction.
D) A murmur occurs normally secondary to turbulent blood flow in children and young

.c
adults.

ep
5. The patient is to undergo a diagnostic electrophysiology study. The nurse should tell the
patient and family that the purpose of this study is what?

pr
A) Identify blockages of coronary arteries.
B) st
Measure volumes and pressures in cardiac chambers.
te
C) Evaluate the efficacy of antiarrhythmic drugs and devices.
ng

D) Evaluate patient response to pharmacologically elevated heart rates.


6. The patient is undergoing an echocardiogram for diagnosis of cardiac valve dysfunction.
si

What factor is most likely to result in an accurate echocardiogram?


ur

A) The patient has chronic obstructive pulmonary disease (COPD).


yn

B) The technologist performing the test is very experienced.


C) The patient is very obese, with large pendulous breasts.
.m

D) The echocardiogram machine is several years old.


w

7. The nurse is caring for a patient using a 5-lead continuous cardiac monitor system. To
w

monitor lead V6, where does the nurse place the chest electrode?
w

A) Fourth intercostal space on right sternal border


B) Right shoulder on muscle tissue close to bone
C) Left shoulder on muscle tissue close to bone
D) Fifth intercostal space on left mid-axillary line
8. The nurse is applying electrodes to the patients chest for use in continuous cardiac rhythm
monitoring. When applying these electrodes, what placement will the nurse use to ensure
maximum accuracy?
A) In the fold beneath the breast tissue
B) Directly over the clavicle
C) In the intercostal spaces
D) Over the clavicle/humerus joint
9. A patient is on continuous cardiac rhythm monitoring. The nurse notices that the low-rate
alarm is sounding frequently. What nursing action takes the highest priority?
A) Assess monitor alarm limits for appropriate setting.

om
B) Check contact with skin for all electrodes.
C) Assess patients level of consciousness and vital signs.

.c
D) Check connections between lead wires and electrodes.

ep
10. The nurse is using the cardiac monitor tracing to evaluate cardiac function. The QRS
interval is 0.14 seconds in width. How does the nurse interpret this finding?

pr
A) Normal rate and rhythm
B) Slowed conduction through the atria st
te
C) Left ventricular bundle branch block
ng

D) Slowed ventricular depolarization


11. A patient in the CCU is suspected of having coronary artery disease (CAD) following
si

myocardial infarction. Which blood test or tests would be most pertinent in evaluating
ur

this patients condition? Select all that apply.


yn

A) Red blood cell count


B) Low-density lipoproteins
.m

C) Prothrombin time
w

D) Triglycerides
w

E) White blood cell count


w

F) High-density lipoproteins
12. A patient in the CCU is suspected to have had a myocardial infarction. His blood work
results have arrived, and the nurse is reviewing them. Which results would indicate
myocardial infarction? Select all that apply.
A) Elevated troponins
B) CK-MB isoenzyme at 10% of total creatine kinase
C) C-reactive protein serum value of 6 mg/dL
D) Decreased levels of D-dimer
13. A patient is being discharged from the CCU, and the cardiologist would like to monitor
her cardiac activity over the coming month. This patient typically experiences infrequent
dysrhythmias and is willing to wear electrodes, carry a recorder, and keep a diary. Which
type of cardiac monitoring would be most appropriate for this patient?
A) Holter monitoring
B) Event (continuous loop) monitoring
C) Implantable loop monitoring

om
D) Signal-averaged electrocardiography
14. A patient in the CCU must perform exercise stress testing. Which of the following would

.c
be the most appropriate intervention for the nurse to make in preparing the patient for this
test?

ep
A) Provide the patient with a cup of coffee to help him be alert during the testing.

pr
B) Continue the patient on beta blockers throughout the test.
C) st
Explain to the patient that he may have to continue exercising after angina develops.
te
D) Feed the patient an hour before the test so that he has plenty of energy.
ng

15. A patient in the CCU will undergo a diagnostic electrophysiology study. Which
complication should the nurse monitor for during this procedure?
si

A) Bleeding
ur

B) Vascular damage
yn

C) Lethal dysrhythmias
D) Cardiac arrest
.m

16. A nurse in the CCU must assess a patients cardiac rhythm strip. Which of the following
w

represents the first three steps in this assessment in correct order?


w

A) Look for P waves, measure the PR interval, evaluate the QRS interval.
w

B) Evaluate the QRS complex, examine the ST segment, identify the rhythm.
C) Determine atrial and ventricular heart rates, examine the rhythm to see if it is
regular, look for P waves.
D) Look for P waves, examine the rhythm to see if it is regular, determine atrial and
ventricular heart rates.
17. A patient in the CCU who is undergoing ECG monitoring has impulses originating at the
sinoatrial node at a rate of 75 bpm, a shortest RR interval of 0.69 seconds, and a longest
RR interval of 0.84 seconds. The P wave is regular. Which sinus rhythm does this patient
exhibit?
A) Sinus tachycardia
B) Sinus bradycardia
C) Sinus dysrhythmia
D) Sinus arrest
18. A patient in the CCU has been found on examination by ECG to have a bundle branch
block. Which of the following should the nurse expect to see on this patients ECG?

om
A) Tall, pointed P waves in leads II, III, and aVF
B) Broad, notched P waves in leads I, II, and aVL

.c
C) A tall R wave in V1 and progressively smaller R waves through V6

ep
D) A broad QRS complex with two peaks (RSR)

pr
19. A patient in the CCU exhibits a tall, narrow, and peaked T wave on ECG. The nurse
st
recognizes that this is most likely an early sign of which of the following?
te
A) Hypokalemia
B) Hyperkalemia
ng

C) Hypocalcemia
si

D) Hypercalcemia
ur

20. A patient in the CCU is undergoing arterial pressure monitoring, and the bedside
yn

physiological monitor alarm sounds. Which of the following should the nurse do? Select
all that apply.
.m

A) Check to see whether the catheter is kinked.


B) Check to see whether the stopcocks are turned the wrong way.
w
w

C) Add IV solution through the arterial pressure monitoring system.


w

D) Make sure that there is sufficient pressure in the pressure bag.

Answer Key

1. C
2. C
3. A
4. B
5. C
6. B
7. D
8. C
9. C
10. D

om
11. B, D, F
12. A, B, C

.c
13. B

ep
14. C

pr
15. A
16. C st
te
17. C
ng

18. D
19. B
si

20. A, B, D
ur
yn

Chapter 18 Patient Management: Cardiovascular System

1. A nurse is working with multiple heart patients in the critical care unit. What information
.m

in the patients charts would indicate intra-aortic balloon pump (IABP) counterpulsation?
Select all that apply.
w

A) Cardiogenic shock subsequent to acute myocardial infarction


w

B) Low cardiac output following cardiac surgery


w

C) Unstable angina
D) Congestive heart failure and an incompetent aortic valve
E) Severe peripheral vascular disease
F) Aortic aneurysm
2. A patient with cardiac failure is being prepped to receive a ventricular assist device for
circulatory support. What procedure should the nurse expect the patient to undergo in the
future?
A) Intra-aortic balloon pump counterpulsation
B) Heart transplantation
C) Coronary artery bypass graft surgery
D) Percutaneous transluminal coronary angioplasty
3. A nurse working with a patient with a Novacor ventricular assist device (VAD) for long-
term circulatory support recognizes that this patient is at risk for thromboembolic events.
Which of the following is the best intervention for this patient?
A) Heparin infusions indefinitely

om
B) Warfarin therapy initially, which is gradually replaced by heparin infusions
C) Warfarin therapy indefinitely

.c
D) Heparin infusions initially, which is gradually replaced by warfarin therapy

ep
4. What should the nurse include in the plan of care for a patient being prepared for

pr
electrical cardioversion? Select all that apply.
A) Obtain informed consent from the patient.
st
te
B) Encourage the patient to eat a meal shortly before the procedure.
C) Record a 12-lead ECG and vital signs.
ng

D) Turn off the synchronizer mode button.


si

E) Smear electrode gel between the two paddles on the chest.


ur

F) Sedate the patient.


yn

5. A nurse is caring for a patient during radiofrequency ablation. What nursing interventions
would most likely put the patient at ease?
.m

A) Communicating with the patient about the progress of the procedure


w

B) Connecting the patient to a cardiac monitor


w

C) Placing a lead apron under the patients lower back


w

D) Establishing a sterile field


6. A patient is being evaluated for possible permanent pacemaker implantation. Which of
the following would be the best indication for permanent pacemaker implantation in this
patient?
A) Myotonic muscular dystrophy with any degree of atrioventricular (AV) block
B) Asymptomatic sinus node dysfunction
C) Chronic, intermittent complete (bifascicular) heart block
D) Transient AV block after acute myocardial infarction (MI)
7. A patient requires a pacemaker that will pace his atria, sense electrical activity in his atria,
and inhibit the response to sensed events. Which mode of pacemaker would he need?
A) ADD
B) VVI
C) AAI
D) VDD

om
8. A patient who had a transvenous pacemaker placed in the subclavian vein 24 hours ago is
experiencing pleuritic pain, hypotension, and respiratory distress. What complication
does the nurse suspect in this patient?

.c
A) Perforation of ventricular wall

ep
B) Pneumothorax

pr
C) Lead dislodgment
D) Phlebitis
st
te
9. A patient is experiencing a malfunction with his permanent implanted pacemaker, and the
nurse is examining his electrocardiogram strip. The patients pacemaker has a ventricular
ng

unipolar catheter and is in VVI mode. Which of the following findings should the nurse
be most concerned about?
si

A) Pacing spikes absent


ur

B) Pacing spike followed by a QRS complex


yn

C) Tall pacing spikes


D) A narrow QRS
.m

10. What information should the nurse include when teaching a patient with incessant
w

ventricular fibrillation who is being considered for an implantable cardioverter-


defibrillator (ICD)?
w

A) An ICD would be beneficial for this patient.


w

B) An ICD would be reasonable to consider for this patient.


C) An ICD may be considered but is questionable for this patient.
D) An ICD would not be useful and may even be harmful for this patient.
11. The physician has ordered fibrinolytic agents for a patient with coronary artery disease.
What patient diagnosis would the nurse most expect?
A) Posterior wall acute myocardial infarction
B) Acute coronary syndrome without ST segment elevation
C) Acute ST segment elevation myocardial infarction
D) New onset of right bundle branch block
12. The patient is receiving an intravenous drip of lidocaine hydrochloride to suppress
ventricular arrhythmias. During repositioning of the patient, the intravenous tubing is
dislodged from the pump and the patient receives a bolus of approximately 300 mg. The
nurse stops the drip temporarily. What is the best nursing action to take next?
A) Observe closely for hallucinations and hypotension.

om
B) Reconnect drip at ordered rate per minute.
C) Administer intravenous magnesium sulfate.

.c
D) Complete unusual occurrence report form.

ep
13. A patient calls emergency services with complaints of crushing chest and left shoulder
pain. He should be instructed to:

pr
A) Take 325 mg acetaminophen.
B) Take 325 mg of aspirin.
st
te
C) Take an antacid.
ng

D) Drive to the ED.


si

14. The patient is receiving a positive inotropic drug by intravenous drip. If this therapy is
effective, what physical change would the nurse most expect to find?
ur

A) New-onset extra heart sound S3


yn

B) Increasing peripheral edema


.m

C) Increased urine output per hour


D) Basilar pulmonary crackles
w
w

15. A patient who has had an acute ST segment elevation myocardial infarction (STEMI) is
started on an angiotensin-converting enzyme (ACE) inhibitor drug. In explaining the
w

action of this drug to the patient, what is the best rationale for the nurse to use?
A) Most patients with acute myocardial infarction also have hypertension.
B) Reduction of afterload reduces stress on the damaged heart and further damage.
C) Use of ACE inhibitors is part of protocol for treatment of a STEMI.
D) ACE inhibitor therapy is not effective for other types of heart disease.
16. The patient is being evaluated for antihyperlipidemic therapy. The patient has two active
risk factors for coronary artery disease. The nurse expects that this therapy will be
initiated if which patient criteria are present?
A) Total cholesterol less than 200 mg/dL
B) Low-density lipoprotein greater than 160 mg/dL
C) High-density lipoprotein less than 40 mg/dL
D) Low-density lipoprotein less than 70 mg/dL
17. The patient is scheduled to undergo a percutaneous cardiac intervention (PCI). What

om
patient history would the nurse least expect to find?
A) Poor left ventricular function and ejection fraction

.c
B) Coronary artery lesions less than 70% narrowing

ep
C) Extreme old age with fragility

pr
D) Unstable angina with activity and at rest

st
18. A female patient has just undergone a percutaneous coronary intervention (PCI). What
symptom, if found by the nurse, requires immediate intervention?
te
A) Three premature ventricular complexes
ng

B) Frank bleeding from the femoral insertion site


si

C) Serum potassium level 4.8 mEq/L


ur

D) Hemoglobin 11.7 g/dL


yn

19. The patient has undergone a percutaneous coronary intervention (PCI) for relief of
coronary stenosis secondary to arteriosclerotic heart disease. During discharge teaching,
what patient statement most indicates the need for reteaching?
.m

A) This procedure means that my heart disease is cured.


w

B) I should continue to take my antilipidemic.


w

C) If I have any chest pain, I will call my doctor.


w

D) I will start a walking program after my doctor agrees.


20. The patient is scheduled for a percutaneous balloon valvuloplasty (PBV). What would the
nurse most expect to find in the patient history?
A) Acute myocardial infarction
B) Increased serum troponin
C) Stenotic cardiac valves
D) Severe peripheral edema
21. A patient is being treated with the intra-aortic balloon pump (IABP). What effect of the
balloon is most important?
A) Reduction of left ventricular workload
B) Improvement in right ventricular perfusion
C) Reduction of right ventricular preload
D) Minimal compromise of cardiac output

om
22. The patient is receiving intra-aortic balloon pump (IABP) counterpulsation. What is an
indication for this therapy?
A) Hypotension after ST elevation myocardial infarction refractory to other therapy

.c
B) High cardiac output in cardiogenic and other acute shock syndrome states

ep
C) Right-sided congestive heart failure refractory to other therapy

pr
D) Routine recovery from coronary artery bypass surgery in low-risk patients
st
23. The nurse is caring for a patient undergoing intra-aortic balloon pump (IABP)
te
counterpulsation. What patient symptom most requires immediate nursing intervention?
A) Patient complains of stiffness in insertion leg.
ng

B) Mean arterial blood pressure is increased by 10 points.


si

C) Balloon is inflating during ventricular systole.


ur

D) Cardiac output is increased by 3 L/min.


yn

24. The patient has a newly inserted VVI pacemaker set at a demand rate of 70. What cardiac
monitor reading, if found by the nurse, most indicates a pacemaker malfunction?
.m

A) Patient pulse is 75 with no pacing stimuli seen on cardiac monitor.


w

B) Patient pulse is 70 with pacing stimulus seen before each R wave.


w

C) Patient pulse is 65 with no pacing stimuli seen on cardiac monitor.


w

D) Patient pulse is irregular with pacing stimuli seen before some R waves.
25. The patient has just had a transvenous cardiac pacemaker lead inserted into the subclavian
vein. What patient symptom, if found by the nurse, would most indicate a possible
complication unique to the use of the subclavian vein?
A) Persistent hiccups at the same rate as the pacemakers set rate
B) Sudden respiratory distress, hypoxia, and hypotension
C) Persistent premature ventricular complexes seen on cardiac monitor
D) Oversensing, undersensing, or failure to capture
26. A patient has a permanent pacemaker and complains of redness and swelling over the site
of the generator implantation. The patient also has fever and leukocytosis. What is the
most appropriate nursing action?
A) Assess for any patient action causing pressure on the site.
B) Assure the patient that these are signs of normal healing.
C) Ensure that pacemaker is working correctly.
D) Facilitate administration of intravenous antibiotics.

om
Answer Key

.c
ep
1. A, B, C
2. B

pr
3. D
4. A, C, F st
te
5. A
ng

6. C
si

7. C
ur

8. B
9. A
yn

10. D
.m

11. C
w

12. A
w

13. B
w

14. C
15. B
16. B
17. B
18. B
19. A
20. C
21. A
22. A
23. C
24. C
25. B
26. D

om
Chapter 19 Common Cardiovascular Disorders

.c
1. A patient presents to the CCU with chest pain made worse by breathing deeply or lying

ep
supine, dyspnea, malaise, fever, and elevated blood cell count. On reviewing his medical
chart, the nurse observes that the patient had a myocardial infarction almost 2 months
ago. What does the nurse suspect the patients condition to be?

pr
A) Dresslers syndrome
B) Dilated cardiomyopathy
st
te
C) Hypertrophic cardiomyopathy
ng

D) Buergers disease
si

2. A 20-year-old competitive swimmer was recently admitted to the CCU and diagnosed
with myocarditis. The nurse is counseling the patient and her family regarding what this
ur

diagnosis means. What should the nurse mention? Select all that apply.
yn

A) Corticosteroids are effective in changing the clinical course of this disease.


.m

B) Immunosuppressive agents are effective in changing the clinical course of this


disease.
w

C) Some cases of myocarditis resolve without further sequelae.


w

D) The patient will need to withdraw from training and competition for at least 6
w

months.
E) The disease is potentially lethal and often has no cure.
F) The disease may require heart transplantation.
3. A patient in the CCU presents with endocarditis. This patient is 40 years old, a smoker,
had rheumatic heart disease as a child, and is obese. Which characteristics of this patient
are risk factors for endocarditis?
A) Age
B) Smoking status
C) History of rheumatic heart disease
D) Obesity
4. A patient in the CCU has an obstruction in his coronary arteries that is resulting in
oxygen levels that are inadequate to meet the metabolic demands of his myocardial cells.
He is exhibiting symptoms of left ventricular heart failure. Which disease does he most
likely have?
A) Ischemic cardiomyopathy

om
B) Nonischemic cardiomyopathy
C) Endocarditis

.c
D) Peripheral vascular disease

ep
5. A patient in the CCU is experiencing a cramping pain in the buttocks whenever she
stands. The nurse also observes that the patients nails on both hands are thick and her skin

pr
dry. The nurse recognizes this condition as which of the following?
A) Deep vein thrombosis st
te
B) Abdominal aortic aneurysm
ng

C) Peripheral arterial disease


D) Thoracic aortic aneurysm
si

6. A patient in the CCU has been confined to bed rest for 3 weeks. The patient is obese, is
ur

on warfarin, and has a peripherally inserted central catheter (PICC). Which of the
following are causative factors for thrombophlebitis in this patient? Select all that apply.
yn

A) Bed rest
.m

B) Obesity
w

C) Presence of a PICC
w

D) Warfarin
w

E) Insulin
7. The nurse observes a balloon-shaped outpouching on the abdomen of a cardiac patient in
the CCU. The nurse recognizes this as which of the following?
A) Fusiform aneurysm
B) False aneurysm
C) Saccular aneurysm
D) Thoracic aneurysm
8. A patient in the CCU has an abdominal aortic aneurysm that is 7 cm in diameter. The
nurse recognizes that which of the following is the most appropriate treatment for this
patient?
A) Anticoagulant therapy
B) Heart transplant
C) Genetic screening and counseling
D) Surgical repair
9. A pregnant patient in the CCU who recently was in a motor vehicle accident suddenly

om
complains of an intense chest pain that she describes as ripping before passing out. On
auscultation, the nurse hears the murmur of aortic regurgitation. What condition does the
woman most likely have?

.c
A) Thoracic aortic aneurysm

ep
B) Aortic dissection

pr
C) Hypertrophic cardiomyopathy
D) Hypertensive crisis st
te
10. An elderly man presents to the CCU with a blood pressure reading of 190/125, severe
headache, visual disturbances, and bleeding from his eyes. The nurse recognizes this
ng

condition as which of the following?


si

A) Pericarditis
ur

B) Endocarditis
C) Dilated cardiomyopathy
yn

D) Hypertensive crisis
.m

11. The patient was admitted with an acute myocardial infarction several days ago and is now
complaining of severe midsternal chest pain at rest and dyspnea. The pain is constant,
w

more severe when the patient lies down, and unaffected by medications. What is the best
w

nursing intervention?
w

A) Obtain cardiac marker blood tests to assess for reinfarction.


B) Auscultate for pericardial and pleural friction rubs.
C) Auscultate for new onset of S3 heart sound.
D) Prepare the patient for cardiac catheterization.
12. A patient has been diagnosed with pericarditis. Which assessment changes unique to
pericarditis would the nurse expect to find?
A) New onset of S3 gallop
B) Severe chest pain at rest
C) Diffuse ST segment elevation on ECG
D) Prolonged, irregular PR intervals on ECG
13. A patient with mitral valve prolapse has been admitted to the CCU with probable
infectious endocarditis. What physical changes unique to infectious endocarditis will the
nurse be most likely to find?
A) Track marks on arms following peripheral veins
B) New onset of or change in cardiac murmur

om
C) Severe midsternal crushing chest pain with activity
D) Fever, tachycardia, and general malaise

.c
14. A patient is admitted with complications and exacerbation of symptoms from dilated

ep
cardiomyopathy. Which easily treated factor might account for the change in the patients
condition?

pr
A) Viral illness before the original diagnosis
B) Acute myocardial infarction
st
te
C) Chronic alcohol abuse
ng

D) Persistent vegetative growth on valves


si

15. A young adult collapses during a sporting event. Cardiopulmonary resuscitation is


successful, and the patient is admitted with a diagnosis of possible hypertrophic
ur

cardiomyopathy. What nursing assessment is a priority for this patient?


yn

A) Cardiac monitor pattern and pulse oximetry


B) Evaluation for dyspnea and heart murmur
.m

C) Level of consciousness and mental status evaluation


w

D) History of recent viral illness or alcohol abuse


w

16. A patient is being treated for hypertrophic cardiomyopathy. During discharge teaching,
w

what is the most important information to include?


A) Cessation of alcohol abuse will improve cardiac function.
B) Only a heart transplant is associated with increased longevity.
C) Make an appointment for genetic screening and counseling.
D) Information about anti-infective medications
17. A patient with chronic arteriosclerotic disease complains of severe pain in the calf, foot,
and ankle, followed by numbness. If the patient has an embolus of the lower extremity
arteries, what additional physical sign will the nurse find?
A) History of claudication
B) Black tips of toes
C) No foot pulses on affected foot
D) Bilateral cool skin on lower extremities
18. A patient is diagnosed with Buergers disease. What would be the most important teaching

om
for this patient?
A) Information about anti-inflammatory medications

.c
B) Importance of protecting the extremities from cold

ep
C) Rationale for quitting smoking

pr
D) Anticoagulant therapy precautions

st
19. A patient is critically ill in the CCU and is on complete bed rest. What strategy should the
nurse use to prevent phlebitis leading to thromboembolism?
te
A) Repeated attempts to insert intravenous catheters
ng

B) Maintain overall blood volume slightly low


si

C) Maintain complete bed rest


ur

D) Passive exercises of lower extremities


yn

20. A patient is admitted for repair of an abdominal aortic aneurysm and begins to complain
of severe abdominal pain and a sense of impending doom. What assessment should the
nurse perform first?
.m

A) Blood pressure
w

B) Respiratory rate
w

C) Auscultation of heart sounds


w

D) Pain scale
21. A patient with a history of hypertension has been admitted for surgical repair of a thoracic
aneurysm. What symptom would indicate probable acute dissection of the aneurysm?
A) Nonspecific mild chest pain
B) Absence of cardiac murmur
C) Wide mediastinum on chest x-ray
D) Equal radial pulses at normal intensity
22. A patient has been admitted with hypertensive crisis and is experiencing headache, visual
disturbances, confusion, nausea, papilledema, and oliguria. The patient has been started
on an intravenous vasodilator to reduce his blood pressure quickly. What is the priority
nursing assessment?
A) Protection of safety with side rails and restraints
B) Continuous blood pressure monitoring by arterial catheter
C) Response to pain and nausea medications

om
D) Insertion of nasogastric tube to intermittent suction
23. A critical care nurse is caring for a client in a hypertensive crisis. Blood pressure is being

.c
reduced slowly. If the blood pressure is lowered too much too quickly, what is the patient
is at risk for?

ep
A) A myocardial infarction

pr
B) A cerebral infarction
C) Renal failure st
te
D) All of the above
ng

Answer Key
si

1. A
ur

2. C, D, E, F
yn

3. C
.m

4. A
w

5. C
w

6. A, C
w

7. C
8. D
9. B
10. D
11. B
12. C
13. B
14. C
15. B
16. C
17. C
18. C
19. D

om
20. A
21. C

.c
22. B

ep
23. C

pr
Chapter 20 Heart Failure
st
1. A patient in the CCU with chronic heart failure is prescribed an ACE inhibitor. What side
te
effects should the nurse mention to him? Select all that apply.
ng

A) Angioderma
si

B) Cough
ur

C) Rebound tachycardia
D) Hyperkalemia
yn

E) Night sweats
.m

F) Anxiety
w

2. A CCU nurse who works frequently with cardiac patients is putting together a teaching
plan to follow when she instructs these patients on how to live with heart failure. Which
w

points should she include in this plan? Select all that apply.
w

A) Discontinue medications once you are feeling better, to avoid adverse effects.
B) Take your medications about the same time every day.
C) Avoid pepper and spices.
D) Remove the saltshaker from your table.
E) The best time to weigh yourself is in the afternoon.
F) Try to perform 15 to 20 minutes of continuous activity each day.
3. A patient with chronic cardiac failure in the CCU is on an ACE inhibitor but still has
significant pitting edema in his extremities. Which medication, in addition to the ACE
inhibitor, is the physician likely to prescribe to this patient?
A) Digoxin
B) A loop diuretic
C) b-blocker
D) Calcium channel blocker
4. A patient presents to the CCU with acute, decompensated heart failure. The nurse

om
observes that this patient has chronic obstructive pulmonary disease and hypotension. She
eats a low-sodium diet and drinks alcohol excessively. Which of the following are
contributing factors to this patients heart failure? Select all that apply.

.c
A) Chronic obstructive pulmonary disease

ep
B) Hypotension

pr
C) Low-sodium diet
D) Excessive alcohol intake st
te
5. A patient is in the CCU with dilated cardiomyopathy and asymptomatic right-sided
chronic heart failure. What finding will the nurse most likely discover in her assessment
ng

of this patient?
si

A) Mitral regurgitation murmur


ur

B) Cheyne-Stokes respiratory pattern


C) Unilateral crackles
yn

D) Rales
.m

6. A nurse in the CCU must assess a cardiac patients fluid status. Which of the following is
the best method for him to use?
w

A) Having the patient measure and record all liquids taken in and all urine excreted
w

B) Weighing the patient daily


w

C) Pulse oximetry
D) Radionuclide ventriculography
7. A patient presents to the CCU with shortness of breath on exertion. Which diagnostic
study would be best for ruling out pneumonia or COPD as the cause of the patients
symptoms?
A) Echocardiography
B) Radionuclide ventriculography
C) Pulse oximetry
D) Chest radiography
8. A patient in the CCU with chronic heart failure experiences shortness of breath even
when at rest. When he stands up and walks across the room, his shortness of breath
worsens. Which class of heart failure does this patient exhibit, according to the New York
Heart Association (NYHA) Functional Classification of Heart Failure?
A) Class I

om
B) Class II
C) Class III

.c
D) Class IV

ep
9. A patient in the CCU has clear evidence of structural heart failure, as he lacks
contractility in his left ventricle and his ejection fraction is only 37%. However, he has

pr
never shown any signs or symptoms of heart failure. According to the American College
of Cardiology (ACC)/American Heart Association (AHA) Guidelines for Stages of Heart
st
Failure, which stage of heart failure best characterizes this patients condition?
te
A) Stage A
ng

B) Stage B
si

C) Stage C
ur

D) Stage D
10. A patient presents to the CCU with cardiac heart failure resulting from atrial tachycardia.
yn

Which of the following explains how atrial tachycardia can cause heart failure?
.m

A) Premature ventricular beats leading to sudden death


B) Increase in muscle mass in ventricle
w

C) Pulmonary embolus leading to acute right-sided heart failure


w

D) Shortened diastole leading to decreased filling and diastolic dysfunction


w

11. The nurse is caring for a patient who has been admitted with a diagnosis of heart failure.
What does the term heart failure mean?
A) It is characterized by rales and alveolar edema.
B) It results from damage caused by acute myocardial infarction.
C) It is a general clinical syndrome with many etiologies.
D) All patients have similar symptoms.
12. A patient has been diagnosed with left ventricular heart failure. What physical findings
would the nurse expect?
A) Enlarged liver
B) Peripheral edema
C) Pulmonary rales
D) Enlarged spleen
13. An elderly patient with uncontrolled hypertension and atrial fibrillation with rapid
ventricular response is admitted with a diagnosis of heart failure. He has crepitant

om
pulmonary rales and his chest x-ray shows pulmonary congestion. The patient probably
has what type of heart failure?

.c
A) Left ventricular systolic failure

ep
B) Left ventricular diastolic failure
C) Right ventricular failure

pr
D) Combination heart failure
st
14. A patient has been admitted with acute left systolic heart failure secondary to acute
te
myocardial infarction. The patient has dyspnea and orthopnea and a cardiac rhythm of
sinus tachycardia. The physiologic dysfunction for this type of heart failure is what?
ng

A) Impaired contractility of the left ventricle


si

B) Impaired compliance of the left ventricle


ur

C) Cardiac valve disease


yn

D) Acute myocardial infarction


15. A patient has been admitted with right heart failure. The physiologic basis of right heart
.m

failure is what?
w

A) Left heart failure


w

B) Impaired right cardiac output


w

C) Pulmonary hypertension
D) Acute pulmonary embolus
16. A patient has been diagnosed with acute left heart failure secondary to acute myocardial
infarction and increased afterload. What changes in assessment findings does the nurse
expect to find?
A) Elevated pulmonary capillary wedge pressure
B) Normal or low blood pressure
C) Enlarged liver and spleen
D) Lungs clear to auscultation
17. A patient with heart failure is being monitored with a pulmonary artery catheter to assess
cardiac output and its components. What pattern of results would indicate a need for
immediate intervention?
A) Heart rate, preload, contractility, and afterload are balanced and cardiac output is
normal.
B) Heart rate is rapid, preload is reduced, contractility and afterload are normal, and

om
cardiac output is reduced.
C) Heart rate is rapid, preload is high, contractility is high, afterload is low, and cardiac
output is normal.

.c
D) Heart rate is low, preload is high, contractility is high, afterload is low, and cardiac

ep
output is normal.

pr
18. A patient has been diagnosed with chronic heart failure, compensated. What symptoms
would the nurse expect to find?
A) st
Frothy sputum progressing to pink frothy sputum
te
B) Severe hypotension when supine
ng

C) Mild dyspnea on exertion or when supine


si

D) 4+ pitting edema of the lower extremities when dependent


ur

19. A patient with long-standing hypertension has a viral upper respiratory illness and is self-
medicating with over-the-counter medications. The patient complains of severe dyspnea
yn

with activity and has new-onset crepitant rales and pitting edema of the lower extremities.
In evaluating the patients medications, what medication should the nurse look for in
.m

particular?
A) Furosemide (Lasix)
w

B) Nonsteroidal anti-inflammatory medications (NSAIDs)


w

C) Hydrochlorothiazide diuretic (HCTZ)


w

D) Angiotensin-converting enzyme (ACE) inhibitor


20. A patient with heart failure is being discharged. What discharge instruction should the
nurse emphasize most?
A) Date and time of next medical appointment
B) Dietary alterations to reduce sodium intake
C) Structured exercise program
D) Maintaining weight within 1 to 2 pounds
21. For a patient with heart failure, maintenance of a steady weight is an important goal. The
nurse should teach the patient that which of the following is the most accurate method for
monitoring fluid volume changes?
A) Intake and output, with 24-hour totals and trends
B) Daily weight at the same time every day
C) Daily assessment of peripheral edema
D) Periodic assessment of serum electrolyte values

om
22. A patient with heart failure has been started on an ACE inhibitor. What lab value related
to the ACE inhibitor would concern the nurse?

.c
A) Elevated serum potassium

ep
B) Elevated CK-MB

pr
C) Diminished sodium
D) Increased prothrombin time
st
te
23. A patient receiving an ACE inhibitor has a blood pressure of 85/50 mm Hg. The patient is
asymptomatic otherwise. What is the best nursing action?
ng

A) Hold the medication because the blood pressure is too low.


si

B) Recheck the blood pressure in 1 hour.


ur

C) Administer the medication because the patient is asymptomatic.


yn

D) Reschedule time of medication administration to hour of sleep.


24. The patient is prescribed hydralazine and a nitrate for management of heart failure. The
.m

patient tells the nurse he has trouble taking medications that must be taken several times a
day. What intervention will be the most helpful to the patient?
w

A) Stress the importance of taking medications as prescribed.


w

B) Ask the physician to change his therapy to long-acting medications that can be taken
w

less often.
C) Ask the physician to change his therapy to other drug classes at the same
frequencies.
D) Inform the physician that the patient is not taking his prescribed medications.
25. A patient with heart failure is taking an ACE inhibitor and a diuretic as prescribed but
continues to be symptomatic. What patient behavior would explain lack of achievement
of therapeutic goals?
A) Exercises regularly (walking and water aerobics)
B) Diet of mostly canned food and soda pop
C) Does not drink any alcoholic beverages
D) Takes medications as prescribed
26. A patient with heart failure comes to the clinic for a routine monitoring visit. What
findings would indicate achievement of therapeutic goals?
A) Lungs have crepitant rales.

om
B) Complains of dyspnea on exertion
C) Has pitting edema of lower extremities

.c
D) Weight is within 2 pounds of ideal weight.

ep
Answer Key

pr
1. A, B, D
2. B, D, F
st
te
3. B
ng

4. A
si

5. A
ur

6. B
yn

7. D
.m

8. D
9. B
w

10. D
w

11. C
w

12. C
13. B
14. A
15. B
16. A
17. B
18. C
19. B
20. D
21. B
22. A
23. C
24. B

om
25. B
26. D

.c
ep
Chapter 21 Acute Myocardial Infarction

pr
1. A patient has been found to be at high risk for cardiovascular disease after a highly
sensitive C-reactive protein (hs-CRP) blood test indicated a value of 3.0 mg/dL. The
st
patient would like to know what this test measures. What would be the best answer for
the nurse to give?
te
A) Systemic inflammation related to atherosclerosis
ng

B) Cardiac output following myocardial infarction


si

C) Blood pressure related to congestive heart failure


ur

D) Lipid levels in connection with stress response


yn

2. A patient newly diagnosed with atherosclerosis wants to know what the plaque building
up in his arteries actually consists of. Which of the following should the nurse mention?
.m

Select all that apply.


A) Fatty substances
w

B) Osteocytes
w

C) Epithelial tissue
w

D) Cholesterol
E) Cellular waste products
F) Collagen and elastic fibers
3. A patient with atherosclerosis acknowledges that he is a smoker but does not understand
how this contributes to his atherosclerosis. Which response would be best for the nurse to
give him?
A) Smoking causes stress, which increases his lipid levels.
B) Smoking triggers in him a craving for high-cholesterol foods.
C) Smoking injures the inner layer of his arteries, facilitating plaque buildup.
D) Smoking causes vasoconstriction.
4. A young patient who seems perfectly healthy is displaying symptoms of angina pectoris.
Which of the following are possible underlying causes of the patients condition? Select
all that apply.
A) Atherosclerotic narrowing of the coronary arteries
B) Hemophilia

om
C) Use of aspirin
D) Spasm of a coronary artery

.c
E) Arterial inflammation

ep
F) Tachycardia

pr
5. A patient in the CCU complains of pain and a squeezing sensation in his chest. He says
st
that it typically affects him in the middle of the night, waking him from sleep. The nurse
recognizes that this patient is most likely experiencing which of the following?
te
A) Stable angina
ng

B) Unstable angina
si

C) Classic angina
ur

D) Variant angina
yn

6. A 78-year-old woman in the CCU complains of shortness of breath, along with prolonged
chest pain unrelieved by rest or sublingual nitroglycerin. The nurse recognizes that this
patient is most likely experiencing which condition?
.m

A) Stable angina pectoris


w

B) Atherosclerosis
w

C) Classic angina
w

D) Myocardial infarction
7. A patient in the CCU has severe angina pectoris and is undergoing ECG assessment. The
nurse knows that this patient had a previous myocardial infarction (MI) 10 years ago and
that this old MI is likely to show up on the ECG. Which of the following would show that
the patient had a previous MI but is not having one now?
A) Abnormal Q waves accompanied by ST-segment elevation
B) Abnormal Q waves accompanied by a normal ST segment
C) Normal Q waves accompanied by a ST-segment elevation
D) Normal Q waves accompanied by ST-segment depression
8. A patient is recovering in the CCU following a myocardial infarction 24 hours ago. She is
now pain free and is beginning to eat solid foods. To prevent the Valsalva maneuver from
occurring in this patient, what intervention should the nurse take?
A) Administer fibrinolytic therapy.
B) Administer a stool softener.
C) Administer sublingual nitroglycerin.

om
D) Administer aspirin.
9. A patient in the CCU is recovering from a myocardial infarction. He complains of new

.c
chest pain and trouble breathing. The nurse observes that he has cool, moist skin and is

ep
breathing rapidly. On taking his pulse, she finds that it is rapid and thready. She takes his
blood pressure and finds it to be 80/60 mm Hg. What complication is this patient most

pr
likely experiencing?
A) Recurrent myocardial ischemia
B) Ventricular septal wall rupture
st
te
C) Cardiogenic shock
ng

D) Pericarditis
si

10. A patient is being discharged from the CCU following a myocardial infarction (MI). He
ur

will be participating in cardiac rehabilitation following discharge. His wife is wondering


what she can do to help her husband during rehabilitation. What instructions should the
yn

nurse include in the discharge teaching plan? Select all that apply.
A) Take cardiopulmonary resuscitation training.
.m

B) Learn to interpret ECG readings.


w

C) Encourage her husband to stop smoking.


w

D) Exercise with her husband.


w

E) Prepare heart-healthy meals.


F) Perform a stress test on her husband monthly.
11. The nurse is teaching several patients how to reduce their risk for coronary artery disease
or atherosclerosis. What risk factors should the nurse most emphasize?
A) Serum cholesterol, high stress, and anger management
B) Male gender, increased age, and hypertension
C) Serum cholesterol, hypertension, and cigarette smoking
D) Cigarette smoking, smokeless tobacco, and ethnicity
12. The nurse is explaining the process of atherosclerosis to a patient and family. What
information should the nurse include?
A) This disease is purely genetic in nature and cannot be prevented.
B) The central process involves the formation of lipid-filled arterial plaques.
C) Alterations in lifestyle have no effect on the formation of plaques.

om
D) Anginal chest pain occurs when a coronary artery is 100% occluded.
13. A patient has been diagnosed with unstable angina. Unstable angina may be described as
what?

.c
A) Occurring with exercise

ep
B) Crescendo pattern

pr
C) Predictable pattern
D) Relieved by rest st
te
14. The patient is complaining of midsternal chest pain that feels like constant severe
pressure. The pain is not relieved by rest or three nitroglycerin tablets and is different
ng

than the pain the patient has had in the past. What is the priority nursing action?
si

A) Administer another nitroglycerin tablet.


ur

B) Obtain a 12-lead electrocardiogram.


yn

C) Use anxiety reduction techniques.


D) Teach risk reduction strategies.
.m

15. The patient is experiencing severe chest pain after a stressful incident at work. On the 12-
lead ECG, the nurse notices ST segment depression in the anterior leads that resolves as
w

the patients pain resolves. What is the most appropriate nursing action?
w

A) Send the patient home, since the symptoms have resolved.


w

B) Implement a risk reduction teaching plan.


C) Schedule the patient for immediate further diagnostic tests.
D) Refer the patient for psychological evaluation and treatment.
16. A patient with stable angina is being treated with a beta-blocker. What assessment finding
would most cause the nurse to question the use of this medication?
A) Heart rate 60 at rest, denies dizziness when standing
B) Systolic blood pressure 82, complains of chronic fatigue
C) Sinus rhythm with rare premature atrial complexes (PACs)
D) Diastolic blood pressure 80 with normal pulse pressure
17. A patient with anginal chest pain is given a low-dose chewable aspirin tablet. In
explaining the rationale for this medication to the patient and family, what information
does the nurse include?
A) Aspirin will help control the pain.
B) Reduction of low-grade fever is important.

om
C) Anticoagulation effects will reduce clot formation.
D) Aspirin is less toxic to the liver than Tylenol.

.c
18. The patient has been diagnosed with an acute anterior myocardial infarction. What

ep
complication does the nurse most anticipate?

pr
A) Second-degree AV block, Mobitz 1
B) Few or no complications
st
te
C) Cardiac failure or cardiogenic shock
D) Intractable nausea and vomiting
ng

19. The patient has been diagnosed with an acute inferior myocardial infarction. What 12-
si

lead ECG changes does the nurse expect to find after several hours?
ur

A) ST segment elevation and large Q wave in leads II, III and aVF
yn

B) ST segment depression and wide QRS in leads II, III, and aVR
C) ST segment elevation and large Q wave in leads V1V4
.m

D) ST segment depression and large S wave in leads V2V4


w

20. The patient is being evaluated for acute myocardial infarction. Elevation in what
w

laboratory value would confirm an acute MI?


w

A) Troponin I or T
B) CK-MB or CK-MM
C) Myoglobin after 12 hours
D) Leukocyte count
21. A patient with an acute myocardial infarction has received three nitroglycerin tablets,
oxygen, and aspirin and is still complaining of severe crushing chest pain. What is the
best nursing action?
A) Give 5 mg intravenous morphine.
B) Assess for drug-seeking behavior.
C) Give intravenous benzodiazepine.
D) Use anxiety reduction measures.
22. As part of treatment for acute myocardial infarction, a patient is receiving an infusion of
tissue plasminogen activator (tPA). Two hours after the initiation of the infusion, the
patient has a short run of accelerated idioventricular rhythm. During the arrhythmia, the
patients blood pressure is 110/78 but he denies any other change in symptoms. What is

om
the best nursing action?
A) Discontinue the tPA.

.c
B) Initiate an intravenous lidocaine drip.

ep
C) Assess for pulmonary adventitious sounds.
D) Continue close observation of the patient.

pr
23. A patient with an acute myocardial infarction has been started on daily enalapril
st
(Vasotec), an ACE inhibitor, to preserve ejection fraction. What is the most important
nursing assessment before giving this medication?
te
A) Intake and output
ng

B) Daily weight
si

C) Blood pressure
ur

D) Pulse oximetry
yn

24. After an acute myocardial infarction, the patient is receiving tissue plasminogen activator
(tPA) and initially on nitroglycerin for chest pain at 10 on 0/10 scale. The patient has rare
.m

premature ventricular contractions (PVCs) and a blood pressure of 82/55 mm Hg. What is
the most important nursing action?
w

A) Discontinue the nitroglycerin infusion rate.


w

B) Increase the tPA infusion rate.


w

C) Administer a bolus of amioderone.


D) Obtain a 12-lead electrocardiogram.
25. The patient is 24 hours post acute myocardial infarction and may have developed a
ventricular septum rupture. What nursing assessment would best indicate this
complication?
A) Loud holosystolic murmur
B) Dyspnea and basal crackles
C) Sinus tachycardia
D) Pain unrelieved by nitroglycerin
26. A patient who has had a myocardial infarction is started on a cardiac rehabilitation
program. The patient asks the nurse why he must participate in this program. What is the
most important information for the nurse to include in answering the patients question?
A) This program has been ordered by the physician and is required.
B) Participation has been shown to decrease the risk of subsequent coronary events.
C) The program includes monitored exercise and risk reduction counseling.

om
D) Participation will improve the patients quality of life and emotional stability.

.c
Answer Key

ep
1. A

pr
2. A, D, E, F
3. C st
te
4. A, D, E, F
ng

5. D
si

6. D
ur

7. B
8. B
yn

9. C
.m

10. A, C, D, E
w

11. C
w

12. B
w

13. B
14. B
15. C
16. B
17. C
18. C
19. A
20. A
21. A
22. D
23. C
24. A
25. A

om
26. B

.c
Chapter 22 Cardiac Surgery

ep
1. A patient is recovering in the CCU following carotid endarterectomy. What intervention

pr
should the nurse make to ensure that a hematoma is not forming in the patients neck?
A) st
Assess neck size by comparing operative side with nonoperative side.
te
B) Monitor pupil reactivity.
ng

C) Assess hand grip.


D) Monitor blood pressure.
si

2. A patient who recently underwent carotid endarterectomy is exhibiting signs of stroke.


ur

Which intervention or interventions should the nurse take to assess this patients
neurological function? Select all that apply.
yn

A) Measure chest tube output.


.m

B) Assess eye movement.


w

C) Monitor level of consciousness.


w

D) Assess urine output.


w

3. A patient with left main coronary artery disease (CAD) experiences persistent angina. She
would like to exercise more, but is limited by shortness of breath and angina. Her
physician believes that she is a good candidate for coronary artery bypass graft. The nurse
recognizes that which of the following are indications for coronary artery bypass graft
(CABG) surgery in this situation?
A) Having left main CAD
B) Being a good candidate for angioplasty and stenting
C) Having persistent angina
D) Availability of new effective CAD medication
E) Limited exercise tolerance
F) The patients spouse was successfully treated with CABG
4. A patient in the CCU is recovering from coronary artery bypass graft (CABG) surgery.
He has had multiple graft surgeries in the past. For this latest surgery, the patients radial
artery was used in the graft. What complication should the nurse most expect to observe
in this patient?
A) Occlusion

om
B) Infection
C) Arterial spasm

.c
D) Internal hemorrhaging

ep
5. A patient is recovering in the CCU following off-pump coronary artery bypass graft
(OPCABG) surgery involving the internal mammary artery graft. What nursing

pr
intervention is most important for the first 48 hours following surgery for this patient?
A) Administration of anticoagulant therapy st
te
B) Administration of calcium channel blockers
ng

C) Monitoring of fluid status


D) Assessment for hematoma
si

6. A patient with moderate mitral stenosis with minimal calcification and regurgitation is
ur

preparing to have surgery. Which procedure would be most appropriate to restore normal
function to this patient?
yn

A) Annuloplasty
.m

B) Valve replacement with biological valve


w

C) Valve replacement with caged ball valve


w

D) Commissurotomy
w

7. A patient recovering from cardiopulmonary bypass surgery is shivering. For what reason
should the nurse be concerned about the shivering?
A) Shivering is a sign of cardiogenic shock
B) Shivering is a sign that the patient has a fever
C) Shivering increases myocardial workload
D) Shivering can cause sutures to rupture
8. A patient recovering in the CCU following coronary artery bypass graft (CABG) surgery
complains of pain in his chest. The patient underwent a sternotomy incision during the
surgery. Which of the following types of pain would indicate anginawhich may indicate
graft failureas opposed to typical pain resulting from the sternotomy?
A) Radiates to arms
B) Is worse with deep breathing
C) Is worse with movement
D) Is sharp

om
9. A patient in the CCU is experiencing premature atrial contractions following coronary
artery bypass graft surgery. Which of the following would be the most appropriate
nursing intervention?

.c
A) Administration of anticoagulants

ep
B) Administration of potassium and magnesium

pr
C) Administration of dopamine
D) Administration of epinephrine st
te
10. A patient who underwent carotid endarterectomy is being discharged from the CCU.
Which of the following instructions should the nurse give to the patient?
ng

A) Avoid rotating your head.


si

B) Bruising and discoloration of the neck are not normal and should be reported
ur

immediately.
C) Eat a low-fat diet.
yn

D) Avoid washing the incision site.


.m

11. A patient with severe coronary artery disease is scheduled for coronary artery bypass
graft surgery. As part of the preoperative teaching, the nurse explains the surgery. Which
w

of the following statements about this procedure is true?


w

A) The diseased artery will be removed and replaced with a graft from another artery.
w

B) A piece of the saphenous vein will be used to go around the diseased part of the
artery.
C) After removal of the diseased artery, the remaining ends will be anastomosed.
D) The wall of the heart will be incised to create a new pathway for blood flow.
12. The patient is scheduled for coronary artery bypass surgery using the off-pump technique.
During preoperative teaching, the nurse explains that using the off-pump procedure has
what advantage over the on-pump procedure?
A) There is a lower risk of a cerebral embolus.
B) The patient can anticipate a shorter hospital stay.
C) There will be less need for anticoagulation therapy.
D) The procedure will be less painful.
13. A patient with severe coronary artery disease has persistent angina that is refractory to
medical management at maximum drug doses and has severe compromise of activities of
daily living from the angina. The patient has had several coronary artery bypass surgeries
and has been told that he is not a candidate for any further surgeries or percutaneous

om
interventions such as stents. In discussing options for further therapy, what should the
nurse include that would offer the patient the most hope?
A) Unless a new medication is invented, there is nothing that can be done.

.c
B) Discussion of hospice and palliative support for end-of-life care

ep
C) Referral to the social worker for financial assistance

pr
D) Referral to the transmyocardial laser revascularization program for evaluation
st
14. The patient has been diagnosed with severe mitral valve stenosis. What physical changes
would the nurse expect to find as a result of the stenosis?
te
A) Prolonged capillary refill
ng

B) Normal left atrial and ventricular pressures


si

C) Clear lung sounds


ur

D) Angina pectoris
yn

15. The patient has been diagnosed with mitral valve insufficiency and left ventricular
hypertrophy. What effect would the nurse expect from the left ventricular hypertrophy?
.m

A) Improved cardiac output from increased left ventricular contractility


w

B) No appreciable signs or symptoms or effects until late in the disease process


w

C) A more obvious and easier-to-auscultate mitral valve regurgitant murmur


w

D) Early onset of pulmonary edema and right-sided congestive heart failure


16. The patient has been diagnosed with severe aortic valve stenosis. Considering the most
common symptoms caused by aortic valve stenosis, what is the most important nursing
intervention?
A) Document characteristics of the aortic stenosis murmur.
B) Teach patient to rise slowly from a supine position.
C) Assess peripheral circulation more frequently.
D) Assess for and document pulmonary adventitious sounds.
17. The patient has developed acute aortic valve insufficiency after experiencing blunt chest
trauma. What symptom, if found by the nurse, is indicative of a counterproductive
compensatory mechanism that should be treated?
A) Low cardiac output
B) Pulmonary edema
C) Elevated blood pressure
D) Aortic insufficiency murmur

om
18. The patient has had coronary artery bypass surgery involving the cardiopulmonary bypass
pump, systemic hypothermia, topical cardiac hypothermia, and cold cardioplegia. As a

.c
result of the various hypothermic therapies, numerous postoperative complications may
ensue. What collaborative postoperative intervention is specifically directed at

ep
ameliorating one or more of these complications?

pr
A) Use of intravenous pain and sedation medications
B) Mechanical ventilation and supplemental oxygen therapy
C)
st
Vital signs every hour until stable or transferred to step-down unit
te
D) Management of mediastinal chest tube drainage
ng

19. The nurse is caring for a patient who has just had coronary artery bypass grafting. As part
si

of the admission procedure to the critical care unit immediately after surgery, what
nursing assessment has the highest priority?
ur

A) Urine output
yn

B) Cardiac index measurement


.m

C) Chest tube drainage measurement


D) Core body temperature
w

20. After coronary artery bypass surgery, the patient experiences significant fluid volume
w

shifts and losses. What nursing assessment would be most indicative of fluid volume
w

deficit?
A) Low central venous pressure
B) Urine output 40 mL/hr
C) Brisk capillary refill
D) Diminished core body temperature
21. A patient who has just has coronary artery bypass surgery has developed tachycardia, a
low-grade fever, and an elevated total white blood cell count. What additional sign or
symptom would support the nurses suspicion of a postoperative infection?
A) Purulent drainage from the chest incision
B) Chest incision edges are red and swollen.
C) Elevated immature neutrophils or bands
D) Severe incisional pain with cough
22. The nurse is caring for a patient who has just had coronary artery bypass grafting and is

om
experiencing significant hypotension. What nursing assessment would best confirm that
the hypotension is related to blood loss?

.c
A) Low hemoglobin and hematocrit, with high central venous pressure

ep
B) Chest tube drainage in excess of 200 mL/hr
C) Urine output 40 to 50 mL/hr

pr
D) Chest tube drainage less than 30 mL/hr
st
te
Answer Key
ng

1. A
si

2. B, C
ur

3. A, C, E
yn

4. C
5. A
.m

6. D
w

7. C
w

8. A
w

9. B
10. C
11. B
12. B
13. D
14. A
15. C
16. B
17. C
18. B
19. B
20. A
21. A

om
22. B

.c
Chapter 23 Anatomy and Physiology of the Respiratory System

ep
1. A patient in the ICU experiences difficulty breathing in during exercise following trauma
to his sternum. Which muscle involved in respiration was likely injured in this patient,

pr
leading to this condition?
A) Diaphragm st
te
B) Sternocleidomastoid
ng

C) External intercostal
D) Scalene
si

2. The nurse is explaining to a young patient with asthma how air is conducted from his
ur

nose through his airways to the alveoli in his lungs. Which of the following represents the
correct order of airways?
yn

A) Nasopharynx, oropharynx, trachea, bronchi, bronchioles


.m

B) Trachea, nasopharynx, oropharynx, bronchi, bronchioles


w

C) Trachea, bronchi, bronchioles, nasopharynx, oropharynx


w

D) Oropharynx, nasopharynx, trachea, bronchioles, bronchi


w

3. The nurse is explaining to a patient in the ICU who has experienced a pneumothorax the
role of the alveoli in respiration. The nurse should mention that which of the following
are present in the alveoli? Select all that apply.
A) Macrophages
B) Surfactant
C) Epiglottis
D) Bronchus
4. A premature infant is experiencing infant respiratory distress syndrome in the NICU. The
nurse recognizes that this condition is most likely caused by which of the following?
A) Defective type I alveolar cells
B) An obstruction at the right bronchus
C) Negative intrapleural pressure
D) Immature type II alveolar cells
5. A patient is experiencing poor perfusion in his right thorax as a result of suspected
partially occluded arteries. Occlusion in which artery or arteries would cause this? Select

om
all that apply.
A) Aorta

.c
B) Internal mammary

ep
C) Subclavian

pr
D) Intercostal

st
6. A patient in the ICU with chronic obstructive pulmonary disease has mucous secretions
throughout her respiratory system. Her breathing is highly labored. What is the primary
te
cause of her labored breathing?
ng

A) Decreased airway resistance due to increased airway diameter


B) Excessive surfactant
si

C) Increased airway resistance due to decreased airway diameter


ur

D) Increased lung compliance


yn

7. A patient with pulmonary edema is experiencing respiratory distress. The nurse


recognizes that which factor affecting gas exchange is most likely responsible for the
.m

patients dyspnea?
w

A) Surface area available for diffusion


w

B) Thickness of the alveolar-capillary membrane


w

C) Partial pressure of alveolar gas


D) Solubility and molecular weight of the gas
8. A patient is recovering from severe altitude sickness after a failed attempt to climb Mount
Everest. Which factor affecting gas exchange was most likely responsible for the patients
illness?
A) Surface area available for diffusion
B) Thickness of the alveolar-capillary membrane
C) Partial pressure of alveolar gas
D) Solubility and molecular weight of the gas
9. A patient in the ICU with pneumothorax is experiencing a ventilation-perfusion
imbalance. Which type of imbalance is the patient most likely experiencing?
A) Physiological shunt
B) Anatomical shunt
C) Alveolar dead space

om
D) Silent unit
10. A patient in the ICU experiences an asthma attack after a guest brings her a flower
arrangement. Stimulation of which receptors is most associated with the patients asthma

.c
attack?

ep
A) Stretch receptors

pr
B) Peripheral chemoreceptors
C) Irritant receptors
st
te
D) Juxtacapillary receptors
11. The patient has experienced a mediastinal shift to the left. Several structures or functions
ng

will be compromised by this condition. What is the most significant effect?


si

A) Right lung will be compressed.


ur

B) Cardiac output will be diminished.


yn

C) Point of maximal impulse will be displaced.


D) Jugular venous distention will occur.
.m

12. The patient has experienced a mechanical obstruction of the lower trachea from a mucous
plug. What is the priority nursing action?
w
w

A) Obtain arterial blood gas results.


w

B) Deep suction the patient.


C) Turn onto left side and elevate head.
D) Increase tidal volume on ventilator.
13. The patient has experienced a toxic inhalation injury that has significantly damaged his
type II alveolar cells. The nurse should expect to find which of the following?
A) Impaired gas exchange and atelectasis
B) Bronchospasm and wheezing
C) Laryngospasm and stridor
D) Eupnea and bradycardia
14. The patient has experienced an acute embolus in the thorax. An embolus in which artery
is most likely to significantly compromise gas exchange?
A) Left main bronchial artery
B) Right peripheral bronchial arteriole
C) Left peripheral pulmonary arteriole

om
D) Right main pulmonary artery
15. The patient has suffered a stab wound to the chest, interrupting the integrity of the pleura
with loss of negative pleural pressure and interruption of the contact between the visceral

.c
and parietal pleural surfaces. What is the most significant effect of this trauma?

ep
A) Pleuritic chest pain

pr
B) Displaced diaphragm
C) Crepitus over ribs
st
te
D) Compromised gas exchange
16. The patient is recovering from acute pulmonary disease. The nurse is comparing his
ng

current assessment findings with older ones. What currently increased assessment finding
would indicate achievement of therapeutic goals?
si

A) Pulmonary compliance
ur

B) Effort of breathing
yn

C) Pressure support
.m

D) Airway resistance
17. The patient is exhibiting poor gas exchange. Four factors influence gas exchange across
w

the alveolar membrane. What condition related to the four factors could be interfering
w

with gas exchange?


w

A) Oxygen at 100%
B) 35% of the alveoli are filled with fluid.
C) Oxygen delivered under pressure
D) Alveolar membranes are 0.3 microns thick.
18. The patient is experiencing a ventilationperfusion mismatch and compromised gas
exchange. Correcting ventilationperfusion mismatch entails what actions?
A) Decreasing anatomic dead space with intubation
B) Decreasing alveolar dead space with positive-pressure ventilation
C) Increasing alveolar perfusion with platelet therapy
D) Increasing apex ventilation with high tidal volumes
19. The patient is in severe metabolic acidosis from diabetic ketoacidosis. Based on the
oxyhemoglobin dissociation curve, what change in the plan of care does the nurse expect?
A) Reduction of inhaled oxygen as hemoglobin is fully saturated
B) Use of positive-pressure ventilation to improve alveolar expansion

om
C) Increase in inhaled oxygen as hemoglobin saturation is low
D) Increase in ventilation rate to help excrete carbon dioxide

.c
20. After a drug overdose, a patient has very slow and shallow respirations. What stimulation
of chemoreceptors or lung receptors will result in an increase in respiratory rate?

ep
A) Decreased carbon dioxide levels at central chemoreceptors

pr
B) PaO2 less than 60 at peripheral chemoreceptors
C) st
Normal airway resistance perceived by stretch receptors
te
D) Lack of stimulation of irritant receptors
ng

Answer Key
si
ur

1. B
2. A
yn

3. A, B
.m

4. D
w

5. B, C, D
w

6. C
w

7. B
8. C
9. D
10. C
11. B
12. B
13. A
14. D
15. D
16. A
17. B
18. B
19. C

om
20. B

.c
Chapter 24 Patient Assessment: Respiratory System

ep
1. A patient in the ICU is producing an increased amount of sputum and has a fever. The
nurse suspects an infection. Which of the following characteristics of the patients sputum

pr
would indicate a bacterial infection, as opposed to a viral infection? Select all that apply.
A) Yellow st
te
B) Clear
ng

C) Blood-streaked
D) Green
si

E) Brown
ur

F) White
yn

2. An older patient in the ICU has labored breathing and a history of smoking. The nurse
suspects that he has chronic obstructive pulmonary disease (COPD). Which of the
.m

following signs would tend to confirm that the patient has COPD? Select all that apply.
w

A) Increased anterior-posterior diameter of the chest


w

B) The patient lying propped onto one side


w

C) Deviation of the trachea from the midline


D) Inspiration taking just as long as expiration
3. A patient in the ICU with a history of smoking is having tests performed to assess his
respiratory function. Which test would be best for measuring the oxygenation of this
patients blood?
A) SaO2
B) Pulse oximetry
C) End-tidal carbon dioxide
D) pH
4. A patient has arterial blood gas testing performed. Her PaO2 is 95 mm Hg, SaO2 is 90%,
pH is 7.4, and HCO3 is 23 mEq/L. Which of these values should the nurse be most
concerned about?
A) PaO2
B) HCO3
C) pH

om
D) SaO2
5. A patient in her first trimester is in the ICU and is displaying signs of metabolic alkalosis.

.c
On speaking with the patient, the nurse learns that she has been experiencing much

ep
vomiting, diarrhea, and anxiety in the past week. The nurse should recognize that which
of the following is the most likely cause of this acid-base imbalance?

pr
A) Pregnancy
B) Diarrhea st
te
C) Vomiting
ng

D) Anxiety
6. A patient has arterial blood gas testing performed. His PaO2 is 83 mm Hg, SaO2 is 91%,
si

pH is 7.5, PaCO2 is 24 mm Hg, and HCO3 is 22 mEq/L. Which of the following indicates
ur

this patients condition?


A) Respiratory acidosis with normal saturation (uncompensated)
yn

B) Metabolic alkalosis with low saturation (uncompensated)


.m

C) Respiratory alkalosis with low saturation (uncompensated)


w

D) Metabolic alkalosis with low saturation (fully compensated)


w

7. A patient in the ICU has just undergone arterial blood gas testing. Her results are as
follows: PaO2 is 90 mm Hg, pH is 7.43, PaCO2 is 24, and HCO3 is 19 mEq/L. Which of
w

the following indicates this patients condition?


A) Respiratory alkalosis, fully compensated
B) Respiratory acidosis, partially compensated
C) Metabolic alkalosis, fully compensated
D) Metabolic acidosis, partially compensated
8. A patient in the ICU with pneumonia and on mechanical ventilation is suspected to have
pulmonary embolus. Which diagnostic study would be best for assessing for this
condition?
A) Chest radiography
B) Ventilation-perfusion scanning
C) Pulmonary angiography
D) Bronchoscopy
9. A patient in the ICU is suspected of having tuberculosis. The nurse understands that

om
which diagnostic test will most likely be performed on this patient to confirm this
suspicion?

.c
A) Sputum culture

ep
B) Thoracentesis
C) Ventilation-perfusion scanning

pr
D) Pulmonary angiography
st
10. The nurse is completing a history on a patient with dyspnea. What characteristic of
te
dyspnea is most indicative of pulmonary disease?
ng

A) Most severe when patient is supine


B) Awakens patient from sleep at night
si

C) Accompanied by anginal chest pain


ur

D) Most severe with exertion


yn

11. During the inspection phase of the physical examination, the nurse notices that the
patients chest wall is round, and the anterior-to-posterior diameter and the lateral diameter
.m

are equal. Based on this information, what disease process does the nurse suspect?
w

A) Lobular pneumonia
w

B) Chronic obstructive pulmonary disease


w

C) Acute respiratory distress syndrome


D) Respiratory syncytial virus
12. The nurse is caring for a patient who has undergone thoracic surgery. What assessment
information would best support the nurses suspicion of pulmonary consolidation?
A) Vesicular lung sounds at bases
B) Bronchial breath sounds over upper airway
C) Egophony at left posterior base of lung
D) Basal crackles that clear with cough
13. A patient is being monitored with continuous pulse oximetry. Under what circumstance
would the nurse question the accuracy of the pulse oximetry reading?
A) The patient is a victim of a fire in an enclosed space.
B) Cardiac monitor pattern shows normal sinus rhythm.
C) Extremities are warm and dry with intact pulses.
D) Respiratory rate and pulse rate are elevated.

om
14. The patient is being monitored with an end-tidal CO2 monitor and has values trending
upward. What acidbase abnormality should the nurse assess for?
A) Metabolic acidosis

.c
B) Metabolic alkalosis

ep
C) Respiratory acidosis

pr
D) Respiratory alkalosis
st
15. The patient has experienced a significant drop in hemoglobin levels and is slightly
te
tachycardic. The pulse oximetry value is 100% and arterial blood gas values are normal.
What is the most important adverse physiologic effect that the nurse would expect?
ng

A) Polycythemia
si

B) Diminished blood pressure


ur

C) Hyperalertness and hyperreflexia


yn

D) Diminished tissue oxygenation


16. The nurse is evaluating the following arterial blood gas values: pH 7.35, PaO2 95 mm Hg,
.m

SaO295%, PaCO2 40 mm Hg, and HCO3 24 mEq/L. How does the nurse interpret these
results?
w

A) Normal
w

B) Respiratory acidosis
w

C) Metabolic acidosis
D) Technical error
17. The patient is experiencing respiratory acidosis. What nursing action is most likely to
alleviate this condition?
A) Suction the endotracheal tube.
B) Reduce the respiratory rate on the ventilator.
C) Administer intravenous bicarbonate.
D) Increase the rate of crystalloid intravenous fluids.
18. A patient has experienced a cardiopulmonary arrest and is receiving cardiopulmonary
resuscitation. As the nurse evaluates the effectiveness of this therapy, what value on
arterial blood gases is most indicative of hypoventilation?
A) Diminished PaO2
B) Diminished SaO2
C) Elevated HCO3

om
D) Elevated PaCO2
19. A critically ill patient has arterial blood gas results of pH 7.6, PaCO2 40 mm Hg, and

.c
HCO3 30 mEq/L. With what medical situation do these results most clearly correlate?

ep
A) Excess nasogastric drainage

pr
B) Severe diarrhea
C) Diabetic ketoacidosis
st
te
D) Lobular pneumonia
20. A patient has arterial blood gas results of pH 7.2, PaCO2 55 mm Hg, and HCO3 24
ng

mEq/L. How does the nurse interpret these results?


si

A) Metabolic acidosis
ur

B) Metabolic alkalosis
yn

C) Respiratory acidosis
D) Respiratory alkalosis
.m

21. A critically ill patient has arterial blood gas results of PaO2 60 mm Hg, SaO2 80%, pH
7.35, PaCO2 35 mm Hg, and HCO3 24 mEq/L. How does the nurse interpret these
w

results?
w

A) Hypoxemia and respiratory acidosis


w

B) Hypoxemia and normal acidbase balance


C) Normal oxygenation and metabolic acidosis
D) Normal oxygenation and acidbase balance
22. A critically ill patient has arterial blood gas results of pH 7.35, PaCO2 55 mm Hg, and
HCO3 28 mEq/L. How does the nurse interpret these results?
A) Respiratory acidosis
B) Metabolic alkalosis
C) Partially compensated metabolic alkalosis
D) Fully compensated respiratory acidosis
23. A patient who is critically ill is attached to a saturation of mixed venous oxygen monitor
(SvO2) and has an SvO2 value that is trending downward and is currently below normal at
55%. What clinical abnormality should the nurse suspect?
A) Increased cardiac output
B) Fever and shivering

om
C) Fluid volume overload
D) Oversedation

.c
24. The nurse is evaluating the chest radiograph of a critically ill patient and notices that the

ep
patients trachea has shifted to the left of midline. What additional finding would confirm
the presence of a pneumothorax?

pr
A) Absent or diminished breath sounds on the right
B)
st
Lung fields generally white on chest radiograph
te
C) Lung fields dull to percussion on the right
ng

D) Blunting of costophrenic angles on chest radiograph


si

Answer Key
ur
yn

1. A, D, E
2. A, B
.m

3. A
w

4. D
w

5. C
w

6. C
7. A
8. C
9. A
10. D
11. B
12. C
13. A
14. C
15. D
16. A
17. A
18. D

om
19. A
20. C

.c
21. B

ep
22. D

pr
23. B
24. A st
te
Chapter 25 Patient Management: Respiratory System
ng

1. A patient with Duchennes muscular dystrophy requires an intermittent short-term therapy


si

to maintain alveolar ventilation. The patient is not a candidate for aggressive mechanical
ventilation as provided through an artificial airway. Which of the following would be the
ur

most appropriate treatment for this patient?


yn

A) Manual resuscitator
B) Negative-pressure ventilator
.m

C) Volume ventilator
w

D) Pressure ventilator
w

2. A patient in the ICU is on a volume ventilator. The nurse recognizes that which of the
w

following are true for this type of ventilator? Select all that apply.
A) Peak inspiratory pressure varies from breath to breath and must be monitored
closely.
B) Amount of pressure depends on patients lung compliance and patient-ventilator
resistance factors.
C) The device fits like a tortoise shell, forming a seal over the chest.
D) Exhaled tidal volume must be monitored closely.
E) Volume varies based on changes in resistance or compliance.
F) Small tidal volumes are used at frequencies greater than 100 breaths/minute.
3. A patient has recently arrived in the ICU following cardiac arterial bypass graft surgery
and has not yet emerged from anesthesia. He requires full ventilatory support. Which
ventilation mode should the nurse use for this patient?
A) Pressure support ventilation mode
B) Synchronized intermittent mandatory ventilation mode
C) Pressure-controlled ventilation mode

om
D) Assist-control mode
4. A patient with acute respiratory distress syndrome (ARDS) requires mechanical

.c
ventilation but is at risk for barotrauma due to decreased lung compliance. Which mode

ep
of ventilation should the nurse use with this patient?
A) Pressure support ventilation mode

pr
B) Synchronized intermittent mandatory ventilation mode
C) Pressure-controlled ventilation mode
st
te
D) Assist-control mode
ng

5. A patient with acute respiratory distress syndrome (ARDS) has been receiving ventilation
in assist-control mode for the past 48 hours. The fraction of inspired oxygen (FiO2) is set
si

to 70% and tidal volume is set to 15 mL/kg of body weight. What complication or
ur

complications should the nurse be most concerned about in this situation? Select all that
apply.
yn

A) Barotrauma
.m

B) Oxygen toxicity
C) Volutrauma
w

D) Hypoxemia
w

6. A patient with acute respiratory distress syndrome (ARDS) is receiving pressure-


w

controlled ventilation with positive end-expiratory pressure (PEEP) of 10 cm H2O. Which


of the following nursing interventions will help protect this patient from one of the
complications of PEEP?
A) Administration of antihypertensive medication
B) Increased administration of IV fluids
C) BiPAP ventilation via full facemask
D) Increase tidal volume to 20 mL/kg of body weight
7. A patient with chronic obstructive pulmonary disease has just been started on pressure-
controlled ventilation with positive end-expiratory pressure (PEEP) of 18 cm H2O. The
fraction of inspired oxygen is set to 50%. Which of the following is a sign of tension
pneumothorax that the nurse should watch for?
A) An abrupt decrease in peak inspiratory pressure
B) Tracheal deviation from the midline
C) Syncope
D) Bradypnea

om
8. A patient in the ICU has been on assist-control ventilation via endotracheal intubation for
24 hours but will need to be shifted to a long-term ventilation method. Which of the
following would be the most appropriate treatment for this patient?

.c
A) Iron lung

ep
B) Manual resuscitator

pr
C) Tracheostomy tube
D) st
Pressure-controlled ventilation via endotracheal intubation
te
9. A patient in the CCU is on a ventilator with oral intubation. The nurse notices that a
pressure sore is forming on the patients lip where it comes into contact with the
ng

endotracheal tube. Which nursing action would be most appropriate?


si

A) Reposition the endotracheal tube from one side to the other


ur

B) Retape the endotracheal tube


C) Use twill tape
yn

D) Place an oral bite block in the patients mouth


.m

10. A patient in the ICU failed long-term ventilation weaning via CPAP trials. What is the
next intervention that should take place?
w

A) The T-piece trial should be performed.


w

B) The patient should be switched to SIMV method.


w

C) The patient should be rested on the ventilator for 24 hours.


D) The patient should be switched to the SIMV plus PSV method.
11. The patient is scheduled for a lobectomy. As part of preoperative teaching, the nurse
stresses the importance of coughing and deep breathing postoperatively. What positive
outcome from this procedure does the nurse expect?
A) Thicker and less mobile secretions
B) Lower incidence of atelectasis
C) Increased risk for pneumonia
D) Increased need for pain medication
12. The patient has consolidation of his right lateral lower lung segments and is receiving
chest physiotherapy. What position is best for draining this portion of the lung?
A) Semi-Fowlers
B) Supine

om
C) Left side-lying
D) Right side-lying

.c
13. The patient is receiving supplemental oxygen therapy. What finding most clearly
demonstrates achievement of one of the goals of oxygen therapy?

ep
A) Therapy is discontinued after 3 days.

pr
B) Increased respiratory rate and depth
C) Verbalization of relief of dyspnea st
te
D) Reduction of arterial carbon dioxide pressure
ng

14. The patient is receiving supplemental oxygen therapy. What finding would be most
indicative of a life-threatening complication of oxygen therapy?
si

A) Increased somnolence
ur

B) Eupneic respirations
yn

C) SaO2 100%
.m

D) Skin irritation under mask


15. The nurse is assisting with endotracheal intubation. In what circumstance would the nurse
w

question the use of nasotracheal intubation?


w

A) Confused patient
w

B) Fractured mandible
C) Acute sinusitis
D) Possible cervical spine fracture
16. After pulmonary surgery, the patient returns to the critical care unit with a left pleural
chest tube in place connected to a disposable three-chamber unit that is connected to
suction at negative 20 cm H2O. What must the nurse ensure so that the goal for chest tube
placement is met?
A) The water-seal chamber has the amount of water defined by the manufacturer.
B) The suction control chamber has vigorous bubbling at all times.
C) The drainage unit is prepared using sterile water only.
D) Suction is applied between negative 10 and 20 cm H2O.
17. A critically ill patient has an intravenous line inserted into the left subclavian vein. After
the procedure, what nursing intervention is used to detect the most serious complication
of this procedure?
A) Connection of ordered intravenous fluids

om
B) Compatibility evaluation for fluids infused
C) Use of a semipermeable plastic site dressing

.c
D) Auscultation of bilateral breath sounds

ep
18. The patient is to undergo chest tube placement. What is the best nursing intervention to

pr
prevent a complication of this procedure?
A) Facilitate chest tube removal on day 3.
st
te
B) Use the supine position during chest tube removal.
C) Premedicate with an intravenous opioid.
ng

D) Keep drainage tubing off the bed.


si

19. A patient with asthma is receiving a bronchodilator medication. If this therapy is helpful
ur

to the patient, the nurse would expect to find which of the following?
yn

A) Increased productiveness of cough


B) More intense wheezing
.m

C) Persistent tachycardia
w

D) Reduced peak expiratory flow rate


w

20. A patient being supported with endotracheal intubation and mechanical ventilation is
w

increasingly agitated. What is the most appropriate nursing intervention?


A) Administer neuromuscular blockade medication.
B) Administer a benzodiazepine.
C) Obtain arterial blood gas measurement.
D) Ask a family member to stay with the patient.
21. A patient in severe congestive heart failure is at risk for the development of acute
respiratory failure and is receiving supplemental oxygen therapy. What nursing
assessment parameter is most indicative of acute respiratory failure?
A) Dependent pitting edema that is worsening
B) New onset of systolic gallop
C) Conversion to atrial fibrillation
D) Arterial PaO2 45 mm Hg
22. The patient is being well supported with a positive-pressure mechanical ventilator.

om
Because of the mechanism of action of this type of ventilator, what common complication
does the nurse watch for?

.c
A) Diminished cardiac output

ep
B) Increased somnolence
C) Deep venous thrombosis

pr
D) Reduced patient control
st
23. The patient is being supported with a positive-pressure mechanical ventilator. The
te
machine in use is a volume-cycled ventilator. Relative to the functioning of this
ventilator, what nursing assessment is most important?
ng

A) Continuous pulse oximetry


si

B) Vital signs per unit protocol


ur

C) Respiratory rate
yn

D) Peak inspiratory pressure


24. The patient is being supported with a positive-pressure mechanical ventilator set to a
.m

synchronized intermittent mandatory ventilation (SIMV) rate of 8 breaths per minute.


What situation, if found by the nurse, would indicate a ventilator malfunction?
w

A) SIMV rate 8, patient rate 30, total rate 38 breaths per minute
w

B) SIMV rate 6, patient rate 2, total rate 8 breaths per minute


w

C) SIMV rate 8, patient rate 0, total rate 8 breaths per minute


D) SIMV rate 8, patient rate 8, total rate 16 breaths per minute
25. The patient is being supported with mechanical ventilation and is requiring very high
levels of inspired oxygen to maintain his arterial blood gases at acceptable levels. In an
attempt to reduce the amount of oxygen required, positive end-expiratory pressure
(PEEP) at 5 cm H2O is added to the ventilator settings. What is the most important effect
of PEEP?
A) Increases number of open alveoli
B) Increases patient comfort
C) Assists in ventilator weaning
D) Compensates for tubing resistance
26. The patient is being supported by mechanical ventilation and is not maintaining adequate
oxygenation on current settings. Positive end-expiratory pressure (PEEP) has just been
increased from 10 to 12 cm H2O. What nursing assessment finding best indicates that the
patient is not tolerating this change?

om
A) Increased cardiac output
B) Increased blood pressure

.c
C) Significant hypotension

ep
D) Increased pulmonary compliance

pr
27. A patient with multiple trauma and several other comorbidities is being supported with
endotracheal intubation and mechanical ventilation and is scheduled for a tracheostomy.
st
The family questions the appropriateness of this procedure. In explaining the rationale for
the procedure to the family, what is the most important nursing point?
te
A) There is a physicians order for the procedure.
ng

B) Unit protocol mandates tracheostomy after a patient has been intubated for 7 days.
si

C) A tracheostomy will greatly increase the patients comfort.


ur

D) Tracheostomies at 3 to 7 days result in fewer ventilated days.


yn

28. As part of the care of a mechanically ventilated patient, the nurse provides oral and
subglottic suctioning every 2 hours. What is the best rationale for this nursing action?
.m

A) Stimulates cough and deep breathing


w

B) Reduces pulmonary microbial colonization


w

C) Maintains oral mucosal moisture


w

D) Part of Universal Precautions protocols


29. The patient has been on a mechanical ventilator for 2 weeks. Weaning from mechanical
ventilation is to start today. Based on the length of time that the patient has been
ventilated, what information should the nurse emphasize to the patient and the family?
A) Extubation is expected later today, as the patient is relatively young.
B) Delays and setbacks are expected before independence is achieved.
C) The best method is continuous positive airway pressure (CPAP).
D) Elevation of the head of the bed will provide for the most patient comfort.

Answer Key

1. B
2. A, B
3. D
4. C

om
5. A, B, C
6. B

.c
7. B

ep
8. C

pr
9. A
10. C st
te
11. B
ng

12. C
13. C
si

14. A
ur

15. C
yn

16. A
.m

17. D
18. C
w
w

19. A
w

20. C
21. D
22. A
23. D
24. B
25. A
26. C
27. D
28. B
29. B

Chapter 26 Common Respiratory Disorders

1. A patient in the ICU presents with a fever, headache, malaise, and myalgia. The nurse
suspects that the patient has a SARS infection. Which other finding or findings would
tend to confirm this suspicion? Select all that apply.

om
A) Mucus-producing cough
B) Dyspnea

.c
C) Positive for human cytomegalovirus

ep
D) Lymphopenia

pr
2. A patient in the ICU with congestive heart failure demonstrates shortness of breath and
complains of pleuritic chest pain. Which of the following conditions, associated with
st
heart failure, should the nurse most suspect in this patient?
te
A) Transudative pleural effusion
ng

B) Exudative pleural effusion


C) Acute asthma
si

D) Emphysema
ur

3. It has been determined that a patient in the ICU has pleural effusion. Which intervention
yn

is needed at this point to distinguish between transudative and exudative pleural effusion?
.m

A) Chest x-ray
B) CT scan
w

C) Ultrasound
w

D) Assessment of pleural fluid


w

4. A tall, thin patient in his mid-twenties presents to the ICU with dyspnea, pleuritic chest
pain, and a heart rate of 120 bpm. A chest radiograph shows a contralateral mediastinal
shift. Which condition does this patient most likely have?
A) Primary spontaneous pneumothorax
B) Secondary spontaneous pneumothorax
C) Traumatic pneumothorax
D) Transudative pleural effusion
5. A young man with spontaneous primary pneumothorax is given supplemental oxygen.
What is the primary purpose for giving this patient supplemental oxygen?
A) Oxygen reverses the formation of excess pleural fluid.
B) Oxygen accelerates the rate of air resorption from the pleural space.
C) Oxygen counteracts the hypoxemia associated with pneumothorax.
D) Oxygen calms the patient.
6. A patient on renal dialysis in the ICU begins to develop worsening dyspnea, pleuritic
chest pain, cough, apprehension, and pain. Moreover, the nurse observes that his right leg

om
is swelling significantly. Which diagnostic study would be most appropriate for
confirming a diagnosis of pulmonary embolus?

.c
A) Computed tomography (CT) angiogram

ep
B) Thoracentesis
C) Ventilation-perfusion scan

pr
D) Arterial blood gas test
st
7. An elderly woman in the ICU has chronic obstructive pulmonary disease (COPD), with
te
FEV1 of 20%, and is worried about developing a serious illness that would cause
complications with her COPD. Which of the following should the nurse recommend to
ng

the patient that could reduce her risk for serious illness or death by about 50%?
si

A) Influenza vaccine
ur

B) Lung transplantation
C) Heparin
yn

D) Lung volume reduction surgery


.m

8. The nurse is working with a 50-year-old patient with emphysema who smokes and eats
poorly. Which of the following should she mention to the patient regarding her disease?
w

Select all that apply.


w

A) The disease may be reversible if the irritant causing inflammation can be identified.
w

B) Lung volume reduction surgery may increase her respiratory function.


C) Smoking cessation can slow the progress of COPD.
D) Improving her nutritional state could increase her respiratory muscle strength.
9. A patient in the ICU is experiencing an asthma attack. The room temperature is
approximately 72F. The patient has recently had symptoms of a cold. Which of the
following is the most likely trigger for the womans asthma attack?
A) Upper respiratory infection
B) Latex allergy
C) Room temperature
D) Pollen
10. A patient in the ICU with acute respiratory failure demonstrates dyspnea, headache,
tachypnea, and tachycardia. Which of these symptoms distinguishes this patients
condition as acute hypercapnic respiratory failure, as opposed to acute hypoxemic
respiratory failure?
A) Dyspnea

om
B) Headache
C) Tachycardia

.c
D) Tachypnea

ep
11. An elderly resident of a long-term care facility is admitted to the CCU with acute
pneumonia. What might be a factor increasing the risk of pneumonia for this patient?

pr
A) Takes a histamine-2 antagonist medication daily
B)
st
Has never smoked or lived with anyone who smoked
te
C) Usually has a good appetite without swallowing difficulty
ng

D) Has his teeth cleaned every 6 months and brushes often


si

12. An elderly patient with community-acquired pneumonia has been admitted to the CCU.
What nursing action will improve the patients chances of survival?
ur

A) Obtain blood and sputum cultures on admission and 2 days after admission.
yn

B) Initiate intravenous antibiotic therapy within 8 hours after admission.


.m

C) Administer intravenous antibiotics for 7 full days.


D) Initiate antibiotics only after culture results are known.
w
w

13. What nursing strategy implemented after a patient is admitted to the CCU will decrease
the risk that he or she will develop hospital-acquired pneumonia?
w

A) Verify that patients received a pneumococcal pneumonia vaccine before admission.


B) Ensure that all members of the health care team follow hand hygiene protocols
strictly.
C) For patients who cannot eat normally, maintain NPO status.
D) Maintain bed rest in supine position for all mechanically intubated patients.
14. A patient has been admitted to the CCU in respiratory failure from probable SARS
pneumonia. Knowing that SARS is very contagious, in addition to ensuring strict hand
hygiene by everyone, what is the most important nursing action?
A) Provide adequate nutrition.
B) Obtain blood and sputum cultures.
C) Place patient in full isolation.
D) Ensure adequate fluid balance.
15. A patient has been admitted with severe dyspnea, orthopnea, and pleuritic chest pain. A

om
chest x-ray shows a pleural effusion. In planning care for this patient, what action is most
appropriate?

.c
A) Thoracentesis for all patients

ep
B) Diuretics and positive inotropes
C) Low-fat, high-carbohydrate diet

pr
D) Oxygen therapy assessed by arterial blood gases
st
16. A patient being mechanically ventilated with positive end-expiratory pressure and
te
pressure support totaling 30 mm Hg has developed unequal chest expansion, absent
breath sounds on the right, and tracheal deviation to the right. The patient is increasingly
ng

tachycardic, anxious, and agitated, and his pulmonary compliance is rapidly decreasing.
What is the most appropriate nursing action?
si

A) Obtain a chest x-ray to rule out pneumothorax.


ur

B) Inititate needle thoracotomy or chest tube insertion.


yn

C) Sedate the patient and evaluate ventilator settings.


.m

D) Obtain arterial blood gases to evaluate gas exchange.


17. Which of the following patients would be at highest risk for the development of deep vein
w

thrombosis progressing to pulmonary embolus?


w

A) Age 45, exercises daily, underwent elective knee repair surgery, high normal
w

platelets
B) Age 85, sedentary, somewhat dehydrated, underwent repair of a pelvic fracture, high
normal platelets
C) Age 26, athletic, underwent repair of a football injury, normal platelets
D) Age 50, on aspirin and beta blockers, underwent open heart surgery, borderline low
platelets
18. A patient is being evaluated for development of chronic obstructive pulmonary disease.
Which symptom is most likely to indicate this disease process?
A) Clubbing of fingernails
B) Chronically elevated carbon dioxide level
C) Development of barrel chest
D) Impaired expiratory air volume
19. A patient with chronic obstructive pulmonary disease is receiving supplemental oxygen at
2 L/min by nasal cannula. What should the nurse particularly watch for in this patient?
A) Somnolence, cherry color of mucous membranes, bradypnea

om
B) Tachypnea, dyspnea, pleuritic chest pain with cough
C) Increased frequency of cough and elevated temperature

.c
D) Change in quality and color of sputum and general malaise

ep
20. What symptom or factor would the nurse find in emphysema but not in chronic
bronchitis?

pr
A) Cough, thick sputum
B) Dyspnea with exertion st
te
C) History of smoking
ng

D) Hyperinflation of lungs
21. A patient has severe wheezing, air hunger, cyanosis, and anxiety and is given an inhaled
si

bronchodilator. If the bronchodilator is effective, the nurse will find what direct effect
ur

after administration?
yn

A) Increased respiratory rate


B) Increased heart rate
.m

C) Unchanged cough frequency


w

D) Reduction of audible wheeze


w

22. A patient has been admitted to the CCU in severe distress with acute respiratory failure.
w

Initial arterial blood gases are pH 7.33, PaCO2 65, HCO3 30, PaO2 65, SaO2 90. What is
the most important nursing action?
A) Administer oxygen at 100%.
B) Prepare for intubation.
C) Obtain arterial blood gases.
D) Measure functional expiratory volume.
Answer Key

1. B, D
2. A
3. D
4. A
5. B

om
6. C
7. A

.c
8. B, C, D

ep
9. A
10. B

pr
11. A
12. B
st
te
13. B
ng

14. C
si

15. D
ur

16. B
yn

17. B
18. D
.m

19. A
w

20. D
w

21. D
w

22. B

Chapter 27 Acute Respiratory Distress Syndrome

1. A patient with worsening tachypnea and dyspnea has a PaO2/FiO2 ratio of 190, bilateral
infiltrates on chest x-ray, pulmonary artery occlusive pressure (PAOP) of 15 mm Hg, and
no indication of left atrial hypertension. Which of these findings indicates that the patient
has acute respiratory distress syndrome (ARDS), as opposed to just acute lung injury
(ALI)?
A) PaO2/FiO2 ratio of 190
B) Bilateral infiltrates on chest x-ray
C) PAOP of 15 mm Hg
D) No indication of left atrial hypertension
2. A patient in the ICU was admitted about 24 hours ago with symptoms of acute respiratory
distress syndrome (ARDS). He now demonstrates shortness of breath and a respiratory
rate of 40 breaths per minute. His heart rate is 115 beats per minute. The nurse notes a
bluish tinge in his fingertips. His chest x-ray shows patchy alveolar infiltrate. His body

om
temperature is normal. Which stage of ARDS is this patient most likely in?
A) Stage 1

.c
B) Stage 2

ep
C) Stage 3
D) Stage 4

pr
3. A patient in the ICU with acute respiratory distress syndrome (ARDS) complains of
st
worsening dyspnea. Which of the following physiological changes associated with ARDS
are most responsible for this patients impaired ventilation? Select all that apply.
te
A) Decreased lung compliance
ng

B) Pulmonary hypertension
si

C) Vasoconstriction of the pulmonary vascular bed


ur

D) Increased airway resistance


yn

E) Cyanosis
F) Multisystem organ dysfunction
.m

4. A patient with acute respiratory distress syndrome (ARDS) in the ICU is receiving high-
w

frequency oscillation ventilation (HFOV). Which complication related to this intervention


should the nurse be looking for?
w

A) Volutrauma
w

B) Pneumothorax
C) Cyanosis
D) Trapping of air in the alveoli
5. Why must patients receiving high-frequency oscillation ventilation (HFOV) be sedated
and paralyzed?
A) HFOV tends to cause muscle spasms.
B) HFOV is traumatic for many patients.
C) HFOV is painful.
D) With HFOV, oscillation will cease with any change in airway pressure.
6. A patient with acute respiratory distress syndrome (ARDS) in the ICU demonstrates
diminished cardiac output, resulting in reduced oxygen delivery. Which of the following
interventions will increase cardiac output in this patient by increasing contractility?
A) Administration of norepinephrine
B) Administration of dobutamine

om
C) Administration of diuretics
D) Hemoglobin transfusion

.c
7. A patient with acute respiratory distress syndrome (ARDS) has shown no improvement

ep
after 8 hours of mechanical ventilation set to maximize fraction of inspired oxygen. The
patient is developing pulmonary hypertension. Which intervention would be best for

pr
reducing the negative effects of the pulmonary hypertension in this patient via selective
pulmonary vasodilation?
A) Nitric oxide
st
te
B) Prophylactic antibiotic therapy
ng

C) Corticosteroids
si

D) Neuromuscular blocking agents


ur

8. A patient in the ICU with acute respiratory distress syndrome is at increased risk for
ventilator-associated pneumonia (VAP). Which of the following interventions should the
yn

nurse implement for this patient? Select all that apply.


.m

A) Elevate the head of the bed 30 to 45 degrees.


B) Weaning protocol
w

C) Antibiotic therapy
w

D) Deep vein thrombosis (DVT) prophylaxis


w

E) Steroid administration
F) Peptic ulcer prophylaxis
9. An elderly patient in the ICU with acute respiratory distress syndrome (ARDS) has
developed sepsis. Which of the following interventions should the nurse implement to
address the sepsis and its symptoms? Select all that apply.
A) Antibiotic therapy
B) Elevating head of the bed 30 to 45 degrees
C) Weaning protocol
D) Daily sedation withholding
E) Steroid administration
F) Activated protein C
10. A patient with acute respiratory distress syndrome (ARDS) demonstrates unmistakable
signs of pneumothorax. What is the next intervention that the nurse should implement?

om
A) Increase the peak end-expiratory pressure (PEEP).
B) Administer an inotropic agent.

.c
C) Elevate the head of the bed to 35 degrees.

ep
D) Assist with a chest tube insertion.
11. A patient is admitted after an acute lung injury from smoke inhalation. If the patient

pr
develops acute respiratory distress syndrome (ARDS), what is the first symptom the
nurse will find?
st
te
A) Patchy infiltrate on chest x-ray
B) Pulmonary capillary wedge pressure less than 18
ng

C) Increasing tachypnea and air hunger


si

D) Low plasma brain natriuretic peptide (BNP)


ur

12. A patient has experienced an acute lung injury. What factors in the patients history will
yn

probably increase his risk for developing adult respiratory distress syndrome (ARDS)?
A) Smokes two packs a day
.m

B) Drinks one glass of wine on holidays


w

C) Has a sedentary lifestyle


w

D) Has no known chronic diseases


w

13. A patient with acute respiratory distress syndrome (ARDS) has severe hypoxia refractory
to high levels of oxygen and mechanical ventilation. What is the underlying
pathophysiology most responsible for the hypoxia?
A) Increased capillary permeability from mediator release
B) Increased patchy infiltrate visible on chest x-ray
C) Reduced pulmonary afterload effects
D) Reduced pulmonary preload effects
14. The nurse is caring for a patient who has developed acute respiratory distress syndrome
(ARDS) after smoke inhalation. Based on the pathologic changes in ARDS, the nurse
expects what outcome during the first several hours?
A) Improvement in airway patency with bronchodilator therapy
B) Persistent and worsening hypoxia despite mechanical ventilation
C) Evidence of hypercarbic respiratory failure with compensation
D) General fluid volume deficit from capillary permeability increase
15. A patient with acute respiratory distress syndrome (ARDS) has these arterial blood gas

om
results: pH 7.33, PaCO2 50, HCO3 26, PaO2 80, SaO2 80. The patient is mechanically
ventilated with 60% oxygen and positive-end expiratory pressure (PEEP) at 5 cm H2O.
These arterial blood gas results, when compared to the previous day, show no

.c
improvement in oxygenation and increased PaCO2. When the nurse is assessing these

ep
results, what is most significant in evaluating the patients progress?
A) Patient isnt responding to conventional ventilation therapy.

pr
B) Development of metabolic alkalosis is compensatory.
C) st
Inhaled oxygen and PEEP should be increased.
te
D) Change in acidbase balance is ominous.
ng

16. A patient has acute respiratory distress syndrome (ARDS) and is mechanically ventilated.
The patient has not responded to high levels of oxygen and low positive-end expiratory
si

pressure (PEEP), and the ventilator settings are being adjusted. What patient response or
ventilator setting limit should the nurse look for to prevent ventilator-associated lung
ur

injury (VALI)?
yn

A) Positive end-expiratory pressure at least 30 cm H2O


.m

B) Fraction inspired oxygen set at 1.0 (100%)


C) Measured airway plateau pressure below 30 cm H2O
w

D) Respiratory rate between 12 and 20 breaths per minute


w

17. The patient has been diagnosed with acute lung injury after submersion in contaminated
w

ground water and probable aspiration. The patient has developed tachycardia, fever, and
increased white blood cell count. What does the nurse anticipate will be added to the plan
of care?
A) Prophylactic antibiotics
B) Nonsteroidal anti-inflammatory drugs (NSAIDs)
C) Obtaining blood, sputum, urine cultures
D) Intravenous controlled sedation
18. A patient with acute respiratory distress syndrome (ARDS) is receiving an inhaled
bronchodilator. What nursing assessment indicates the most important expected outcome
of the bronchodilator for this patient?
A) Decreased audible wheezing
B) Slight tachycardia during treatment
C) Increase in mucus suctioned
D) Reduction of peak airway pressure
19. Infection and sepsis are significant complications of ARDS. Which nursing action has the

om
highest priority in preventing infection and sepsis?
A) Maintain plateau airway pressure below 30.

.c
B) Infuse enteral nutrition with minimal interruptions.

ep
C) Keep patient sedated to facilitate ventilation.

pr
D) Place in prone position every 2 to 4 hours.

st
20. A patient with acute respiratory distress syndrome (ARDS) is being cared for in a CCU.
The nurse implements the ventilator bundle of care. Which of the following statements
te
about this action is true?
ng

A) Bundles of care strategies combine several strategies that have been shown to
improve patient outcomes.
si

B) A ventilator bundle is a group of ventilator equipment ordered together from


ur

respiratory care.
C) Protocols for care are called bundles in critical care to differentiate them from
yn

routine care.
.m

D) Each bundle is a defined set of ventilator settings to be used for particular disease
processes.
w

21. A patient with acute respiratory distress syndrome (ARDS) is receiving mechanical
w

ventilation. The patients high airway pressure alarm sounds, and the nurse finds
asymmetrical chest expansion and absent breath sounds on the right. What is the probable
w

cause of this situation?


A) Oversedation
B) Need for suctioning
C) Impending respiratory arrest
D) Acute pneumothorax
22. The nurse is caring for a patient who is being mechanically ventilated. What strategies
have been shown to prevent ventilator-acquired pneumonia (VAP)? Select all that apply.
A) Routine prone positioning
B) Elevate head of bed 30 degrees if not contraindicated
C) Oral care with plaque removal every 2 to 4 hours
D) Balanced nutrition by gastric tube
E) Continuous subglottic suctioning of secretions
F) Frequent turning and chest physiotherapy

om
Answer Key

1. A

.c
2. B

ep
3. A, D

pr
4. D
5. D st
te
6. B
ng

7. A
8. A, B, D, F
si

9. A, E, F
ur

10. D
yn

11. C
.m

12. A
13. A
w
w

14. B
w

15. D
16. C
17. C
18. D
19. B
20. A
21. D
22. B, C, E, F

Chapter 28 Anatomy and Physiology of the Renal System

1. A patient has been admitted to the ICU following a car wreck in which she suffered blunt,
penetrating trauma. She has lost quite a bit of blood and is in shock. Which of the
following conditions should the nurse expect in this patient? Select all that apply.
A) Hypotension
B) Vasodilation of afferent arterioles

om
C) Increased glomerular filtration rate
D) Anuria

.c
E) Homeostasis

ep
F) Effective autoregulation

pr
2. A child in the ICU has been diagnosed with Bartter syndrome, a rare genetic defect in the
thick ascending limb of the loop of Henle that impairs the function of this structure.
st
Which characteristic should the nurse expect in this patient, based on the function of the
thick ascending limb of the loop of Henle?
te
A) Excessive loss of sodium in urine
ng

B) Overly diluted urine


si

C) Impaired reabsorption of water


ur

D) Decreased concentration of chloride ions in urine


yn

3. The nurse is working with a patient with a juxtaglomerular cell tumor, which accelerates
the function of these cells. What effects should the nurse expect to see with this
.m

condition? Select all that apply.


A) Increased renin production
w

B) Decreased angiotensin I levels


w
w

C) Constriction of smooth muscle surrounding arterioles


D) Hypertension
E) Decreased glomerular filtration rate
F) Decreased urine production
4. A patient has a severe deficit of sodium and potassium. The nurse should expect impaired
reabsorption of which substance in this patient?
A) Glucose
B) Water
C) Urea
D) Chloride
5. A patient is currently taking an aldosterone blocker, which reduces reabsorption of
sodium. What other condition should the nurse expect to see as a result of the patient
taking this medication?
A) Hyperkalemia
B) Increased hydrogen level

om
C) Hypernatremia
D) Edema

.c
6. An athlete has been admitted to the ICU for heat stroke and severe dehydration after

ep
participating in a marathon on a hot day. Which of the following would the nurse expect
to find in this patient?

pr
A) Decreased serum osmolality
B) Increased renin secretion
st
te
C) Increased glomerular filtration rate
ng

D) Hypertension
si

7. The nurse observes that an elderly patients urine samples have become increasingly dark
yellow. Which of the following would best explain this phenomenon?
ur

A) The patient is drinking too much water.


yn

B) The patient is diabetic.


.m

C) The patient is hypertensive.


D) The patient is dehydrated.
w
w

8. A patient with severe anorexia is admitted to the ICU. Which of the following would the
nurse expect to find in this patient? Select all that apply.
w

A) Hypoproteinemia
B) Decreased glomerular filtration rate
C) Lower filtrate osmotic pressure
D) Hyperglycemia
9. A patient with acute renal failure is in the ICU. Which of the following would best help
the nurse assess the health of the patients renal tubules?
A) Sodium concentration of urine
B) pH
C) Clearance of inulin
D) Urine output
10. A patient in the ICU is experiencing a flare-up of arthritis. A lack of fluid in which
subcompartment of the body would contribute most directly to this condition?
A) Transcellular

om
B) Interstitial
C) Plasma

.c
D) Intracellular

ep
11. A patient is found to have low blood levels of magnesium. What effect should the nurse
expect this to have in the patient?

pr
A) Impaired active transport via the sodiumpotassium pump
B) Alkalosis st
te
C) Stimulation of parathyroid hormone release
ng

D) Osmotic regulation
12. A patient is experiencing a sudden drop in blood pressure with a mean arterial pressure
si

ranging between 78 and 82 mm Hg. What effect on the functioning of the renal system
ur

does the nurse expect?


yn

A) Dilation of afferent renal arterioles


B) Increase in glomerular filtration rate
.m

C) Dilation of efferent renal arterioles


w

D) Reduction of glomerular filtration rate


w

13. A patient has a condition resulting in malfunction of the proximal tubules of nephrons.
w

What would the nurse expect to find?


A) All urine values within normal limits
B) Low levels of electrolytes in urine
C) Reduction of drug clearance rates
D) Decrease in urine urea levels
14. A patient has a reduced urine output. What situation will reduce glomerular filtration rate
and hence urine output?
A) Increase in mean arterial pressure to 100 mm Hg
B) Decrease in hydrostatic pressure in Bowmans capsule
C) Decrease in plasma protein concentrations
D) Condition resulting in afferent arteriole constriction
15. The patient has a reduced clearance rate for a drug usually secreted by the nephrons. The
nurse should assess for malfunction in which of the following?
A) Osmosis in Bowmans capsule

om
B) Active transport in distal tubules
C) Filtration rate in glomerulus

.c
D) Rate of flow in collecting duct

ep
16. The patient has a low serum sodium level and a high serum potassium level. In normal
compensation, what body function would increase?

pr
A) Proximal tubule reabsorption of sodium
B) Conversion of angiotensin II to renin st
te
C) Secretion of aldosterone
ng

D) Response to diuretic therapy


17. The patient is experiencing a fluid volume overload from excess intravenous fluids.
si

Assuming that normal kidney function exists, what hormone actions would the nurse
ur

expect?
yn

A) Hypothalamic secretion of antidiuretic hormone


B) Adrenal secretion of renin
.m

C) Reduction of renin and aldosterone functions


w

D) Increased conversion of angiotensin I to angiotensin II


w

18. The patient has a low serum sodium level. Based on the major role of sodium in the body,
w

the nurse would look for alterations in which of the following?


A) Fluid balance
B) Calcium balance
C) Energy levels
D) Pulmonary function
19. The patient has an abnormally high level of serum potassium. What associated life-
threatening alteration should the nurse look for?
A) Gastrointestinal distress
B) Cardiac dysrhythmias
C) Muscle spasm
D) Increased aldosterone
20. The patient is experiencing metabolic acidosis. What kidney function is impaired?
A) Hydrogen ion filtration by the glomerulus
B) Electrochemical bicarbonate reabsorption

om
C) Hydrogen ion secretion by the distal tubules
D) Sodium reabsorption by bicarbonate

.c
ep
Answer Key

pr
1. A, B, D
2. A st
te
3. A, C, D
ng

4. A
5. A
si

6. B
ur

7. D
yn

8. A, C
.m

9. A
10. A
w
w

11. A
w

12. D
13. C
14. D
15. B
16. C
17. C
18. A
19. B
20. C

Chapter 29 Patient Assessment: Renal System

1. The nurse is working with a patient with suspected acute renal failure. Which of the
assessments would provide accurate information about this patients fluid balance? Select
all that apply.
A) Daily measure of patients weight at the same time, using same scale

om
B) Monitoring of intake and output every 1 to 2 hours
C) Monitoring of blood sugar levels hourly

.c
D) Taking x-rays of the kidneys weekly

ep
2. A nurse is assessing a patient with acute renal failure for pitting edema. She gently
presses her finger into the edematous area on the patients foot. She measures an indention

pr
of 8 mm, which slowly returns to baseline after about 3 minutes. Which level of pitting
edema does this patient exhibit?
A) 4+ (severe)
st
te
B) 3+ (moderate)
ng

C) 2+ (mild)
si

D) 1+ (trace)
ur

3. A nurse has just received a clean-catch urine sample from the patient and is about to
yn

deliver it to the laboratory for analysis. She observes that it is dark yellow, has a cloudy
or turbid appearance, and has an ammonia odor. Which of the following does the nurse
suspect about the patients condition based on her observations? Select all that apply.
.m

A) The patient is diabetic.


w

B) The patient has above-normal creatinine clearance.


w

C) The patient is dehydrated.


w

D) The patient has proteinuria.


E) The patient has intrarenal hemorrhaging.
F) The patient has a renal infection.
4. On auscultating the patients heart, the nurse hears a third heart sound. What condition
would this finding be most indicative of?
A) Renal artery stenosis
B) A faulty stethoscope
C) Hypotension
D) Fluid overload
5. A nurse is examining a patient on dialysis. Which of the following should she especially
look for in this patient?
A) The presence of a third heart sound
B) Pitting edema

om
C) Inflammation or infection near the access site
D) Crackles on auscultation of lungs

.c
6. A patient in the ICU is found to have a steady glomerular filtration rate (GFR) of 180
L/day. Over the first 3 days following admission, his urine volume averages 1 L per day.

ep
On the fourth day, however, his urine volume drops to almost zero, while his GFR
remains constant. The nurse recognizes that which of the following is the most likely

pr
explanation of this phenomenon?
A) Chronic kidney disease st
te
B) Abdominal compartment syndrome
ng

C) Renal artery stenosis


D) Pulmonary edema
si

7. An elderly woman in the ICU has an indwelling urinary catheter and is being treated for a
ur

myocardial infarction. She is diabetic but is in the habit of having several glasses of
orange juice each day. Her lab test results show that her urinary pH is 7.6. The nurse
yn

should suspect which of the following factors as the cause of the elevated pH level?
.m

A) A urinary tract infection


B) Ketoacidosis caused by untreated diabetes
w

C) Consumption of acidic foods


w

D) Dehydration
w

8. A patients lab results include a finding of glucose in the urine (glycosuria). What other
lab result should the nurse expect in this patient?
A) Decreased ketones in the urine
B) Alkaline urinary pH
C) Elevated blood glucose
D) Decreased urine volume
9. A patient in the ICU with chronic renal disease has just received his lab results. The nurse
knows that one of the first functions to be lost in renal disease is the ability to concentrate
urine. Which of the following lab results would indicate this loss of ability to concentrate
the urine?
A) A specific gravity of urine that varies between 1.001 and 1.022, depending on the
patients water balance
B) Urine osmolality fixed within 150 mOsm of the simultaneously determined serum
osmolality
C) Urinary pH of 6.0

om
D) Urine volume of 1 L/day
10. A patient in the ICU is severely dehydrated. Which of the following findings would the

.c
nurse expect in this patient?

ep
A) Orthostatic hypotension on standing

pr
B) Third heart sound
C) Pitting edema
D) Rales
st
te
11. The patient is being evaluated for renal disease. During the initial patient history, what
ng

system should the nurse focus on in addition to the renal system?


si

A) Pulmonary and cardiac


ur

B) Neurologic and integumentary


C) Psychological and social support
yn

D) All physical and other systems


.m

12. A patient with chronic kidney disease has a low body temperature. What is the best
nursing response to this finding?
w

A) Assess for signs of sepsis.


w

B) Provide extra blankets.


w

C) Give warm intravenous fluids.


D) Reassess with a different thermometer.
13. During a routine physical examination, the nurse palpates the patients left kidney just
below the costal margins on the posterior aspect of the trunk. What is the most
appropriate nursing action?
A) Document this normal finding.
B) Obtain an order for renal diagnostics.
C) Assess for toxic ingestion.
D) Evaluate coagulation studies.
14. During physical examination of an ambulatory patient, the nurse finds elevated blood
pressure, pulmonary crackles that do not clear with cough, jugular venous distention, liver
congestion and enlargement, an S3 heart sound, and pitting edema of the feet and lower
legs. What is the most likely cause of this group of findings?
A) Fluid volume deficit

om
B) Fluid volume excess
C) Hyponatremia

.c
D) Leukocytosis

ep
15. A patient has had an arteriovenous graft inserted for dialysis. What nursing physical
examination finding indicates patency of the graft?

pr
A) Auscultation of a bruit over the renal artery
B) Palpation of a thrill over the graft
st
te
C) Distal pulses are 3+ to palpation
ng

D) Blood pressure within normal limits


si

16. A patient with renal impairment is found to have a positive Chvosteks sign. What
additional information should the nurse assess for?
ur

A) Hyperkalemia
yn

B) Hyponatremia
.m

C) Hypocalcemia
D) Hypermagnesemia
w
w

17. The patients urinalysis shows proteinuria. What is the significance of this finding?
w

A) Normal in late pregnancy


B) Intrarenal acute renal failure
C) Diminished glomerular filtration rate
D) Compromised erythropoietin production
18. The nurse is caring for a patient who was run over by an automobile. The patient has
hematuria on admission to the emergency department. What is the most likely cause of
the hematuria?
A) Kidney trauma
B) Kidney stones
C) Prostatic disease
D) Toxic damage
19. What laboratory value is most likely to indicate renal failure?
A) Elevated blood urea nitrogen (BUN)
B) Low hemoglobin and hematocrit

om
C) Elevated serum creatinine
D) Normal urine osmolarity

.c
20. A patient has significantly decreased creatinine clearance and an elevated serum

ep
creatinine. What does this value indicate to the nurse?
A) Improved renal function

pr
B) Kidney damage from disease
C) Muscle wasting
st
te
D) Rhabdomyolysis
ng

21. After a renal biopsy, what symptom indicates a significant complication of this
procedure?
si

A) Increasing hematuria
ur

B) Increased urine volume


yn

C) Heart rate 60 to 100


.m

D) Mean arterial pressure 85


w

22. The nurse is caring for a patient with renal disease and is monitoring fluid balance. What
is the most accurate method for assessing fluid balance?
w

A) Daily weights at same time of day


w

B) Episodic intake and output totals


C) Heart rate trends
D) Jugular venous volume measurement

Answer Key

1. A, B
2. A
3. C, F
4. D
5. C
6. B
7. A
8. C

om
9. B
10. A

.c
11. D

ep
12. B

pr
13. B
14. B st
te
15. B
ng

16. C
17. B
si

18. A
ur

19. C
yn

20. B
.m

21. A
22. A
w
w

Chapter 30 Patient Management: Renal System


w

1. A patient has just had a dual-lumen venous catheter inserted in his subclavian vein for
hemodialysis for acute renal failure. What intervention or interventions should the nurse
make in working with this patient? Select all that apply.
A) Verify central line catheter placement radiographically before use.
B) Inject all medications directly into the catheter.
C) Leave the catheter unclamped to prevent clotting.
D) Maintain sterile technique in handling vascular access.
E) Observe catheter exit site for signs of inflammation.
2. A patient with chronic renal failure has an arteriovenous fistula in her forearm for dialysis
access. What intervention or interventions should the nurse make in working with this
patient? Select all that apply.
A) Take the patients blood pressure on the forearm containing the fistula.
B) Palpate the fistula for thrill every 8 hours.
C) Draw blood samples from the vein that forms the fistula.
D) Avoid placing any restraints on the access arm.

om
E) Check access patency less frequently in hypotensive patients.
F) Occlude the fistula vein using firm pressure in the event of postdialysis bleeding

.c
from the needle site.

ep
3. An elderly patient in the ICU with chronic renal failure has just undergone surgery for a
synthetic arteriovenous graft in her left forearm for dialysis access. The nurse recognizes

pr
that the most likely reason this patient received a graft instead of a fistula is which of the
following?
A)
st
Thrombosis is less likely to occur with grafts than with fistulas.
te
B) Fistulas are more prone to infection than are grafts.
ng

C) The patients own blood vessels were not adequate for fistula formation.
si

D) An aneurysm is more likely to occur in a fistula than in a graft.


ur

4. A patient in the ICU is scheduled to begin peritoneal dialysis for acute renal failure. The
patient tells the nurse that he understands hemodialysis but is not familiar with peritoneal
yn

dialysis. He asks her what the difference between them is. The nurse explains that the
biggest difference between these two approaches is which of the following?
.m

A) Peritoneal dialysis relies on diffusion whereas hemodialysis relies on active


w

transport.
w

B) The peritoneum of the body serves as the semipermeable membrane in peritoneal


dialysis, whereas an extracorporeal semipermeable membrane is used in
w

hemodialysis.
C) Hemodialysis uses a Tenckhoff catheter, whereas peritoneal dialysis uses a venous
catheter.
D) Hemodialysis uses machines called cyclers to cycle the infusion and removal of
blood, whereas peritoneal dialysis uses the bodys own vascular system to do this.
5. A patient in the ICU with severe hypotension is experiencing acute renal failure and
uremia and needs dialysis. She requires a large infusion of intravenous fluids regularly.
The nurse recognizes that which method of dialysis would be best for this patient?
A) Continuous venovenous hemofiltration (CVVH)
B) Continuous venovenous hemofiltration with dialysis (CVVH/D)
C) Intermittent hemodialysis
D) Peritoneal dialysis
6. A patient on continuous venovenous hemofiltration with dialysis experiences a significant
decrease in blood pressure. Which intervention would be most appropriate for the nurse
to carry out?
A) Decrease the amount of fluid removal.

om
B) Decrease the infusion rate of replacement fluid.
C) Administer heparin.

.c
D) Use a blood warmer to warm the dialysis lines.

ep
7. A patient on peritoneal dialysis develops a low-grade fever and complains of abdominal

pr
pain when fluid is being inserted. The nurse also observes that the peritoneal drainage
fluid is cloudy. What intervention should the nurse make?
A) Measure the patients blood pressure.
st
te
B) Begin the patient on antibiotic therapy.
ng

C) Assess the patient for signs of pulmonary congestion.


si

D) Turn the patient from side to side.


ur

8. A patient is severely hyponatremic. What would be the best nursing action?


yn

A) Put the patient on dialysis.


B) Administer 3% saline.
.m

C) Administer 0.33% saline solution.


w

D) Administer 5% dextrose in water.


w

9. A patient receiving a blood transfusion in the ICU has developed relative hypocalcemia.
w

What is the most likely cause for the hypocalcemia?


A) Citrate binding to calcium
B) Loop diuretics
C) Malabsorption syndrome
D) Lack of vitamin D
10. A patient in the ICU has acute renal failure and is an alcoholic. Which electrolyte
imbalance would the nurse most expect to find in this patient?
A) Hypokalemia
B) Hyperkalemia
C) Hypermagnesemia
D) Hypomagnesemia
11. The nurse is explaining the underlying principles of dialysis to a patient who is starting
peritoneal dialysis for the management of chronic renal failure. As part of the teaching,
what physiologic process does the nurse explain produces the most waste product
removal?

om
A) Water molecule movement by osmosis
B) Diffusion to a less concentrated area

.c
C) Active transport by an energy-driven process

ep
D) Increased osmotic gradient from the abdomen

pr
12. The patient requires urgent hemodialysis or continuous renal replacement therapy after a
suicide attempt with a variety of antidepressants. What access route for the dialysis does
the nurse anticipate? st
te
A) Vascular catheter
ng

B) Arteriovenous fistula
C) Synthetic vascular graft
si

D) Peritoneal dialysis catheter


ur

13. A critically ill patient is receiving continuous renal replacement therapy (CCRT) by
yn

continuous venovenous hemofiltration with dialysis (CVVHD) or continuous venovenous


hemofiltration (CVVH). What difference in care of this patient does the nurse anticipate
.m

when compared with other methods of CCRT?


A) Filtrate lost is equal to patient weight loss
w

B) Differences are in brands of machines used


w

C) Produced ultrafiltrate is not replaced


w

D) Replacement fluid is necessary to maintain fluid balance


14. A critically ill patient has developed acute renal failure and needs dialysis. Under what
circumstance would the nurse question the use of continuous renal replacement therapy
(CRRT)?
A) Patient requires large amounts of hourly intravenous fluids
B) Metabolic imbalances can be corrected with 3 hours of dialysis per day
C) High risk of hemodynamic instability with dialysis
D) Unlikely to tolerate rapid fluid shifts without destabilizing
15. A patient receiving continuous renal replacement therapy (CRRT) is placed on low-dose
heparin for anticoagulation of the CRRT circuit. What laboratory result would cause the
nurse to question the use of heparin?
A) Normal total platelet count
B) Partial thromboplastin time two times control
C) Partial thromboplastin time normal

om
D) Critically low total platelet count
16. The patient is receiving continuous renal replacement therapy (CRRT). The rate of

.c
ultrafiltration shows a downward trend and coagulation of the filter is suspected. What is

ep
the most appropriate first nursing action?
A) Immediately disconnect circuit from the patient.

pr
B) Decrease the rate of blood flow by pump.
C)
st
Use a saline bolus to diagnosis clot location and extent.
te
D) Raise the ultrafiltration collection device.
ng

17. A patient who developed chronic renal failure after a severe hypotensive episode has just
been told that dialysis will be necessary for the rest of her life. She and her family are
si

very upset and crying. What is the best nursing intervention?


ur

A) Ask the family to leave, as they are upsetting the patient.


yn

B) Administer intravenous sedation to the patient.


C) Encourage patient and family to express their feelings.
.m

D) Begin dialysis education immediately.


w

18. A patient has had an arteriovenous fistula placed for access for long-term hemodialysis.
w

What nursing assessment result indicates a patent and functional fistula?


w

A) Diminished intensity of palpated fistula thrill


B) Clearly auscultated bruit over fistula
C) Ability to draw blood from proximal vessel
D) Full range of motion of joints below fistula
19. The patient is scheduled to receive hemodialysis for 4 hours this morning, and several
medications for chronic diseases are scheduled to be given now. All of the medications
will be at least partially removed by dialysis. What is the best nursing action?
A) Give all medications as scheduled.
B) Give double doses of all medications.
C) Withhold medications for today only.
D) Administer medications after dialysis.
20. A patient with severe vascular fluid loss from third spacing is being treated with
intravenous hypertonic saline in an attempt to pull fluid from the extravascular space to
the vascular space. What nursing assessment result is most indicative of a serious
complication of the use of intravenous hypertonic saline?

om
A) Increased urine output
B) Decreased peripheral edema

.c
C) Slightly elevated blood pressure

ep
D) Pulmonary adventitious sounds

pr
21. The patient is receiving maintenance IV fluids, has no active fluid loss site, and has
normal renal function. Based on the physiology of fluid volume balance, what IV fluid
use does the nurse anticipate? st
te
A) Dextrose 5% in water
ng

B) 0.9% saline
C) 0.45% saline
si

D) 3% saline
ur

22. The patient has very low serum potassium and is to receive an IV bolus of potassium
yn

mixed 20 mEq in 50 mL. To give a total dose of 60 mEq potassium at the highest
recommended rate, what is the maximum intravenous rate in mL per hour? (Round your
.m

answer to the nearest whole number.)


w

Answer Key
w
w

1. A, D, E
2. B, D
3. C
4. B
5. B
6. A
7. B
8. B
9. A
10. D
11. B
12. A
13. D
14. B

om
15. D
16. C

.c
17. C

ep
18. B

pr
19. D
20. D st
te
21. C
ng

22. 100 mL per hour


si

Chapter 31 Acute Kidney Injury and Chronic Kidney Disease


ur

1. An elderly male patient in the ICU is diagnosed with acute kidney injury. This patient
demonstrates a decreased glomerular filtration rate and lowered urine sodium
yn

concentration, as well as increased BUN and serum creatinine levels. The nurse observes
that the patient takes several minutes to empty his bladder when he uses the bathroom.
.m

His blood pressure and blood glucose levels are normal. What should the nurse suspect as
the cause of this patients acute kidney injury?
w

A) Tubular necrosis as a result of accumulation of radiocontrast dye in the renal tubular


w

cells
w

B) Obstruction of the flow of urine due to benign prostatic hypertrophy


C) Lack of perfusion due to congestive heart failure
D) Hypotension due to systemic inflammatory response to sepsis
2. A patient develops toxic acute tubular necrosis (ATN) as a result of exposure to a
radiocontrast dye. Which of the following should the nurse most expect to observe in this
patient as this condition progresses beyond the onset phase?
A) Normal potassium levels
B) Duration of 7 to 14 days
C) Normal urine concentrating function
D) Normal urine volume
3. A patient with acute kidney injury (AKI) demonstrates blue mottling of the skin in her
fingers. What other finding would tend to indicate that the cause of this condition is
intrarenal?
A) Distended bladder
B) Edema

om
C) Strep throat infection
D) Kinked Foley catheter

.c
4. A patient with acute kidney injury (AKI) demonstrates oliguria, a urine osmolality of 550

ep
mOsm/kg H2O, increased urine specific gravity, urine sodium of 15 mEq/L, and a
BUN:creatinine ratio of 23:1. Which of the following is a cause of AKI that would best fit

pr
with these findings?
A) Congestive heart failure st
te
B) Nephrotoxicity due to aminoglycoside antibiotics
ng

C) Hypertension
D) Retroperitoneal tumor
si

5. A patient is concerned about her steadily worsening chronic kidney disease and asks the
ur

nurse at what point she will require dialysis or renal transplantation. Which of the
following should the nurse mention?
yn

A) When your urine albumin-to-creatinine ratio is greater than 25 mg/g


.m

B) When your urine output is less than 0.5 mL/kg/h 6 h


w

C) When your glomerular filtration rate (GFR) falls below 15 mL/min/1.73 m2


w

D) When your urine osmolality is greater than 500 mOsm/kg H2O


w

6. A patient with chronic kidney disease is receiving an ACE inhibitor. The nurse
understands that this medication helps slow the progression of this disease through what
process?
A) It lowers the level of blood glucose.
B) It prevents nephron hyperfiltration.
C) It increases the urine output.
D) It filters waste from the blood.
7. A patient with prerenal acute kidney injury is oliguric. The nurse is administering an IV
bolus to the patient. What should be of primary concern to the nurse while performing
this task?
A) Restricting the patients protein intake
B) Monitoring the patients potassium level
C) Evaluating the patient for signs of nephrotoxicity
D) Preventing fluid overload
8. A patient with acute kidney injury (AKI) complains of a headache. He vomits several

om
times and breathes deeply and rapidly. His heart rate is 110 bpm, and his serum
potassium level is elevated. The nurse recognizes in this patient which condition
commonly associated with AKI?

.c
A) Fluid overload

ep
B) Anemia

pr
C) Metabolic acidosis
D) Pericarditis st
te
9. A patient with chronic kidney disease has a serum potassium level of 5 mEq/L and no
changes on the ECG. What is the proper nursing intervention?
ng

A) Administer sodium polystyrene as an enema.


si

B) Administer IV calcium gluconate.


ur

C) Administer IV insulin and dextrose.


yn

D) Begin dialysis.
10. A patient has been diagnosed with prerenal acute renal failure. What condition most
.m

likely caused this situation?


w

A) Toxic levels of medications


w

B) Poststreptococcal glomerulonephritis
w

C) Severe sepsis and shock


D) Benign prostatic hypertrophy
11. The patient is in hypovolemic shock, with mean arterial pressures below 90 mm Hg and a
very low urine output. An IV drip of norepinephrine is prescribed to keep blood pressure
above 90 mm Hg. No other therapy is initiated. What effect on kidney function does the
nurse expect?
A) Improvement in renal perfusion secondary to improved blood pressure
B) Reduction in urine output secondary to constriction of renal arteries
C) Augmentation of water reabsorption from distal tubular fluid
D) Decrease in urine sodium concentration to critically low levels
12. In a patient with acute ischemic tubular necrosis, urine output has increased from below
normal to very high. What is the nursing priority of care during this phase of renal
failure?
A) Restrict fluid intake
B) Monitor serum potassium

om
C) De-emphasize dialysis
D) Monitor serum creatinine

.c
13. A patient with a history of diabetes mellitus has had a procedure using radiocontrast dye.

ep
The patients laboratory results include high urine sodium, urine with muddy-brown
granular casts and tubular epithelial cells, and increased blood urea nitrogen (BUN) and

pr
serum creatinine. Renal ultrasonography is normal. Urine volume is normal. Which
treatment does the nurse anticipate?
A) Increased fluids
st
te
B) Renal stent placement
ng

C) Irrigation of urinary catheter


si

D) Diuretic therapy
ur

14. A patient in intensive care with acute tubular necrosis from a toxic ingestion has been
started on renal replacement therapy. The family expresses concern that the patient will
yn

not be able to afford dialysis after discharge from the hospital. In responding to the
family, what should the nurse consider?
.m

A) The family is in crisis and unable to respond rationally.


w

B) Toxic acute tubular necrosis has a higher likelihood of complete healing.


w

C) Since the patient is currently oliguric, renal replacement therapy is indicated.


w

D) The patient is unlikely to survive this illness, so the cost of long-term dialysis is not
an issue.
15. A patient has just been diagnosed with type 2 diabetes mellitus. During teaching, what
strategy should the nurse emphasize as protective of kidney cells?
A) Monitoring glycosylated hemoglobin every 3 months
B) Strict adherence to prescribed weight-loss diet
C) Restriction of sodium-containing beverages and food
D) Strict control of serum glucose levels with diet and medication
16. A patient with chronic renal disease is involved in a motor vehicle crash and experiences
severe hypovolemia. In caring for this patient in the CCU, which of the following is the
most important for the nurse to monitor?
A) Blood pressure
B) Fluid volume recovery
C) Urine output
D) Cardiac dysrhythmias

om
17. A patient has been diagnosed with chronic renal failure. What closely associated
pathophysiologies should the nurse assess for? Select all that apply.

.c
A) Hypertension

ep
B) Arteriosclerotic disease

pr
C) Traumatic injury
D) Type 2 diabetes mellitus
st
te
E) Preeclampsia
F) Type 1 diabetes mellitus
ng

18. A patient in oliguric renal failure is receiving IV furosemide (Lasix). What nursing
si

assessment has the highest priority?


ur

A) Daily weights
yn

B) Intake and output


C) Serum potassium
.m

D) Blood urea nitrogen


w

19. A patient with chronic renal disease has mild metabolic acidosis with a pH 7.30 and
w

bicarbonate level 16 mEq/L. What treatment does the nurse anticipate?


w

A) IV sodium bicarbonate
B) Reduction of respiratory rate
C) Sodium citrate and citric acid (Bicitra)
D) Massive IV fluids
20. A patient with chronic renal failure also has chronic anemia, arteriosclerotic disease, and
diabetes mellitus. The patient asks the nurse why the anemia is persisting. In answering
the patients question, what should the nurse most consider?
A) The patient most likely has preexisting chronic anemia.
B) Erythropoietin is primarily produced in the kidney.
C) The patient is receiving low-dose aspirin therapy.
D) Chronic renal failure results in persistent uremia.
21. The nurse is teaching a patient with chronic renal failure and diabetes mellitus about
nutrition. What should be included?
A) Calorie restriction based on ideal body weight is necessary.

om
B) Sodium and potassium should be supplemented while on dialysis.
C) Renal diet restrictions take the place of those for diabetes mellitus.

.c
D) Moderate protein restriction is recommended while otherwise healthy.

ep
Answer Key

pr
1. B
2. D
st
te
3. C
ng

4. A
si

5. C
ur

6. B
yn

7. D
.m

8. C
9. A
w

10. C
w

11. B
w

12. B
13. A
14. B
15. D
16. B
17. A, B, D, F
18. C
19. C
20. B
21. D

Chapter 32 Anatomy and Physiology of the Nervous System

1. A patient in the ICU has a malignant brain tumor. Based on knowledge of the cellular
units that make up the nervous system, the nurse understands that which cell type is the

om
least likely to be the origin of this patients tumor?
A) Neuron

.c
B) Astrocyte

ep
C) Ependymal cell

pr
D) Oligodendroglia

st
2. A patient was involved in a car accident and is in hypovolemic shock, with severe
hypotension. Due to damage to the glossopharyngeal nerve, the baroreceptor reflex is
te
inhibited in the patient. What effect of this impaired reflex should the nurse expect to see
in this patient?
ng

A) Rapid increase in blood pressure


si

B) Lack of immediate withdrawal from painful stimuli


ur

C) Lack of compensatory increase in heart rate


yn

D) Continuance of motor activity in the intestines


.m

3. A patient in the ICU, in addition to his critical illness, suffers from clinical depression
and is receiving medication for it. The nurse understands that this patients anti-depression
medications are most likely to inhibit the reuptake of which of the following
w

neurotransmitters? Select all that apply.


w

A) Acetylcholine
w

B) Endorphin
C) Enkephalin
D) Serotonin
E) Norepinephrine
F) Substance P
4. A patient in the ICU is receiving an experimental drug to treat a brain tumor. The nurse
understands that which of the following characteristics would be important in such a
drug? Select all that apply.
A) Protein-based
B) Lipid-soluble
C) Small-molecule
D) Water-soluble
5. A patient in the ICU has suffered severe head trauma as the result of a diving accident.

om
Damage was sustained only in the frontal lobes of the brain. Which function is most
likely to be impaired with this injury?

.c
A) Breathing

ep
B) Maintain steady posture when standing
C) Sneezing

pr
D) Writing with a pen
st
6. After having a stroke, an elderly patient is unsteady on his feet and cannot walk across the
te
room without assistance. The nurse recognizes that which sensation is most likely
impaired in this patient?
ng

A) Visceral
si

B) Proprioceptive
ur

C) Exteroceptive
yn

D) Visual
7. A neurologist is testing a patients responsiveness to various stimuli. He pricks the patients
.m

foot with the point of a safety pin, and the patient jerks her foot back immediately. The
nurse recognizes that the integrative central nervous system component involved in this
w

reflex is which of the following?


w

A) Spinal cord interneuron


w

B) Medulla oblongata
C) Midbrain
D) Cerebrum
8. A patient who experienced severe trauma in a multiple car accident on the freeway
explained that he didnt even notice the pain of his injuries as he was helping his daughters
escape from their burning car. He asked the nurse what kept him from feeling the pain
immediately. What would be the nurses best reponse?
A) The accident occurred when it was cold outside.
B) The patient has fewer nociceptors than normal.
C) The patient has an impaired thalamus.
D) Endogenous opioid peptides blocked transmission of the pain message to the brain.
9. A nurse witnesses a patient having a panic attack in the ICU after learning that he has
terminal lung cancer. Recognizing that he is in the alarm phase of the general adaptation
syndrome, what physiological adaptations should the nurse expect?
A) Drop in blood pressure

om
B) Release of cortisol
C) A shock state

.c
D) Heart rate drops from its peak but remains elevated

ep
10. A patient was recently admitted to the ICU with a knife wound to his chest that punctured

pr
his lung following a fight with his brother. He has undergone surgery and is stable, but is
afraid of the possibility of infection. He will have to miss work for at least 6 weeks while
st
he recovers. Which stressor will likely have the most long-term negative impact on this
patient?
te
A) His relationship with his brother
ng

B) The pain from his wound


si

C) The financial hardship of missing 6 weeks of work


ur

D) Fear of infection
yn

11. A patient is receiving a medication that has difficulty crossing the bloodbrain barrier.
What type of healthy neuroglia does an intact bloodbrain barrier indicate?
.m

A) Microglia
w

B) Astrocytes
w

C) Neurons
w

D) Oligodendroglia
12. A patient has a disease resulting in a deficit of acetylcholinesterase. What effects will the
patient experience?
A) Increased rapidity of neural transmission
B) Reversed direction of neural transmission
C) Rapid resetting of postsynaptic membrane receptors
D) Delayed resetting of postsynaptic membrane receptors
13. A patient has experienced a traumatic and significant loss of cerebrospinal fluid. To
prevent further injury, what is the highest nursing priority?
A) Institute seizure precautions.
B) Prevent blows to the head.
C) Keep on strict bed rest.
D) Pharmacologic paralysis.
14. A patient has a ruptured aneurysm on a vessel that joins the circle of Willis. If this results
in significant neurologic deficit, what additional factor would the nurse expect?

om
A) Fully patent circle of Willis
B) Additional atrophic vessel on the circle

.c
C) Affected vessel is the internal carotid artery

ep
D) Affected vessel is the basilar artery

pr
15. A patient has experienced head trauma involving damage to the thalamus. What behavior
does the nurse anticipate?
st
te
A) Persistent wakefulness
B) Complete lack of pain perception
ng

C) Disturbance of diurnal rhythms


si

D) Disturbances in speech formation


ur

16. After a head injury, a patient has entered a state of persistent coma. What brainstem
yn

network has been injured?


A) Bulboreticular formation
.m

B) Limbic system
w

C) Sleep centers
w

D) Reticular activating system (RAS)


w

17. A patient has damage to a spinal nerve involving the dorsal root of the nerve. What
effects does the nurse expect?
A) Loss of sensation and motor control
B) Loss of sensation
C) Loss of motor function
D) Varies with specific nerve
18. The patient is experiencing an episode of supraventricular tachycardia at a rate of 280
beats per minute. The physician uses carotid sinus massage in an attempt to slow the heart
rate. What normally occurring reflex is the underlying basis of this therapy?
A) Baroreceptor
B) Deep tendon
C) Superficial spinal
D) Withdrawal
19. The patient has a painful extremity injury and is receiving an opioid analgesic and a

om
nonsteroidal anti-inflammatory drug (NSAID) as needed along with rest, elevation, and
cold therapy to the injured area. What therapy is most directly related to the cause of the
pain?

.c
A) Opioid analgesic

ep
B) Immobilization of extremity

pr
C) Nonsteroidal anti-inflammatory drug
D) Elevation of extremity st
te
20. A patient has suffered a serious injury complicated by critical illness and is being cared
for in the CCU. One of the goals of therapy is to support the patients native state of
ng

homeostasis. What nursing action is designed to support or maintain homeostasis?


si

A) Administer antacids to prevent gastric alkalinity.


ur

B) Give large amounts of IV fluids.


C) Administer pharmacologic paralyzing agents.
yn

D) Identify and remove stressors.


.m

21. A patient who has recently been through a job change and a divorce has been admitted to
the CCU after an accidental trauma. He has had massive blood loss and exposure to
w

extreme cold. Considering the stress response, what does the nurse understand?
w

A) Healing from the trauma will progress more rapidly.


w

B) As stressors are additive, adverse effects are more likely.


C) Only the physiologic stress is significant for this patient.
D) Alleviation of psychological stress is the lowest priority.
22. The patient has had a massive myocardial infarction resulting in severe compromise of
contractility and hence cardiac output. The patients body responds to this insult by
activating the stress response. What is the most significant adverse effect that activation
of the stress response will have for this patient?
A) Inhibition of parasympathetic outflow and gastrointestinal activity
B) Stimulation of alpha receptors and increase in blood pressure
C) Inhibition of venous constriction and reduction of volume return
D) Stimulation of beta-1 receptors and increased cardiac contractility

Answer Key

1. A

om
2. C
3. D, E

.c
4. B, C

ep
5. D

pr
6. B
7. A st
te
8. D
ng

9. B
10. A
si

11. B
ur

12. D
yn

13. B
.m

14. B
15. C
w
w

16. D
w

17. B
18. A
19. C
20. D
21. B
22. B
Chapter 33 Patient Assessment: Nervous System

1. The nurse is conducting tests of a patients cerebellar synchronization of movement with


balance. What result of the finger-to-nose test would indicate cerebellar dysfunction?
A) The patient has trouble understanding the nurses instructions.
B) The patient cannot reach his or her nose due to restricted range of motion in the
elbow.
C) The patient overshoots when trying to touch his or her nose.

om
D) The patient cannot touch his or her nose due to a trembling hand.
2. A neurologist who is testing a patient for neurological deficits has her close her eyes and

.c
then hands her a stethoscope. He asks her to identify the object by touch, but she cannot
do so. The nurse, observing this result, should suspect possible damage to which part of

ep
the brain?

pr
A) Cerebellum
B) Temporal lobe
C) Frontal lobe
st
te
D) Parietal lobe
ng

3. A nurse is assessing the plantar reflex in a patient. Which of the following results would
si

indicate abnormal response and a possible lesion in the pyramidal tract? Select all that
apply.
ur

A) Plantar flexion of all toes


yn

B) Dorsiflexion of the big toe with fanning of the other toes


.m

C) No response at all
D) Dorsiflexion of the big toe without fanning of the other toes
w

E) Ticklishness
w
w

4. A patient admitted to the ICU following a car accident in which she suffered multiple
traumatic injuries. She has a fever of 101F and complains of headache. When the
physician tries to examine her eyes with a bright light, she jerks away. The nurse suspects
meningeal irritation. What other signs would likely accompany this condition? Select all
that apply.
A) Nuchal rigidity
B) Drainage of cerebrospinal fluid from the nose
C) Drainage of cerebrospinal fluid from the ear
D) Bruising over the mastoid areas
E) Pain in the neck when the thigh is flexed and the leg is extended at the knee
F) Involuntary flexion of the hips when the neck is flexed toward the chest
5. A patient suffered damage to cranial nerve VIII in a jet ski accident. What symptom or
symptoms should the nurse tell the patient to expect? Select all that apply.
A) Loss of the sense of smell
B) Partial blindness in one eye

om
C) Paralysis of face muscles on one side of the face
D) Tinnitus

.c
E) Dizziness

ep
F) Inhibited ability to move the tongue
6. A patient has trouble shrugging his shoulders and turning his head from side to side

pr
against resistance. Which nerve should the nurse suspect to be involved?
A) Cranial nerve XI st
te
B) Cranial nerve XII
ng

C) Cranial nerve X
D) Cranial nerve IX
si

7. A patient begins to demonstrate loss of consciousness as a result of intracranial pressure.


ur

He also demonstrates bradycardia and decreased irregular respirations. Which of the


yn

following best describes the identified manifestations?


A) Rhinorrhea
.m

B) Meningeal irritation
w

C) Cheyne-Stokes respirations
w

D) Cushings triad
w

8. A patient is preparing to undergo magnetic resonance imaging for possible diagnosis of


cerebral infarction. What is the nursing priority in preparing the patient for this diagnostic
study?
A) Checking the patients history for renal insufficiency, which could complicate use of
a contrast medium.
B) Confirming that the patient does not have any ferrous surgical clips or implants.
C) Determining whether the patient is on anticoagulant therapy, which would be a
contraindication.
D) Preparing an 18-gauge needle for insertion.
9. A patient is undergoing myelography for assessment of a spinal cord tumor. How should
the nurse position the patient for this procedure?
A) Head elevated 30 to 45 degrees
B) Head lowered below the level of the feet
C) Side-lying with the patients bed level
D) Prone with the patients bed flat

om
10. Following a lumbar puncture for CSF analysis, a patient with elevated intracranial
pressure develops a headache, nuchal rigidity, fever, and difficulty voiding. What
intervention should the nurse expect?

.c
A) Administration of IV fluids

ep
B) Administration of antibiotic

pr
C) Injection of blood into the dura
D) Cardiopulmonary resuscitation
st
te
11. The nurse is performing a physical examination on a patient with neurologic disease.
What finding from the examination is the most indicative of diminished cerebral
ng

hemisphere functioning?
A) Deteriorating level of consciousness
si

B) Positive Romberg test


ur

C) Unequal pupillary response


yn

D) Glasgow Coma Scale score of 15


.m

12. While assessing motor function, the nurse applies pressure to a toenail. What patient
response is most normal?
w

A) Extension of both feet


w

B) Flexion of knee and ankle


w

C) Extension of one or both arms


D) Kicking the nurses hand away
13. The patient has been in a motor vehicle crash and is in the critical care unit with severe
brain injury. She is comatose but when painful stimuli are applied she extends, adducts,
and hyperpronates her upper extremities and has plantarflexion of the feet. This action is
called what?
A) Decorticate posturing
B) Decerebrate posturing
C) Clonic-tonic activity
D) Flacidity
14. When describing a patients responsiveness, the nurse uses the term obtunded. What is the
most accurate meaning of this term?
A) Unable to arouse with any stimulus
B) Sedated with intravenous medications

om
C) Having inborn mental retardation
D) Arousable but drowsy and slow to respond

.c
15. During a neurologic examination, the nurse finds bilateral pronator drift and diminished
ability to raise legs against resistance. These findings are consistent with what neurologic

ep
deficit?

pr
A) Damage to motor neuron pathways
B) Hyperthyroidism
st
te
C) Demyelinization of afferent fibers
D) Cerebral cortex hypoperfusion
ng

16. As part of the neurologic examination, the nurse instructs the patient to perform a
si

Romberg test. What nursing action best provides for patient safety if the results are
abnormal?
ur

A) Have suction equipment on hand


yn

B) Be prepared to catch the patient if he or she falls


.m

C) Have the patient perform the test in a seated position


D) Have the patient perform the finger-to-nose test before this test
w
w

17. Although awake and alert, a patient who has experienced a neurologic insult is having
difficulty maintaining a patent airway and requires frequent jaw thrust maneuvers. What
w

neurologic damage is this finding most closely associated with?


A) High cervical spinal cord lesion
B) Cerebral infarction of the brainstem
C) Damage to the eighth cranial nerve
D) Damage to the second and third cranial nerves
18. During a craniotomy, the patient experienced peripheral damage to cranial nerve VII,
resulting in diminished movement of the left side of the face. What nursing action
demonstrates best understanding of the effect of this lesion on the patient?
A) Referral for speech therapy and swallowing assessment
B) Use of ocular moisturizers
C) Teaching about falling and syncope risks
D) Referral for evaluation for a hearing aid
19. One of the major goals of therapy for a patient with a head injury is to control rising

om
intracranial pressure (ICP). What assessment data would first cause the nurse to suspect
rising ICP?

.c
A) Deteriorating level of consciousness

ep
B) Brisk pupils with equal reactivity
C) Absence of speech secondary to sedative use

pr
D) Narrow pulse pressure and hypotension
st
20. A patient with a suspected cervical spine fracture is undergoing computed tomography
te
(CT) of the cervical spine for definitive diagnosis. During the procedure, what is the
nursing priority?
ng

A) Protection of cervical spine stability


si

B) Reassurance and anxiety reduction


ur

C) Explanation of the reason for the test


yn

D) Evaluation for allergic response to contrast medium


21. A patient with serious neurologic trauma is being evaluated for brain death using cerebral
.m

blood flow studies. What is the most accurate information the nurse can provide when
discussing the test with the family?
w

A) If the test shows no blood flow to the cerebral hemispheres, brain death is definite.
w

B) If the test shows adequate flow to the cerebral hemispheres, the brain is viable.
w

C) The test will give a more accurate measurement of intracranial pressure.


D) The test is performed only after all sedative and pain medications are discontinued.
22. The patient is undergoing myelography to evaluate his back pain. What are appropriate
nursing interventions? Select all that apply.
A) Elevate the head of the bed to 30 to 45 degrees.
B) Restrict total fluid intake.
C) Avoid phenothiazines.
D) Institute seizure precautions.
E) Initiate ambulation immediately.
F) Turn, cough, and deep breathe every 2 hours.

Answer Key

1. C

om
2. D
3. B, D

.c
4. A, E, F

ep
5. D, E

pr
6. A
7. D st
te
8. B
ng

9. A
10. C
si

11. A
ur

12. D
yn

13. B
.m

14. D
15. A
w
w

16. B
w

17. A
18. B
19. A
20. A
21. A
22. A, C, D
Chapter 34 Patient Management: Nervous System

1. A nurse is monitoring a patient recently admitted to the critical care unit with an acute
brain injury. She is aware that intracranial hypertension is a major risk associated with
brain injury. Which of the following findings would definitively indicate that the patient
has intracranial hypertension?
A) Cerebral perfusion pressure (CPP) of 75 mm Hg
B) Intracranial pressure (ICP) of 25 mm Hg

om
C) Mean arterial pressure (MAP) of 150 mm Hg
D) Systolic pressure of 110 mm Hg

.c
2. A patient is demonstrating increased pulse pressure, decreased pulse, and irregular
respiration. The nurse recognizes these symptoms of increased intracranial pressure and

ep
understands that the patients autoregulation of cerebral blood flow in the brain has failed.
Which of the following findings would be consistent with a failure of autoregulation of

pr
blood flow in the brain? Select all that apply.
A) Cerebral perfusion pressure of 40 mm Hg st
te
B) Mean arterial pressure of 170 mm Hg
ng

C) Systolic pressure of 120 mm Hg


D) Intracranial pressure of 35 mm Hg
si

3. A nurse is working with a patient with acute head trauma. The nurse understands that the
ur

cerebral blood flow in this patient is decreased to compensate for the cerebral edema
caused by the trauma. What other compensatory mechanisms should the nurse be aware
yn

of that are likely occurring in this patient to maintain a constant intracranial pressure?
Select all that apply.
.m

A) Expansion of cisterns and ventricles


w

B) Increased cerebrospinal fluid (CSF) production


w

C) Increased CSF absorption


w

D) Shunting of CSF into the spinal subarachnoid space


4. A patient in the ICU with severe head trauma remains stable for the first 24 hours after
admission, with no indication of intracranial hypertension. Suddenly, however, the patient
begins showing signs of Cushings triad. The nurse recognizes that this occurrence
indicates that the patients compensatory mechanisms have become exhausted. What
physiological change occurs as part of this exhaustion of compensatory mechanisms?
Select all that apply.
A) Decrease in volume of contents of the intracranial compartment
B) Decrease in intracranial pressure
C) Decrease in cerebral perfusion
D) Decrease in compliance within the intracranial compartment
5. Two patients, a husband and wife, are admitted to the ICU after sustaining traumatic head
injuries in a motor vehicle accident. The husband is in a coma but shows no abnormalities
on a CT scan. He is 45 years old and has a systolic blood pressure of 85 mm Hg. The
wife, 42 years old, is not comatose and has a normal CT scan, but shows signs of brain
injury, and has a systolic blood pressure of 80 mm Hg. The nurse recognizes that
intracranial pressure monitoring is indicated for which of these patients?

om
A) Both the husband and wife
B) Neither the husband nor the wife

.c
C) The husband only

ep
D) The wife only

pr
6. A patient recently admitted to the ICU for head trauma has a Glasgow Coma Scale score
of 4 and a hematoma apparent on CT scan of the head. This patient has multiple fractures
st
in her skull and an intracranial pressure (ICP) of 30 mm Hg. Which ICP monitoring
te
device would be contraindicated for this patient?
ng

A) Intraventricular
B) Intraparenchymal
si

C) Lumbar/subarachnoid
ur

D) Subdural
yn

7. A patient with head trauma requires intracranial pressure (ICP) monitoring. The physician
insists that the most accurate monitoring device feasible should be used for this patient.
.m

This patient also requires frequent draining of cerebrospinal fluid (CSF) while being
monitored. The nurse recognizes that which ICP monitoring device would be best for this
w

patient?
w

A) Intraventricular
w

B) Subarachnoid
C) Subdural
D) Epidural
8. A patient with head trauma is being monitored with an intraventricular catheter device
(IVC). The patients intracranial pressure (ICP) had been staying around 20 mm Hg, but
moments ago, it spiked up to 55 mm Hg. What complication related to the monitoring
device itself would best explain this dramatic increase in ICP?
A) Infection at the catheter access site
B) Obstruction of the catheter
C) Hemorrhage
D) Misplacement of catheter
9. A patient with a brain injury is receiving IV mannitol to reduce cerebral edema and
intracranial pressure (ICP) during the early resuscitation phase. The patient is rapidly
becoming hypovolemic. What intervention should the nurse make to help correct the
hypovolemia?

om
A) Administer crystalloid solution.
B) Administer morphine.

.c
C) Discontinue mannitol.

ep
D) Administer propofol.

pr
10. A patient with severe and refractory elevated intracranial pressure (ICP) has been in an
induced barbiturate coma for 48 hours. Over the first 24 hours, the patients ICP decreased
st
from 30 to 14 mm Hg and her systolic blood pressure decreased from 130 to 80 mm Hg.
These changes were sustained in the second 24 hours. The nurse recognizes that which of
te
the following is the appropriate intervention for this patient?
ng

A) Administer IV solution.
si

B) Discontinue barbiturate therapy.


ur

C) Initiate hypothermia therapy.


D) Administer a sedative.
yn

11. A patient with a brain injury is undergoing intracranial pressure (ICP) monitoring and
.m

cerebrospinal fluid (CSF) drainage, along with mannitol therapy, to relieve ICP. What
other intervention can the nurse make to aid in relieving this patients ICP?
w

A) Extend and rotate the patients head.


w

B) Flex the patients hips to greater than 90 degrees.


w

C) Elevate the head of the patients bed to 20 degrees.


D) Perform frequent blood draws.
12. Regulation of intracranial pressure is explained by the Monro-Kellie doctrine. Based on
this doctrine, under normal circumstances, if a patients brain volume increases, what
compensatory change does the nurse expect?
A) Reduction of cerebral blood volume
B) Increased cerebrospinal fluid volume
C) Increased cerebrospinal fluid production
D) Elevation of systemic blood pressure
13. A patient with a head injury is being monitored with an intracranial pressure monitor.
What nursing assessment best indicates intracranial hypertension?
A) Intracranial pressure 8 mm Hg
B) Glasgow Coma Scale score 3
C) Intracranial pressure 25 mm Hg

om
D) Glasgow Coma Scale score 15
14. The patient has undergone intracerebral surgery. Knowing that interruption of the skull
interferes with the brains ability to autoregulate, what nursing assessment information

.c
most clearly indicates the highest patient risk?

ep
A) Pulmonary adventitious sounds

pr
B) Capillary refill less than 2 seconds
C) Blood pressure consistently elevated
st
te
D) Pain at 8 on 0-to-10 scale
15. The patient is in a critical care unit after an acute head injury and has developed
ng

respiratory and ventilatory failure and hypotension. What effect will this development
have on the patients cerebral perfusion pressure?
si

A) Elevated above 100 mm Hg


ur

B) Reduced below 60 mm Hg
yn

C) Will make it very labile


.m

D) Will have very little effect


16. Intracranial pressure monitoring can be a valuable diagnostic tool but also has significant
w

complications. In what patient with a severe head injury would the nurse question the use
w

of intracranial pressure monitoring?


w

A) Glasgow Coma Scale score of 3


B) Declared brain dead
C) Subarachnoid hematoma
D) Severe stroke
17. The patient has elevated intracranial pressure secondary to overproduction of
cerebrospinal fluid. During therapy, an intracranial pressure monitor is placed. What type
of monitor should be used if withdrawal of cerebrospinal fluid is also desired?
A) Intraventricular
B) Epidural
C) Intraparenchymal
D) Subdural
18. The patient is being monitored with an intracranial pressure monitor. What nursing
assessment most indicates the development of a complication of intracranial pressure
monitoring?
A) Purulent drainage around monitor access site

om
B) Intracranial pressure 12 mm Hg at rest
C) Intracranial pressure 20 mm Hg during suctioning

.c
D) Development of slight respiratory alkalosis

ep
19. The patient has an intracranial pressure monitor. After the patient returns from a

pr
computed tomography (CT) scan of the head, the nurse notices that the patients
intracranial pressure is significantly lower than before the scan. What nursing action is
most likely to identify a cause of this change?st
te
A) Take vital signs.
ng

B) Flush monitor tubing toward patient.


C) Relevel the transducer.
si

D) Drain cerebrospinal fluid.


ur

20. A patient who is on an intracranial pressure monitor after an acute head injury has an
yn

intracranial pressure of 10 mm Hg at rest. When the patient is being suctioned, the


intracranial pressure rises briefly to 20 mm Hg but returns quickly to 10 mm Hg once the
.m

suctioning has ceased. What is the most appropriate nursing intervention?


A) Administer intravenous sedation
w

B) Suction no longer than 15 seconds each time


w

C) Drain 10 mL cerebrospinal fluid


w

D) Return to supine position


21. Seven days after a traumatic head injury, a patient has elevated intracranial pressure that
is refractory to sedation, paralysis, cerebrospinal fluid drainage, and osmotic diuretics.
The patients arterial blood gas results are pH 7.45, PaCO2 33, and bicarbonate ion 18.
What is the best nursing decision?
A) Reduce respiratory rate
B) Administer intravenous bicarbonate
C) Increase intravenous sedation
D) Continue with current plan of care
22. A patient with an acute brain injury is receiving IV mannitol, an osmotic diuretic. If this
medication is effective, what does the nurse expect?
A) Increased cerebral perfusion pressure
B) Increased serum osmolarity above 320 mOsm
C) Reduction of Glasgow Coma Scale values

om
D) Development of fixed and dilated pupils

.c
Answer Key

ep
1. B

pr
2. A, B, D
3. C, D
4. C, D
st
te
5. A
ng

6. C
si

7. A
ur

8. B
yn

9. A
.m

10. B
11. C
w

12. A
w

13. C
w

14. C
15. B
16. B
17. A
18. A
19. C
20. B
21. D
22. A

Chapter 35 Common Neurosurgical and Neurological Disorders

1. A patient has recently undergone successful surgery to remove a brain tumor and will be
discharged soon. Which of the following would be the appropriate nursing intervention or
interventions at this point? Select all that apply.

om
A) Encourage the patient to return to her normal routine as soon as she is able.
B) Instruct the patient to continue bed rest for 3 months.

.c
C) Refer the patient to hospice care.

ep
D) Discuss the side effects of new medications with the patient.

pr
E) Use teaching sheets to review the patients plan of care.
F) st
Have family members leave the room while discussing the patients plan of care.
te
2. A patient with a brain tumor is experiencing headaches, seizures, and nausea. The nurse
understands that this patient has an increased possibility of having which types of tumor?
ng

Select all that apply.


A) High-grade tumor
si

B) Posterior fossa tumor


ur

C) Cerebral hemisphere tumor


yn

D) Low-grade tumor
.m

3. A nurse is caring for a patient with an arteriovenous malformation (AVM). What


symptom is the nurse most likely to observe in this patient?
w

A) Seizure
w

B) Headache
w

C) Intracranial pressure
D) Hemorrhage
4. A patient has a pituitary tumor that must be removed surgically. The nurse understands
that this patient will most likely undergo what type of surgery in this case?
A) Craniotomy
B) Transsphenoidal surgery
C) Transcranial surgery
D) Stereotactic biopsy
5. A patient has recently had a hemorrhagic stroke. The nurse should most strongly suspect
which precipitating factor in this patient?
A) Myocardial infarction
B) Hypertension
C) Atrial infarction

om
D) Diabetes
6. A patient has been found to have expressive, or nonfluent, dysphasia following a stroke
and is having difficulty communicating with his family. What would be the appropriate

.c
nursing intervention for this patient?

ep
A) Explain to the family that the patient is intellectually impaired.

pr
B) Explain to the family that the patient is unable to understand the meaning of spoken
language.
C)
st
Refer the patient to a rehabilitation specialist.
te
D) Give the patient a note pad so that he can write responses to questions posed to him.
ng

7. A patient with epilepsy experiences a tonic-clonic seizure while in the ICU. What is the
most appropriate nursing intervention?
si

A) Leave the patient and find a physician immediately.


ur

B) Restrain the patient by tying his wrists to the sides of the bed.
yn

C) Turn the patient to his side.


.m

D) Administer warfarin.
8. A patient has just been diagnosed with Guillain-Barr syndrome and is experiencing
w

ascending, symmetrical muscle weakness and paralysis. Which nursing intervention could
w

best help prevent complications of immobility?


w

A) Assess strength of neck flexor muscles.


B) Promote use of antiembolism stockings.
C) Assist with plasmapheresis treatment.
D) Administer intravenous immunoglobulin.
9. A 45-year-old man in the ICU is diagnosed with generalized myasthenia gravis. The
physician is discussing treatment options with the patient. The nurse understands that
which treatment option would likely be the most effective for this patient in the long term
in terms of remission, overall survival, and clinical improvement?
A) Anticholinesterase
B) Plasmapheresis
C) Intravenous immunoglobulin
D) Thymectomy
10. A patient has been admitted to the CCU after surgery to remove a brain tumor. What
nursing assessment indicates development of a significant complication?

om
A) Gradually improving level of consciousness
B) Serum sodium levels 138 to 142 mEq/L

.c
C) Minimal serous drainage on dressing

ep
D) Sudden onset of confusion

pr
11. A patient with a probable leak from a cerebral aneurysm is scheduled for surgical repair.
st
What is the priority of nursing care in the preoperative period?
te
A) Administration of prescribed stool softener medication
B) Awaken every hour to assess neurologic status
ng

C) Encourage ambulation and exercise to tolerance


si

D) Keep on high-fiber and low-calorie diet


ur

12. The patient has a cerebral aneurysm leak. While awaiting surgical repair, the patient has
yn

an episode of extreme light-headedness and is found to have a mean arterial pressure less
than 55 mm Hg. The patients urinary output has been excessive and increasing over the
past several hours. What is the best nursing intervention for this patient?
.m

A) Enforce complete bed rest


w

B) Cautious volume replacement


w

C) Intravenous diuretic therapy


w

D) Continuous seizure precautions


13. A patient is admitted to the CCU after aneurysm clipping. What is the best nursing
strategy to prevent complications?
A) Group care tasks to allow longer rest periods.
B) Keep head elevated and in neutral position.
C) Suction every 30 minutes to maintain airway.
D) Keep monitor alarm volume on high.
14. The patient has had a stroke or brain attack, believed to be ischemic in nature. The causes
of an ischemic stroke are least likely to include which of the following?
A) Thrombus of a cerebral artery
B) Embolus of a cerebral artery
C) Ruptured cerebral aneurysm
D) Cerebrovascular obstruction

om
15. A patient is being cared for in the CCU after a ruptured cerebral aneurysm. The nurse
finds new onset of hemiparesis, slight lethargy, and complaints of diplopia. What
complication does the nurse suspect?

.c
A) Aneurysm rebleed

ep
B) Increased intracranial pressure

pr
C) Cerebral artery vasospasm
D) Carbon dioxide retention
st
te
16. While shopping, a nurses friend experiences sudden unilateral weakness and slurred
speech. What is the most important nursing intervention?
ng

A) Activate the emergency response system.


si

B) Observe for return of neurologic stability.


ur

C) Notify close family members of the victim.


yn

D) Stabilize airway and cervical spine.


17. A patient is in the emergency department being treated for an ischemic stroke. What is
.m

the nursing priority of care?


A) Refer for rehabilitation care.
w
w

B) Initiate fibrinolysis within 3 hours.


w

C) Initiate intravenous glucose therapy.


D) Administer 100% oxygen by mask.
18. A patient has been diagnosed with partial seizures. What behavior during this patients
seizures that would not occur during a generalized seizure does the nurse expect?
A) Aura prior to seizure
B) Twitching confined to one arm
C) Absence of purposeful movement
D) Postictal period or state
19. A patient with seizure disorder is being treated with oral phenytoin (Dilantin). To prevent
undesirable side effects of this medication, what is a nursing priority?
A) Monitor all seizure activity.
B) Use seizure precautions.
C) Give medication with milk.
D) Teach good oral care.

om
20. Three weeks after a viral illness, a patient has an onset of weakness in the lower
extremities that is progressing to the arms. The patient is admitted to the CCU with a
diagnosis of rapidly progressing Guillain-Barr syndrome. What is the nursing care

.c
priority?

ep
A) Monitor respiratory system closely.
B) Prevent hazards of immobility.

pr
C) Provide emotional support for patient and family.
D) Control musculoskeletal pain.
st
te
21. A patient with myasthenia gravis is being treated with corticosteroids. What symptom
ng

would indicate an undesirable side effect of the corticosteroid therapy?


A) Increased weakness with activity
si

B) Persistent ptosis
ur

C) Inability to swallow
yn

D) Stool positive for guaiac


.m

Answer Key
w
w

1. A, D, E
w

2. C, D
3. D
4. B
5. B
6. D
7. C
8. B
9. D
10. D
11. A
12. B
13. B
14. C

om
15. C
16. A

.c
17. B

ep
18. B

pr
19. D
20. A st
te
21. D
ng

Chapter 36 Traumatic Brain Injury


si
ur

1. A patient who was in a motor vehicle accident struck her forehead on the windshield of
her car after crashing into the back of another car. Given this mechanism of injury, which
yn

regions of the brain are most likely to be injured? Select all that apply.
A) Frontal lobes
.m

B) Parietal lobes
w

C) Occipital lobes
w

D) Temporal lobes
w

E) Diencephalon
F) Medulla oblongata
2. A patient was involved in a fight in which he was struck in the back of the head with a
blunt object. Scalp laceration is immediately evident. The nurse suspects cerebral edema
and ischemia. On CT scan, it appears that he has contusions and a fractured skull. On
neurological assessment, the nurse finds evidence of concussion, as the patient
demonstrates short-term memory impairment. Of these findings, which are secondary
brain injuries? Select all that apply.
A) Scalp laceration
B) Cerebral edema
C) Ischemia
D) Contusions
E) Fractured skull
F) Concussion
3. Following cerebral angiography, a patient is found to have carotid artery dissection. What

om
critical complication associated with this injury should the nurse be most concerned about
because it could lead to stroke?
A) Concussion

.c
B) Hemorrhage

ep
C) Diffuse axonal injury

pr
D) Coma
st
4. A patient with traumatic brain injury is experiencing cerebral edema, which has led to
te
severely elevated intracranial pressure. He has increased pulse pressure, decreased heart
rate, and an irregular respiratory pattern. He has lost consciousness and demonstrates
ng

bilateral pupillary dilation. The nurse recognizes that these symptoms point to which
condition?
si

A) Central herniation syndrome


ur

B) Uncal herniation syndrome


yn

C) Cerebrovascular injury
D) Diffuse axonal injury
.m

5. A patient with traumatic brain injury is being assessed. This patient demonstrates gross
w

defects in visual acuity on reading a Snellen eye chart. Which cranial nerve is most likely
damaged?
w

A) Cranial nerve I
w

B) Cranial nerve II
C) Cranial nerve III
D) Cranial nerve IV
6. A comatose patient with a traumatic brain injury is being tested for cranial nerve damage.
The nurse passes a wisp of cotton over the lower conjunctiva of each eye. The patients
lower eyelid in each eye twitches when the cotton makes contact with the cornea. The
nurse recognizes that this result indicates which of the following?
A) The trigeminal nerve is functioning properly but the facial nerve is not
B) The facial nerve is functioning properly but the trigeminal nerve is not
C) Neither the trigeminal nerve nor the facial nerve is functioning properly
D) Both the trigeminal and the facial nerves are functioning properly
7. A patient with traumatic brain injury is found to have bilateral lesions deep in the cerebral
hemispheres. What respiratory pattern should the nurse most expect to find in this
patient?
A) Alternating hyperpneic and apneic phases (Cheyne-Stokes breathing)

om
B) Sustained, regular, rapid, and deep hyperventilation (neurogenic hyperventilation)
C) Long pause at full inspiration or full expiration (apneustic breathing)

.c
D) Gasping breaths with irregular pauses (cluster breathing)

ep
8. A patient is suspected of having injury to his carotid artery following trauma to his neck

pr
after engaging in a fight during a hockey game. Which diagnostic test would be most
effective in investigating this injury?
A) Computed tomography (CT)
st
te
B) Magnetic resonance imaging (MRI)
ng

C) Electroencephalogram (EEG)
si

D) Cerebral angiography
ur

9. An inexperienced nurse who is new to the ICU is examining the eyes of a comatose
patient with traumatic brain injury who is on a ventilator. In doing so, she turns the
yn

patients head sharply to one side. After she is finished, she leaves the patients head turned
to the side. A more experienced nurse sees this and cautions the new nurse not to turn the
.m

patients head so sharply or leave it in that position. What is the best rationale for the more
experienced nurses admonition?
w

A) Compression of the jugular vein leading to increased intracranial pressure


w

B) Lack of a patent airway


w

C) Lack of dignity for the patient


D) Cramping of neck muscles
10. A nurse is caring for a patient with a traumatic brain injury who is paralyzed. The nurse
must decide how best to meet the nutritional needs of this patient. What intervention is
best to support the nutritional needs of the patient?
A) Replace 140% of the patients resting energy expenditure via parenteral nutrition.
B) Meet metabolic demands of the patient within 8 to 10 days of the injury.
C) Collaborate with a nutrition support team to meet the patients nutritional needs.
D) Maintain the patients blood sugar level at 300 mg/dL.
11. A patient has experienced a traumatic brain injury. During initial assessment, the nurse
determines that the mechanism of injury was accelerationdeceleration. What is the best
rationale for this nursing assessment?
A) Helps to predict nature of internal injuries
B) Satisfied the nurses curiosity
C) Required on admission form

om
D) May be part of legal evidence
12. A patient with a traumatic brain injury is at high risk for a secondary brain injury. What is

.c
the best nursing explanation of a secondary brain injury?

ep
A) Result of a repeated assault incident

pr
B) From a penetrating gunshot wound
C) Trauma inflicted by another person
st
te
D) Cerebral edema and ischemia
13. The patient has a depressed skull fracture resulting in a tear of the dura mater. What
ng

nursing intervention is most directed at preventing a significant complication of this


particular injury?
si

A) Elevating the head of the bed to 15 degrees


ur

B) Giving supplemental oxygen by mask


yn

C) Ensuring compliance with hand hygiene protocols


.m

D) Obtaining consent for surgical repair of fracture


14. A patient with a skull fracture has a positive halo sign. What does this sign indicate?
w
w

A) Fracture of the anterior fossa


w

B) Presence of a basilar skull fracture


C) Impingement of cranial nerves
D) Cerebrospinal fluid leak
15. A patient with a suspected skull fracture is observed to have raccoon eyes, or bilateral
periorbital bruising. What other symptom does the nurse expect?
A) Positive Battles sign
B) Cerebrospinal fluid rhinorrhea
C) Cerebrospinal fluid otorrhea
D) Maxillarycranial separation
16. About 6 weeks after a concussion injury, the patient is complaining of headaches,
decreased attention span, and short-term memory impairment. The patient expresses
extreme frustration and anxiety. What is the best nursing intervention?
A) Obtain an order for a repeat computed tomography (CT) scan
B) Admit the patient for a complete neurologic evaluation
C) Provide emotional support and explanations

om
D) Refer to psychiatry for evaluation and treatment
17. The patient has an acute subdural hematoma from an acute head injury. What is the most

.c
typical symptom that the nurse would expect during the first 2 days after the injury?

ep
A) Decreasing level of consciousness

pr
B) Labile blood pressure
C) Cardiac dysrhythmias
st
te
D) Impingement of cranial nerve 8
18. The nurse is evaluating the cognitive function of a patient with impaired neurologic
ng

functioning after an acute brain injury. What is the best nursing approach for evaluation
of orientation to person, place, and time?
si

A) Ask exactly the same questions each time.


ur

B) Ask if the patient knows where he is.


yn

C) Vary the questions slightly each time.


.m

D) Ask the family to corroborate information.


19. The nurse is assessing a patients level of arousal. Since the patient is unresponsive to
w

verbal and touch stimulation, the nurse decides to try a central pain stimulus. What
w

technique would the nurse be least likely to use?


w

A) Squeeze the trapezius muscle.


B) Apply pressure over the supraorbital notch.
C) Apply pressure to closed eyelids.
D) Perform a sternal rub.
20. A patient with a traumatic brain injury is given IV phenytoin to prevent seizures. Three
days after the drug is started, the patient develops a red, vesicular rash on her trunk. What
is the most appropriate collaborative intervention?
A) Administer an antihistamine.
B) Discontinue phenytoin.
C) Evaluate for contact dermatitis.
D) Place in contact isolation.
21. A patient with a neurologic deficit following traumatic brain injury is making very slow
progress toward normal. The family expresses distress and worry about financial and
other matters to the nurse. What is the nurses best response?
A) Referral for nursing home placement

om
B) Questions about insurance status
C) Referral to psychiatry for evaluation

.c
D) Referral to multidisciplinary rehabilitation team

ep
pr
Answer Key

1. A, C st
te
2. B, C
ng

3. B
si

4. A
ur

5. B
6. D
yn

7. A
.m

8. D
w

9. A
w

10. C
w

11. A
12. D
13. C
14. D
15. B
16. C
17. A
18. C
19. C
20. B
21. D

Chapter 37 Spinal Cord Injury

om
1. A teenaged boy jumped from a two-story building and landed on his feet, injuring his
spine, and is now in the ICU. The nurse recognizes his injury as which of the following?

.c
A) Rotational injury

ep
B) Axial loading
C) Hyperflexion

pr
D) Hyperextension
st
2. The nurse is assessing a patient with a spinal cord injury in the ICU. The patient is
te
completely paralyzed from the waist down but has sensation in his shoulders, chest, arms,
and hands. He has no control of his bowel or bladder. Which of the following are possible
ng

sites for this patients injury, given his loss of function? Select all that apply.
A) C4
si

B) C7
ur

C) T3
yn

D) T7
.m

E) T12
w

F) L4
w

3. A patient presents to the ICU with a spinal cord injury at C3 and the following: loss of
position sense, light touch, and vibratory sense below the level of the injury. However,
w

the patient has retained all motor function and pain and temperature sensation. The nurse
suspects that the injury has occurred on what portion of the spinal cord?
A) Central
B) Lateral
C) Anterior
D) Posterior
4. A patient involved in a snowmobile accident struck a tree and sustained a fractured
vertebra at C4. She demonstrates signs of ischemic areas near the injury, along with
hypoperfusion, microscopic hemorrhage, and edema. The nurse observes signs of
concussion, including loss of consciousness. Which of the following are considered
secondary injuries? Select all that apply.
A) Fractured vertebra
B) Cord ischemia
C) Hypoperfusion

om
D) Microscopic hemorrhage
E) Edema

.c
F) Concussion

ep
5. A patient with a mild spinal cord injury becomes light-headed every time she attempts to
rise from her bed. At rest, her heart rate and blood pressure are normal. All of her motor,

pr
sensory, reflex, and autonomic functions are intact. The nurse recognizes which condition
in this patient?
A) Spinal shock st
te
B) Neurogenic shock
ng

C) Orthostatic hypotension
si

D) Central cord syndrome


ur

6. A patient with a spinal cord injury has been stabilized in the ICU and now must undergo
diagnostic testing. Which test would be most appropriate for detecting a fracture of the
yn

vertebra?
A) Magnetic resonance imaging (MRI)
.m

B) Blood urea nitrogen (BUN)


w

C) Glasgow coma scale (GCS)


w

D) Computed tomography (CT)


w

7. A patient is recovering from a lumbar spine injury and requires an immobilization device
for this region. Which device would be most appropriate for this patient?
A) Halo vest
B) Aspen collar
C) Minerva brace
D) Jewett brace
8. A nurse is monitoring a patient with spinal cord injury for respiratory complications.
Which of the following findings would indicate that the patient should be intubated?
A) Respiratory rate of 20 breaths/minute
B) Vital capacity of 30 mL/kg
C) PaO2 of 90 mm Hg
D) PaCO2 of 60 mm Hg
9. After failing to effectively clear a patients airway by having him cough, the nurse is now
suctioning his airway. What complication related to suctioning should the nurse be aware

om
of?
A) Bradycardia

.c
B) Tachycardia

ep
C) Hyperglycemia

pr
D) Hypertension

st
10. A patient with a spinal cord injury and who smokes is at risk for developing deep vein
thrombosis (DVT). The nurse provides the patient with antiembolism stockings and
te
encourages her to stop smoking, as it contributes to vasoconstriction in the periphery and
thus to DVT. What other measure would be appropriate to help prevent DVT in this
ng

patient?
si

A) Administration of atropine sulfate


ur

B) Administration of heparin
C) Administration of reserpine
yn

D) Administration of methyldopa
.m

11. A patient is admitted to the emergency department after a near-drowning accident. The
patient dove head-first into shallow water and has a high blood-alcohol level.
w

Cardiopulmonary resuscitation was used at the scene. The patient is awake and alert.
w

Considering the mechanism of injury, what is the highest nursing priority?


w

A) Check vital signs often.


B) Obtain an order for radiography studies.
C) Monitor pulse oximetry closely.
D) Provide cervical spine stability.
12. A patient is in critical care recovering from a spinal cord injury. As part of shift report,
the nurse is told that the patients injury is between C1 and C4 and involves the entire
cord. The patient is on a mechanical ventilator. What is the best nursing action to provide
for patient safety?
A) Be sure all side rails are up at all times.
B) Keep the bed in low position when unattended.
C) Verify that a functioning bag-mask resuscitator is at the bedside.
D) Place the call light in the patients hand.
13. A patient was struck in the jaw and had hyperextension of the cervical spine. If the patient
has central cord syndrome, what would the nurse most expect?
A) Full loss of motor function below the lesion

om
B) Ipsilateral increased cutaneous pain at the lesion
C) Arm paralysis with intact motor function in the legs

.c
D) Full motor paralysis and loss of touch sensation below the lesion

ep
14. Patients with spinal cord injury may experience both spinal shock and neurogenic shock,
and differentiating between the two is essential. What symptoms are unique to neurogenic

pr
shock?
A) Loss of motor and sensory function
st
te
B) Flaccid paralysis below the lesion
C) Presence of poikilothermia
ng

D) Hypotension and bradycardia


si

15. During the initial assessment of patient with a probable spinal cord injury, the nurse
ur

performs a digital rectal examination. What is the best rationale for this examination?
yn

A) Part of routine admission physical


B) Checks for fecal impaction
.m

C) Assesses for sensation or movement


w

D) Preliminary for rectal medications


w

16. A patient involved in a motor vehicle accident has a high risk of spinal cord injury. At the
w

scene, what is the priority patient assessment?


A) Level of consciousness
B) Respiratory rate
C) Independent mobility
D) Peripheral sensation
17. A patient with a cervical spine fracture has been fitted with a halo vest and is to ambulate
for the first time today. What is the priority nursing action?
A) Put rubber corks on the ends of the pins.
B) Pad the edges of the vest to prevent chafing.
C) Have the patient sit on the side of the bed for several minutes.
D) Teach about loss of peripheral vision.
18. A patient has a C7-C8 spinal cord injury. During recovery, what is the nursing priority of
care?
A) Encourage the patient to do incentive spirometry exercises.

om
B) Monitor neurologic status every 4 hours.
C) Collaborate with physical therapy for exercises.

.c
D) Refer to social services for financial assistance.

ep
19. A patient with a spinal cord lesion at C6-C7 has developed pneumonia and is placed on
kinetic therapy producing constant lateral rotation to 40 degrees bilaterally. What is a

pr
nursing priority of care for this patient relative to the kinetic therapy?
A) Measure intake and output hourly.
st
te
B) Provide nutrition with adequate protein.
C) Auscultate bowel sounds every 4 hours.
ng

D) Inspect skin surfaces every 4 hours.


si

20. The patient has a spinal cord lesion at T1-T2. About an hour after being turned, the
ur

patient experiences a sudden throbbing headache accompanied by extreme blood pressure


elevation and profound bradycardia. The patient has a very flushed face. What is the
yn

nursing priority?
A) Administer pain medication immediately.
.m

B) Give intravenous beta-antagonist medication.


w

C) Turn on a fan.
w

D) Check Foley catheter for twisting or kinks.


w

21. A patient recovering from a partial spinal cord lesion is experiencing muscle spasticity.
Relative to this complication, what is the nursing priority?
A) Monitor neurologic status every 4 hours
B) Ensure compliance with exercise program
C) Medicate often for pain and discomfort
D) Emphasize nutritional balance
Answer Key

1. B
2. D, E
3. D
4. B, C, D, E
5. C

om
6. D
7. D

.c
8. D

ep
9. A
10. B

pr
11. D
12. C
st
te
13. C
ng

14. D
si

15. C
ur

16. B
yn

17. C
18. A
.m

19. D
w

20. D
w

21. B
w

Chapter 38 Anatomy and Physiology of the Gastrointestinal System

1. A child in the ICU has a cleft palate. The nurse should expect which of the following
dysfunctions in this child related to this congenital defect?
A) Nasal regurgitation
B) Vomiting
C) Diarrhea
D) Choking
2. A patient has an incompetent lower esophageal sphincter. What complication should the
nurse expect in this patient?
A) Overdistension of the lower esophagus
B) Gastric reflux
C) Duodenal reflux
D) Aspiration of food

om
3. A patient is experiencing severe abdominal pain from peptic ulcers. Which of the
following should the nurse suspect as causes? Select all that apply.
A) Bile salts from duodenal reflux

.c
B) Alcohol

ep
C) Hypersecretion of mucus

pr
D) Aspirin
E) st
Nonsteroidal anti-inflammatory drugs (NSAIDs)
te
F) Helicobacter pylori infection
ng

4. A patient who is facing open heart surgery later in the day is noticeably fearful and
agitated. The nurse should expect that the patients emotional state will have which of the
si

following effects on his gastrointestinal function?


ur

A) Decreased gastrointestinal secretion


yn

B) Increased mucus production


C) Increased risk for aspiration of food
.m

D) Decreased risk for gastric reflux


w

5. A patient has celiac disease and has damage to the villi in his small intestine. The nurse
w

understands that this condition will have what effect on the patient?
w

A) Decreased absorption
B) Gastric reflux
C) Duodenal reflux
D) Decreased peristalsis
6. A patient has trouble digesting protein. Decreased secretion of which enzyme would best
explain this dysfunction?
A) Chymotrypsin
B) Amylase
C) Bile salts
D) Intrinsic factor
7. A patient has papillary stenosis of the sphincter of Oddi. Which of the following is an
effect of this dysfunction that the nurse should expect to see?
A) Increased esophageal distension
B) Duodenal reflux

om
C) Aspiration of food
D) Decreased bile in the duodenum

.c
8. A patient has a low bile level in his small intestine. Absence of which substance would
explain this low bile level?

ep
A) Gastrin

pr
B) Pepsin
C) Cholecystokinin st
te
D) Mucus
ng

9. A patient in the ICU vomits repeatedly. The contents of his emesis are brownish-black.
What should the nurse suspect based on this finding?
si

A) Gastrointestinal hemorrhaging
ur

B) Bile escaped through the pylorus and mixed with stomach acid
yn

C) Bile escaped through the pylorus but did not have time to mix with stomach acid
.m

D) Deficit of bile in the small intestine


10. A patient in the ICU has been experiencing constipation, with his feces being small, dry,
w

and difficult to pass. What would be the most appropriate nursing intervention in this
w

case?
w

A) Recommend more protein in his diet.


B) Recommend more vegetable fiber in his diet.
C) Recommend more fat in his diet.
D) Recommend that the patient increase his caloric intake.
11. A patient has received large doses of a sympathomimetic drug, reducing saliva
production. What effect will this situation have on digestion?
A) Swallowing will be more difficult
B) Fat digestion will be impaired
C) Microbial overgrowth may occur
D) Mouth pH may decrease
12. A patient is receiving a histamine-2 receptor antagonist agent to reduce the amount of
hydrochloric acid in the stomach. What gastric function will this action reduce?
A) Fat digestion
B) Intrinsic factor production

om
C) Protein digestion
D) Stomach pH

.c
13. Impairment of digestion could occur in what structures? Select all that apply.

ep
A) Mouth
B) Esophagus

pr
C) Stomach
D) Duodenum
st
te
E) Jejunum
ng

F) Colon
si

14. In a patient with pyloric stenosis, what symptoms does the nurse expect?
ur

A) Constant hunger
yn

B) Lower abdominal cramping


C) Watery diarrhea
.m

D) Gastric reflux
w

15. A patient is having repeated episodes of vomiting. During emesis, what is the highest
w

nursing priority?
w

A) Administer replacement oral fluids.


B) Follow episode with oral care.
C) Maintain supine position.
D) Protect patients airway.
16. A patient has a gallstone that obstructs the ampulla of Vater. What impairment of
digestion will this situation result in?
A) Carbohydrates
B) Pepsin conversion
C) Gastrin production
D) Vitamin B12 deficiency
17. A patient has a gallstone, preventing the excretion of bile into the duodenum. What effect
on absorption of nutrients will this situation have?
A) Improve protein absorption
B) Reduce fat emulsion

om
C) Increase use of pepsin
D) Improve vitamin absorption

.c
18. In a patient who has had his gallbladder removed, what digestive substance will be less
concentrated as a result of this surgery?

ep
A) Gastrin

pr
B) Trypsin
C) Pepsin st
te
D) Bile
ng

19. Due to ulceration, a patient has had most of the small intestine removed. What effect on
general health does the nurse most anticipate?
si

A) Increased fluid loss


ur

B) Decreased vitamin absorption


yn

C) Improved protein digestion


.m

D) Increased antibody production


20. In a patient with a transected lower spinal cord, what reflex remains intact?
w

A) Voluntary defecation inhibition


w

B) Brachioradialis
w

C) Defecation reflex
D) Hormone secretion

Answer Key

1. A
2. B
3. A, B, D, E, F
4. A
5. A
6. A
7. D
8. C
9. A

om
10. B
11. B

.c
12. C

ep
13. A, B, C, D, E

pr
14. D
15. D st
te
16. A
ng

17. B
18. D
si

19. B
ur

20. C
yn

Chapter 39 Patient Assessment: Gastrointestinal System


.m

1. A patient with severe ascites is scheduled for a paracentesis. Before the paracentesis,
w

what is the most important nursing intervention?


w

A) Assessing electrolyte and serum protein values


w

B) Administering colon preparation medication


C) Ensuring that the patients bladder is empty
D) Shaving the patients abdomen
2. A nurse is assisting a patient with Crohns disease who has just undergone surgery to
remove part of her small intestine. The nurse notices blood in the stool of the patient.
Which of the following should be included as part of the nurses responsibilities in caring
for this patient? Select all that apply.
A) Determining the origin of blood in the stool
B) Performing emergency surgical repair of the small intestine
C) Monitoring the patients blood pressure
D) Administering parenteral nutrition for the patient
3. The nurse is gathering information regarding the health history of a patient recently
admitted to the critical care unit. Which pieces of information would the nurse need to
update continually during this patients stay in the hospital? Select all that apply.
A) Presence of bleeding
B) Preexisting gastrointestinal conditions

om
C) Nutritional status
D) Previous abdominal surgeries

.c
E) Location of pain

ep
F) Family history of gastrointestinal conditions

pr
4. A nurse is performing an oral examination of a patient. Which of the following should
she do as part of her examination?
st
te
A) Palpate the temporomandibular joint for mobility.
B) Leave the patients dentures in during the oral examination.
ng

C) Dim the lights to provide a calm environment for the patient.


si

D) Recline the patient to the supine position.


ur

5. The nurse carefully assembles at the bedside the tools she needs for the oral examination
yn

of a patient who is on a ventilator. She then performs a thorough examination of the


patients oral cavity, in compliance with her facilitys protocol. What is the best rationale
for this nurses approach to examining the patients oral cavity?
.m

A) Prevention of cavities
w

B) Elimination of halitosis
w

C) Reduction in risk for ventilator-associated pneumonia


w

D) Determination for the need for orthodontics


6. A nurse is palpating a patients abdomen as part of the physical examination. The patient
is suspected of having acute appendicitis. Which of the following would be an example of
referred pain caused by the appendicitis?
A) Rebound tenderness over the hypogastrium
B) Dull pain from below the surface of the skin
C) Pain on the surface of the skin
D) Pain in the epigastric region of the abdomen, with no other apparent cause
7. The nurse is examining the oral cavity of a patient. Which of the following findings
should she be most concerned about?
A) A pool of saliva under the tongue
B) Symmetrical rising of soft palate and uvula when patient says ah
C) Fruity breath odor
D) Strong gag reflex

om
8. A patient in the intensive care unit has abdominal cramping and is suspected of having a
small intestine obstruction. Which finding on auscultation of the abdomen would tend to
confirm this suspicion?

.c
A) High-pitched tinkling and rushing sounds

ep
B) High-pitched, rapid, and loud sounds

pr
C) Soft bubbling sounds with clicks and gurgles
D) Absent bowel sounds
st
te
9. A nurse is examining the anus and rectum of a patient. Which of the following findings
would be of greatest concern to the nurse?
ng

A) The skin around the anus being darker than the surrounding area
si

B) The finding of fecal impaction


ur

C) Even pressure being exerted by the anal sphincter


yn

D) Rectum walls being smooth, even, and uninterrupted


10. A child with an undiagnosed gastrointestinal condition is malnourished. The nurse is
.m

monitoring and adjusting his parenteral nutrition as needed. What single evaluation is the
most important for the nurse to perform to track this patients nutritional status?
w

A) Height measurement
w

B) Serial weight measurement


w

C) Albumin level
D) Visual inspection of skin
11. A patients lab results are as follows: increased urobilingogen, albumin of 20 g/L
(decreased from normal), globulin of 39 g/L (increased from normal), and transferrin of
198 mcg/dL (decreased from normal). Which gastrointestinal disorder do these results
most strongly indicate?
A) Cirrhosis
B) Acute pancreatitis
C) Ulcerative colitis
D) Escherichia coli infection
12. The nurse is caring for a critically ill patient and completes a gastrointestinal assessment
every 4 hours. What is the best rationale for this nursing action?
A) Part of routine care for all critically ill patients
B) Ordered by the physician for this patient

om
C) Satisfies the nurses curiosity
D) Early identification of changes guides treatment decisions

.c
13. A patient is admitted for evaluation of right lower quadrant abdominal pain. The nurse
hypothesizes that the pain may be related to disease of what?

ep
A) Sigmoid colon

pr
B) Duodenum
C) Appendix st
te
D) Pancreas
ng

14. A patient with multiple trauma, including blunt trauma to the abdomen, is admitted to the
critical care unit. The patient has unstable vital signs and is scheduled for immediate
si

surgery. While completing the admission health history, what aspect would the nurse be
most likely to defer until after the patient is stabilized?
ur

A) Allergies
yn

B) Diet preferences
.m

C) Quality of abdominal pain


D) Location of abdominal pain
w
w

15. While completing an admission examination on a critically ill patient, the nurse examines
the patients oral cavity. What is the best rationale for this examination?
w

A) Oral lesions can prevent adequate nutrition


B) Part of routine admission assessment
C) Appropriate only if the patient has facial trauma
D) Can be done quickly
16. The nurse is assessing a critically ill patients abdomen. What is the most appropriate
order of this examination? Rank the answers from first to last.
A) Palpate over painful areas.

B) Inspect for bruising.

C) Auscultate bowel sounds.

D) Percuss border of liver.


17. A patient is admitted after a motor vehicle crash resulting in blunt trauma to the abdomen.
On initial examination, the nurse notices that the abdomen is bruised and slightly

om
rounded, and the patient complains of abdominal pain. What is the most appropriate
nursing action?

.c
A) Auscultate for bowel and vascular sounds.

ep
B) Measure abdominal girth at umbilicus.
C) Reassess vital signs immediately.

pr
D) Elevate the head of the bed to 15 degrees.
st
18. During examination of the abdomen, the nurse identifies an area of pain to light palpation
te
in the right lower quadrant. What is the most appropriate next action?
A) Deep palpation of the area
ng

B) Administering pain medication


si

C) Assessing for rebound tenderness


ur

D) Auscultating for vascular sounds


yn

19. Laboratory test results in a critically ill patient include low prealbumin, low albumin, low
total lymphocyte count, normal neutrophils, low sodium and chloride, and low
.m

hemoglobin and hematocrit. The patient has gained 10 pounds in the past 3 days. Prior to
the weight gain, the patient was slightly underweight for height. Dietary intake before
w

admission is not known. What is the best interpretation of this information?


w

A) Blood loss
w

B) Infection
C) Malnutrition
D) Fluid overload
20. In a patient with severe parenchymal liver dysfunction, what laboratory test result does
the nurse expect will be elevated?
A) Serum albumin
B) Transferrin
C) Ammonia
D) Cholesterol

Answer Key

1. C
2. A, C, D
3. A, C, E

om
4. A
5. C

.c
6. D

ep
7. C

pr
8. A
9. B st
te
10. B
ng

11. A
12. D
si

13. C
ur

14. B
yn

15. A
.m

16. B, C, D, A
17. B
w
w

18. C
w

19. C
20. C

Chapter 40 Patient Management: Gastrointestinal System

1. A young refugee from Uganda who recently arrived in the United States has been
admitted to the critical care unit for treatment of severe infection of a sore on his leg.
What nutrient should the nurse realize will be most deficient in this patient?
A) Protein
B) Lipids
C) Carbohydrates
D) Minerals
2. A patient requires a nasogastric tube for enteral feeding. The nurse selects a small-caliber
feeding tube for the patient. What are the advantages of using a small-caliber tube? Select
all that apply.
A) Can be used to decompress the stomach
B) Allows monitoring of gastric pH

om
C) Is more comfortable for the patient
D) Is capable of delivering medications and feedings

.c
E) Has lower risk of self-extubation

ep
3. A patient in the critical care unit is on a ventilator and requires nutritional support, likely

pr
for 3 weeks. This patient has a transected spinal cord at C4. Which would be the best
choice of nutritional support for this patient?
A) Nasogastric tube
st
te
B) Nasoduodenal tube
ng

C) Gastrostomy tube
si

D) Total parenteral nutrition


ur

4. A patient with gastric cancer must receive long-term nutritional support. Due to the
cancer, the patient has impaired gastric function and delayed gastric emptying, but the
yn

remainder of her gastrointestinal tract is intact and functional. Which of the following
would be the best choice of nutritional support for this patient?
.m

A) Jejunostomy tube
w

B) Nasoduodenal tube
w

C) Gastrostomy tube
w

D) Central parenteral nutrition


5. A patient with Crohns disease is receiving enteral feeding. The nurse administers an
elemental formula to the patient. What is the best rationale for this choice?
A) Lower cost
B) Pleasant taste
C) Easier to digest
D) More complete nutrition
6. A patient is receiving enteral feeding via a nasogastric tube of a polymeric formula.
Under what conditions should the nurse discontinue enteral feeding? Select all that apply.
A) Three instances of gastric residual volume of 175 mL
B) Regurgitation
C) Two instances of gastric residual volume of 600 mL
D) Vomiting
E) Aspiration

om
F) Diarrhea
7. A patient with severe hemorrhagic pancreatitis requires long-term nutritional support.
Which of the following would be the best form of nutritional support for this patient?

.c
A) Nasoduodenal tube

ep
B) Gastrostomy tube

pr
C) Total parenteral nutrition
D) Peripheral parenteral nutrition st
te
8. A patient with anorexia nervosa requires parenteral nutrition support. This patient
demonstrates signs of marasmus. What intervention should the nurse make to prevent a
ng

complication for which this patient is at increased risk?


si

A) Limiting of initial dextrose dosing


ur

B) Depletion of potassium before TPN initiation


yn

C) Proper hand hygiene


D) Change of TPN solution bag every 24 hours
.m

9. A patient who is receiving total parenteral nutrition (TPN) via a temporary catheter has
previously experienced catheter occlusion due to fibrin buildup. What substance can be
w

added to the standard parenteral formula to best help prevent this complication?
w

A) Insulin
w

B) Dextrose
C) Potassium
D) Heparin
10. A patient in the ICU is experiencing gastroesophageal reflux disease. Which medication
should the nurse administer to this patient?
A) Promethazine
B) Loperamide
C) Cimetidine
D) Misoprostol
11. A patient with a serious illness is being cared for in a CCU. Considering energy
expenditure during illness and the potential for healing, what is most appropriate nursing
action?
A) Maintain complete bed rest with minimal activity
B) Ensure complete pharmacological sedation

om
C) Limit visits to immediate family only
D) Facilitate nutrition assessment and delivery

.c
12. After a serious injury, a patient has not received nutritional supplementation for several

ep
days. What body system is most likely to experience damage first?

pr
A) Cardiac
B) Musculoskeletal
st
te
C) Neurologic
D) Integumentary
ng

13. A patient has been in the CCU with serious illness for several days. What collaborative
si

nursing action is most important to prevent immunosuppression?


ur

A) Nutritional support
yn

B) Antimicrobial administration
C) Erythropoietin administration
.m

D) Adhering to hand hygiene protocol


w

14. A patient has been admitted with severe malnutrition from prolonged calorie deficiency.
w

The patient has severe muscle wasting but normal electrolytes and serum albumin. He is
lethargic, with a flat affect. What treatment does the nurse anticipate?
w

A) Rapid infusion of high-calorie enteral feedings


B) Use of total parenteral nutrition
C) Slow balanced enteral feedings
D) Rapid infusion of high-protein feedings
15. A patient with critical illness on complete bed rest has been without oral intake for
several days. The patients general instability has prevented routine turning and
repositioning for the past 24 hours. The patients weight has increased by 10 pounds since
admission and the nurse notices evidence of skin breakdown starting. What is the most
likely cause of this breakdown?
A) Proteincalorie malnutrition
B) Hazards of immobility
C) Peripheral edema formation
D) Effects of medications
16. During a critical illness, a patient has been without enteral nutrition for several days.
What is the most significant complication this situation places the patient at increased risk

om
for?
A) Sepsis

.c
B) Pneumonia

ep
C) Fecal impaction

pr
D) Abdominal pain

st
17. A critically ill patient is being evaluated for nutritional support using the enteral route.
What contraindication for enteral feedings does the nurse assess for?
te
A) Lack of evidence of severe malnutrition
ng

B) Ability to chew and swallow


si

C) Evidence of prolonged ileus


ur

D) Presence of intravenous access


yn

18. A patient is being fed continuously through a nasogastric tube. According to unit policy,
the nurse checks tube placement regularly. What method is most accurate?
.m

A) Air insufflation and auscultation over the stomach


B) Inspection of tube aspirate for color and consistency
w
w

C) Measuring gastric pH after holding feeding 60 minutes


w

D) Obtaining a radiograph of the upper abdomen


19. A critically ill patient is receiving intravenous total parenteral nutrition (TPN). The
patient is also being supported by several constant IV vasoactive medications, is
pharmacologically sedated and paralyzed with constant IV medications, and is receiving
three antimicrobial mediations by intermittent IV infusion. The nurse ensures that the
TPN is infusing into a separate line and not mixing with any other fluid or medication.
What is the best rationale for this precaution?
A) Complies with hospital and unit policy
B) Prevents risk of contamination and precipitation
C) Contains trace amounts of electrolytes
D) Contains high fat concentrations
20. A patient is receiving total parenteral nutrition (TPN). To monitor for the development of
a complication directly related to the composition of the TPN, what intervention does the
nurse initiate?
A) Vital signs every 15 minutes
B) Strict adherence to hand hygiene protocol

om
C) Continuous capnography
D) Frequent monitoring of blood glucose

.c
ep
Answer Key

pr
1. A
2. C, E st
te
3. B
ng

4. A
si

5. C
ur

6. B, C, D, E
7. C
yn

8. A
.m

9. D
w

10. C
w

11. D
w

12. C
13. A
14. C
15. A
16. A
17. C
18. D
19. B
20. D

Chapter 41 Common Gastrointestinal Disorders

1. A 20-year-old man presents with upper gastrointestinal bleeding. His health history
indicates binge drinking on weekends. The nurse inquires about the patients alcohol use

om
and learns that the patient frequently vomits violently after drinking. Which of the
following underlying causes should the nurse suspect for this patients bleeding?
A) Mallory-Weiss tears

.c
B) Dieulafoys lesions

ep
C) Peptic ulcer disease

pr
D) Stress-related erosive syndrome
st
2. The nurse places a large-bore nasogastric tube in a patient who has acute upper
te
gastrointestinal bleeding. What is the rationale for this intervention? Select all that apply.
A) Aspiration of gastric contents
ng

B) Improvement of ventilation
si

C) Decompression
ur

D) Enteral feeding
yn

E) Lavage of gastric contents


.m

F) Control of bleeding
3. An elderly patient is experiencing lower gastrointestinal bleeding. On questioning the
w

patient, the nurse learns that she consumes little fiber in her diet, takes aspirin daily, and
w

frequently has constipation. The nurse should suspect which of the following as the most
likely cause of this patients bleeding?
w

A) Hemorrhoids
B) Crohns disease
C) Diverticulosis
D) Ulcerative colitis
4. The source of lower gastrointestinal bleeding has been found in a patient during
colonoscopy. Which of the following should the nurse expect as the next intervention?
A) Endoscopy
B) Angiography
C) Radionucleotide imaging
D) Thermal coagulation
5. A patient with small bowel obstruction (SBO) due to adhesions is no longer passing gas.
She has a fever and has shown no signs of improvement in 24 hours. There is no evidence
of stool or gas in the distal intestine on radiograph. What is the most important
intervention for this patient at this point?

om
A) Withholding oral food and fluid
B) Laparoscopic lysis of adhesions

.c
C) Placement of nasogastric tube

ep
D) Aggressive treatment with IV fluid and electrolytes

pr
6. A patient in the critical care unit presents with abdominal pain, distention, and
progressive obstipation. The patient is currently being treated for colon cancer and
st
mentions a gradual decrease in amount and size of stool passed in recent weeks. What
finding should the nurse most expect in this patient?
te
A) Marked leukocytosis
ng

B) A history of laxative use


si

C) Left lower quadrant pain


ur

D) Iron deficiency
yn

7. A patient presents with massive colonic dilation in the absence of mechanical obstruction,
with electrolyte imbalance. Which of the following would be the most appropriate initial
.m

nursing intervention for this patient?


A) Administer a laxative.
w

B) Encourage the patient to walk around his room.


w

C) Monitor the patients heart.


w

D) Prepare the patient for colonoscopy.


8. A patient is diagnosed with hepatitis B. The nurse recognizes that which of the following
is the most likely etiology for this patients illness?
A) Sexually transmitted viral infection
B) Viral infection from ingesting contaminated water
C) Excessive alcohol consumption
D) Autoimmune disorder
9. A patient is diagnosed with cirrhosis. The nurse sees that there is no indication of alcohol
use in the patients health history, and the patient denies drinking alcohol. What is another
likely cause of this disease that the nurse should suspect?
A) Peptic ulcer disease
B) Hepatitis C virus (HCV) infection
C) Dieulafoys lesions
D) Diverticulosis

om
10. A patient with esophageal variceal bleeding unresponsive to endoscopic therapy is
receiving a balloon tamponade tube. Which of the following is an appropriate nursing

.c
intervention related to this procedure?

ep
A) Warm the tube before the physician inserts it.
B) Lower the head of the bed so that the patients feet are above his heart.

pr
C) Encourage the patient to cough.
D)
st
Clean and lubricate the patients nostrils.
te
11. The patient has an upper gastrointestinal hemorrhage. What pathological basis does the
ng

nurse least expect?


A) Peptic ulcer disease
si

B) Helicobacter pylori infection


ur

C) Infectious colitis
yn

D) Esophageal varices
.m

12. A patient has an acute upper gastrointestinal hemorrhage secondary to esophageal


varices. What is the underlying pathophysiology of this type of gastrointestinal
w

hemorrhage?
w

A) Helicobacter pylori infection of esophagus, stomach, and duodenum


w

B) Prolonged stress from multiple causes


C) Portal hypertension from cirrhosis or other liver disease
D) Overwhelming infection of jejunum and ileum
13. An elderly patient with several comorbidities has experienced a massive upper GI
hemorrhage from esophageal varices. No treatment has been initiated yet. What signs and
symptoms does the nurse expect?
A) Tachycardia and hypotension
B) Alert and oriented times 3
C) Mild anemia without hypoxemia
D) Normal peripheral circulation
14. A patient with a massive upper gastrointestinal hemorrhage is exhibiting signs and
symptoms of hypovolemic shock. What initial treatment does the nurse anticipate?
A) Admission to a general care nursing unit for monitoring
B) Initial fluid resuscitation with intravenous crystalloids

om
C) Diet of clear liquids, advanced as tolerated
D) Bed rest in semi-Fowlers position

.c
15. Intravenous vasopressin (Pitressin) has been ordered for a patient with recurrent
esophageal varices causing upper gastrointestinal hemorrhage. What is the purpose of this

ep
therapy?

pr
A) Increases systemic blood pressure in shock
B)
st
Increases myocardial oxygen use and demand
te
C) Reduces blood flow to the mesenteric circulation
D) Causes constriction of the splanchnic arteries
ng

16. In a patient with a complete small bowel obstruction, what symptoms would the nurse be
si

most likely to find?


ur

A) Absence of bowel sounds


yn

B) Acute, intermittent abdominal cramps


C) Pain increased with emesis
.m

D) No evidence of hypovolemia
w

17. In a patient with acute pancreatitis, what elevation in laboratory study results would the
w

nurse expect?
w

A) Serum potassium and calcium


B) Arterial ionized calcium
C) Serum amylase and lipase
D) Arterial partial pressure of oxygen
18. What is the underlying pathophysiologic process for pancreatitis?
A) Inflammation from bowel endotoxins
B) Autodigestion by pancreatic enzymes
C) Inability of the pancreas to activate its enzymes
D) Destruction of pancreatic tissue by bile
19. A patient with acute pancreatitis is being cared for in intensive care. After several days of
therapy, the patient has worsening hypoxemia that does not respond to oxygen therapy,
lung sounds are diminished, and there is a patchy infiltrate on the chest x-ray. The patient
also has a fever, abdominal distention, and severe pain despite narcotic therapy. What
complication of pancreatitis does the nurse most suspect?
A) Acute respiratory distress syndrome (ARDS)

om
B) Nonspecific arterial hypoxemia
C) Compression of lung by abdominal fluid

.c
D) Hypoventilation secondary to severe pain

ep
20. A patient with pancreatitis is complaining of pain at a level of 10 on a scale of 0 to 10.
The doctor has ordered IV fentanyl citrate (Sublimaze) every 4 hours prn for pain and

pr
lorazepam (Ativan) every hour prn for agitation. The last dose was given 3 hours ago.
The patients respiratory rate is 22 per minute with full depth. What is the best nursing
action? st
te
A) Obtain an order for meperidine (Demerol) to prevent sphincter of Oddi spasm.
ng

B) Administer IV fentanyl now, as the patient is in pain.


C) Administer IV lorazepam for agitation.
si

D) Consult the physician for an increase in fentanyl frequency.


ur

21. Immune serum globulin has been ordered for a patient with acute hepatitis A. What does
yn

the nurse emphasize when explaining the purpose of this medication to the patient?
A) The medication will eradicate the virus.
.m

B) The medication will significantly ameliorate the symptoms.


w

C) This is the vaccine for hepatitis A.


w

D) This medication will obviate any need for isolation.


w

22. A patient has been diagnosed with hepatitis B virus. While teaching the patient and
family, what mode of transmission will the nurse be least likely to include?
A) Blood
B) Semen
C) Saliva
D) Feces
23. The patient has been diagnosed with hepatitis C virus (HCV). At present, what is the
primary risk factor for HCV transmission?
A) Lack of hand hygiene by food industry workers
B) Multiple sexual contacts with either gender
C) Contaminated needlestick exposure
D) Contaminated blood transfusions
24. In a patient with severe hepatic cirrhosis, what laboratory findings would the nurse expect
to be decreased?

om
A) Serum proteins
B) Serum amylase

.c
C) Aspartate aminotransferase (AST)

ep
D) Prothrombin time (PT)

pr
25. A patient with severe hepatic cirrhosis has developed severe ascites. What is the most
important nursing assessment?
st
te
A) Abdominal girth
B) Intake and output
ng

C) Respiratory status
si

D) Nutritional intake
ur

26. A patient with severe ascites is being treated with spironolactone (Aldactone) and
yn

furosemide (Lasix). What laboratory test result indicates a complication from this
therapy?
.m

A) Prolonged prothrombin time (PT)


B) Leukocytosis
w
w

C) Thrombocytopenia
w

D) Hypokalemia
27. A critically ill patient with cirrhosis has developed hepatic encephalopathy. What is the
most effective therapeutic management strategy?
A) Surgical portosystemic shunt
B) Increased nutritional protein
C) Lactulose or neomycin enterally
D) Sedation with midazolam (Versed)
Answer Key

1. A
2. A, C, E
3. C
4. D
5. B

om
6. D
7. B

.c
8. A

ep
9. B
10. D

pr
11. C
12. C
st
te
13. A
ng

14. B
si

15. D
ur

16. B
yn

17. C
18. B
.m

19. A
w

20. D
w

21. B
w

22. D
23. C
24. A
25. C
26. D
27. C
Chapter 42 Anatomy and Physiology of the Endocrine System

1. A patient has just suffered a serious traumatic injury, resulting in the release of high
levels of endogenous catecholamines. As part of the admission laboratory assessment, the
nurse notices a high serum glucose level. What is the most appropriate nursing
assessment?
A) Assume the patient is diabetic.
B) Restrict caloric intake.
C) Reassess glucose levels.

om
D) Assume the result is a laboratory error.
2. Patients being cared for in ICUs secrete increased amounts of glucocorticoids in response

.c
to stress. What laboratory test result is most indicative of the development of a life-
threatening complication secondary to the increased secretion of glucocorticoids?

ep
A) Increased blood glucose

pr
B) Increased neutrophils
C) Decreased lymphocytes st
te
D) Decreased serum myoglobin
ng

3. A patient is found to have a vitamin D deficiency, putting her at risk for bone
demineralization. What factors would contribute to this deficiency? Select all that apply.
si

A) Lack of exposure to sunlight


ur

B) Lack of exercise
yn

C) Tumor of the posterior pituitary


.m

D) Dietary deficiency
E) Insulin resistance
w

F) Presence of 7-dehydrocholesterol provitamins in the skin


w

4. A patient has a plasma calcium level of 10.5 mg/dL due to excessive intake of vitamin
w

Denriched foods. In response, the nurse should expect to see which of the following?
A) Decreased parathyroid hormone
B) Vitamin C deficiency
C) Increased serum phosphate
D) Increased serum calcitonin
5. A patient with poorly controlled type 2 diabetes demonstrates high levels of serum C-
peptide. What other findings should the nurse expect in this patient? Select all that apply.
A) High levels of serum insulin
B) High levels of blood glucose
C) Decreased pancreatic activity
D) Low body weight
6. A patient with stable type 1 diabetes is in the ICU. He has been taking insulin injections
regularly. The nurse receives blood test results back for this patient. What should she
most expect to see in the test results?
A) Elevated blood glucose

om
B) Low C-peptide
C) Elevated cholesterol

.c
D) Decreased blood pH

ep
7. A patient in the ICU has been under tremendous stress following myocardial infarction.

pr
She has no history of diabetes, but her blood glucose level is elevated. Increase in the
serum level of which substance would most likely explain the elevated blood glucose
level in this patient? st
te
A) Insulin
ng

B) Cortisol
C) C-peptide
si

D) Calcium
ur

8. A patient has been prescribed an incretin. Which of the following should the nurse expect
yn

to seen in the patient as a result of taking this medication?


A) Increased blood glucose
.m

B) Delayed gastric emptying


w

C) Increased appetite
w

D) Dehydration
w

9. A patient just underwent a kidney transplant and is receiving a glucocorticoid to suppress


his immune system to avoid organ rejection. Which side effect should the nurse most
expect to find in this patient?
A) Hives
B) Asthma
C) Minimal-change glomerular disease
D) Cushings syndrome
10. A patient has been diagnosed with a tumor of the hypothalamus gland and demonstrates
sustained elevated corticotropin-releasing hormone (CRH) levels. What should the nurse
most expect to see in this patient?
A) Abnormal glucocorticoid secretion pattern over the course of 24 hours
B) Decreased adrenocorticotropic hormone (ACTH) levels
C) Absence of glucocorticoid secretion
D) Hypoglycemia
11. A patient in the ICU has cardiac failure and resultant low renal perfusion, which triggers

om
the renin-angiotensin mechanism. Which of the following should the nurse expect to
happen as a result of this mechanism?

.c
A) Elevation of systemic blood pressure

ep
B) Decreased aldosterone secretion
C) Dehydration

pr
D) Dilation of vascular smooth muscle
st
12. A patient with hypertension is receiving medication that disrupts the renin-angiotensin
te
mechanism and thus lowers his systemic blood pressure. Which of the following is most
likely the medication he is on?
ng

A) Propranolol
si

B) Insulin
ur

C) Glucocorticoid
yn

D) Somatostatin
13. Due to an interruption in the communication between the hypothalamus and the posterior
.m

pituitary gland, a patient has a deficit of antidiuretic hormone (ADH). What physical
symptom does the nurse expect?
w

A) Very dilute urine with high volume


w

B) Very concentrated urine with low volume


w

C) Very dilute urine with low volume


D) Very concentrated urine with high volume
14. An adult has experienced damage to the anterior pituitary gland that has interfered with
the secretion of growth hormone. The nurse expects what adverse effect on the patient as
a direct result?
A) None, since the patient is an adult
B) Increased risk for anemia
C) Increased sensitivity to insulin
D) Decreased cellular repair
15. In a patient with hyperthyroidism, what symptoms does the nurse most expect?
A) Lethargy and hair loss
B) Tachycardia and muscle weakness
C) Hypotension and weight gain

om
D) Increased serum lipids
16. A patient has a parathyroid tumor causing oversecretion of parathyroid hormone. What

.c
intervention does the nurse implement to prevent a complication of this oversecretion?

ep
A) Fall precautions
B) Frequent vital signs

pr
C) Weight-bearing exercise
D) Water-soluble vitamins
st
te
17. What is the main aspect of diabetes mellitus type 2 that is not found in type 1?
ng

A) Increased secretion of insulin by the pancreas


si

B) Increased insulin receptors on most cells


ur

C) Need for exogenous insulin therapy


yn

D) Inability to self-regulate glucose levels


18. A patient is receiving a glucocorticoid to reduce inflammation and has a persistently high
.m

blood glucose level. What action is this glucose level most likely due to?
w

A) Glycogen
w

B) Glucagon
w

C) Insulin resistance
D) Intravenous amino acids
19. Patients being cared for in critical care units secrete increased amounts of glucocorticoids
in response to stress. What laboratory test result is most indicative of the development of
a life-threatening complication secondary to the increased secretion of glucocorticoids?
A) Increased blood glucose
B) Increased eosinophils
C) Decreased lymphocytes
D) Decreased serum myoglobin
20. A patient is in hypovolemic shock but has a normal blood pressure. The nurse
understands that part of the rationale for this effect is activation of the renin-angiotensin
system and the subsequent secretion of aldosterone. What physiologic effect does the
activation of aldosterone result in?
A) Inhibition of vasoconstriction
B) Reabsorption of sodium and water

om
C) Promotion of central vasodilation
D) Inhibition of peripheral vasoconstriction

.c
21. A patient is receiving a glucocorticoid medication for treatment of severe allergic

ep
inflammation. What is the most significant effect of increased glucocorticoid levels that
the nurse teaches the patient?

pr
A) Water retention
B) Sodium retention st
te
C) Immune suppression
ng

D) Complement activation
si

Answer Key
ur

1. C
yn

2. C
.m

3. A, D
w

4. D
w

5. A, B
w

6. B
7. B
8. B
9. D
10. A
11. A
12. A
13. A
14. B
15. B
16. A
17. A
18. B

om
19. C
20. B

.c
21. C

ep
Chapter 43 Patient Assessment: Endocrine System

pr
1. Pituitary hormones are under the control of which of the following?
A) Brainstem
st
te
B) Feedback loops
ng

C) Autonomic nervous system


si

D) Hypothalamus
ur

2. Which hormone is secreted by the posterior pituitary gland?


yn

A) TSH
B) ACTH
.m

C) Cortisol
w

D) ADH
w

3. Increased levels of ADH, low serum osmolarity, and increased urine osmolarity are a sign
w

of which of the following?


A) Dehydration
B) Diabetes mellitus (DM)
C) Diabetes insipidus (DI)
D) Syndrome of inappropriate ADH secretion (SIADH)
4. Which of the following patients is most likely to experience hypothyroidism?
A) A 38-year-old female, admitted to the ICU for burns
B) A 68-year-old male, admitted to the orthopedic floor for a broken leg
C) A 12-year-old male, admitted to pediatrics post-tonsillectomy
D) A 25-year-old female, admitted to gynecology for a threatened abortion
5. Thyroid malignancy can be detected through a needle biopsy. Which test below is the
best method for detecting masses, nodules, and cysts in the thyroid gland?
A) Arteriogram

om
B) PET scan
C) Ultrasound

.c
D) MRI

ep
6. A patient tested positive for hyperparathyroidism. Which of these serum levels would you
expect to be elevated?

pr
A) Free T4 (thyroxine)
B) TSH (thyroid stimulating hormone) st
te
C) ACTH (adrenocorticotropic hormone)
ng

D) Calcium
7. A 30-year-old patient was admitted to the hospital for complaints of sudden weight gain,
si

insomnia, and uncontrolled diaphoresis. Her latest tests show increased serum cortisol
ur

levels, as well as increased free cortisol levels in her 24-hour urine. Her CT scan showed
a possible pituitary adenoma. Which of the following is the most likely diagnosis for this
yn

patient?
A) Cushings disease
.m

B) Addisons disease
w

C) Type 2 diabetes
w

D) Diabetes insipidus
w

8. A provider orders a cortisol stimulation test for a patient suspected of having an


endocrine disorder. Which of the following disorders does the provider suspect?
A) Hepatitis
B) Cushings syndrome
C) Addisons disease
D) Primary hyperparathyroidism
9. Which of the following are hormones that are produced by the adrenal cortex? Select all
that apply.
A) Aldosterone
B) Epinephrine
C) Cortisol
D) Dopamine
E) ACTH

om
10. A 45-year-old patient is experiencing an acute rise in blood pressure due to a suspected
pheochromocytoma. Which of the following is the best test to diagnose this disease?
A) Total plasma catecholamine concentration

.c
B) Plasma A1C

ep
C) Fasting blood glucose

pr
D) Cortisol level
st
11. A patient has been diagnosed with syndrome of inappropriate antidiuretic hormone
te
(SIADH). What physical symptoms would the nurse expect?
A) Tachycardia and hypotension
ng

B) History of weight loss


si

C) Edema and hypertension


ur

D) Dry skin and mucosa


yn

12. A patient with a head injury has developed central diabetes insipidus. What elevated
laboratory test result would the nurse expect?
.m

A) Serum antidiuretic hormone (ADH)


w

B) Urine osmolality
w

C) Urine sodium level


w

D) Serum osmolality
13. In a patient with trauma-induced central diabetes insipidus, what urine specific gravity
result does the nurse expect?
A) Specific gravity 1.001 to 1.010
B) Specific gravity 1.011 to 1.024
C) Specific gravity 1.025 to 1.030
D) Specific gravity varies diurnally
14. A patient is complaining of extreme thirst and is drinking water constantly. He also
excretes large volumes of dilute-appearing urine and is being evaluated for psychogenic
water drinking versus central diabetes insipidus. If the patient has diabetes insipidus, what
pattern of laboratory results would the nurse expect?
A) High urine osmolality, low serum osmolality, and high urine specific gravity
B) High urine osmolality, normal serum osmolality, and high urine specific gravity
C) Low urine osmolality, normal serum osmolality, and low urine specific gravity
D) Low urine osmolality, high serum osmolality, and low urine specific gravity

om
15. A patient has critically low thyroid levels. What symptoms does the nurse expect?
A) Reduced deep tendon reflexes

.c
B) Uncontrolled hypertension and hyperthermia

ep
C) Tachycardia, tachypnea, and hyperventilation

pr
D) Rapid, agitated speech and anxious look

st
16. In a patient with primary hypothyroidism, what pattern of laboratory results would the
nurse expect?
te
A) High thyroid-stimulating hormone (TSH) and low free thyroxine (T4)
ng

B) High thyroid-stimulating hormone (TSH) and high free thyroxine (T4)


si

C) Low thyroid-stimulating hormone (TSH) and low free thyroxine (T4)


ur

D) Low thyroid-stimulating hormone (TSH) and high free thyroxine (T4)


yn

17. A patient has had a complete thyroid gland resection that included the parathyroid glands.
What nursing assessment indicates a significant complication of this procedure?
.m

A) Positive Trousseaus sign


B) Positive Kernigs sign
w
w

C) Negative Babinski reflex


w

D) Negative dolls eyes reflex


18. The patient has been admitted to the critical care unit after a traumatic incident and is
unresponsive. No family members are present. The nurse finds a card in the patients
wallet indicating that the patient has diabetes mellitus, but it does not state the type. If the
patient develops diabetic ketoacidosis, what unique symptom would the nurse expect?
A) Elevated blood glucose level
B) Pulmonary excretion of acetone
C) Mental confusion or coma
D) Polydipsia and polyuria
19. In a patient with a pheochromocytoma, what symptoms would the nurse expect?
A) Bradycardia and bradypnea
B) Hypotension and warm extremities
C) Severe headache and diaphoresis
D) Increased urine output and diarrhea
20. In a patient with stress-induced adrenal insufficiency, what physical symptoms does the

om
nurse expect?
A) Constant bradycardia

.c
B) Lethargy and apathy

ep
C) Acute severe pain
D) Edema and fluid overload

pr
Answer Key st
te
1. D
ng

2. D
si

3. D
ur

4. A
yn

5. C
.m

6. D
7. A
w

8. C
w

9. A, C
w

10. A
11. C
12. D
13. A
14. D
15. A
16. A
17. A
18. B
19. C
20. B

Chapter 44 Common Endocrine Disorders

om
1. Hyponatremia without edema or hypovolemia is a sign of water intoxication caused by
which of the following?

.c
A) Syndrome of inappropriate ADH secretion (SIADH)

ep
B) Diabetes insipidus (DI)
C) Newly diagnosed type 1 diabetes

pr
D) Prerenal kidney failure
st
2. Patients experiencing DKA (diabetic ketoacidosis) demonstrate a much higher level of
te
ketoacidosis than patients experiencing HHS (hyperosmolar hyperglycemic state). Why
does this occur?
ng

A) Patients with HHS produce just enough insulin to prevent ketosis.


si

B) Only patients with Type 1 diabetes experience ketoacidosis.


ur

C) HHS develops too quickly to build up a high level of ketoacidosis.


yn

D) The fluid loss in HHS is too mild to allow a high level of ketoacidosis.
3. Which of the following are precipitating factors for thyrotoxic crisis? Select all that
.m

apply.
w

A) Infection
w

B) Hypothermia
w

C) Steroid therapy
D) Long, chronic illness
E) Pregnancy
4. Which of the following descriptive words are indicative of myxedema coma?
A) Rare, hypothyroid
B) Chronic, hypothyroid
C) Common, hyperthyroid
D) Rare, hyperthyroid
5. Sodium and osmolarity levels of both plasma and urine are helpful in the diagnosis of
endocrine disorders. When serum sodium and serum osmolarity values are low while
urine sodium and urine osmolarity are high, what is the most likely diagnosis?
A) SIADH
B) Diabetes insipidus
C) Hypoglycemia
D) Water intoxication

om
6. Most endocrine disorders are not diagnosed until patients are acutely ill with a second
condition. What are some common precipitating illnesses these patients may have?

.c
A) Severe illness/stress activating the HPA axis

ep
B) High glucose levels from gluconeogenesis

pr
C) Low cortisol levels from depleting the flight or fight response
D)
st
Decreased basal metabolic rate from prolonged hyperthermia
te
7. A nursing plan of care for a patient in diabetic ketoacidosis (DKA) includes altered tissue
perfusion. Why is this an appropriate nursing diagnosis?
ng

A) Decreased extracellular fluid volume leads to decreased tissue perfusion through


anaerobic metabolism.
si

B) Kussmauls respirations cause panting and water loss through evaporation.


ur

C) Ketoacidosis causes respiratory acidosis


yn

D) Low blood pH shifts the oxyhemoglobin dissociation curve to the left, causing poor
tissue oxygenation
.m

8. A nursing plan of care for a patient who is hypoglycemic includes which of the
w

following?
w

A) Monitoring neurologic status changes


w

B) Monitoring temperature status changes


C) Monitoring cardiac status changes
D) Monitoring blood alcohol levels
9. A patient in HHS (hyperosmolar hyperglycemic state) has a serum glucose of 900, serum
osmolarity of 400, and an anion gap of 8. Treatment for this condition includes which of
the following?
A) Administer intravenous normal saline as quickly as possible.
B) Correct the fluid deficit with intravenous normal saline at a moderate rate.
C) Administer a high-dose insulin drip.
D) Administer sodium bicarbonate to correct the acidosis.
10. A patient was resuscitated from a cardiopulmonary arrest and is now being managed with
mechanical ventilation, an intravenous amiodarone drip, an angiotensin-converting
enzyme (ACE) inhibitor, and a beta-adrenergic antagonist. The patient has severe
tachycardia, tachypnea, diaphoresis, temperature above 104F, frequent premature
ventricular contractions, clear lung fields, normal oxygen status, and agitation and
restlessness. What complication and cause does the nurse suspect?

om
A) Septic shock from ventilator-acquired pneumonia
B) Hypovolemia from cardiopulmonary arrest

.c
C) Thyrotoxic crisis from amiodarone use

ep
D) Adverse reaction to ACE inhibitor

pr
11. A patient has been diagnosed with Graves disease. What physical symptoms would the
nurse most expect to find?
A) Bradycardia
st
te
B) Diminished bowel sounds
ng

C) Alert and oriented times 3


si

D) Exophthalmos
ur

12. The patient has been diagnosed with hyperthyroidism. What laboratory result would the
nurse least expect to be elevated?
yn

A) Total thyroxine (T4)


.m

B) Free triiodothyronine (T3)


w

C) Free thyroxine (T4)


w

D) Thyroid stimulating hormone (TSH)


w

13. A patient with hyperthyroidism is being treated with propylthiouracil and potassium
iodide. When giving these medications, what precaution should the nurse take?
A) Check apical pulse before administration.
B) Check serum potassium before administration.
C) Administer medications at least 1 hour apart.
D) Give medications intravenously.
14. A patient with severe thyrotoxicosis has a very high temperature. What medication or
treatment is least appropriate to suppress the temperature?
A) Acetylsalicylic acid (aspirin)
B) Acetaminophen (Tylenol)
C) Cooling blanket
D) Intravascular cooling system
15. A patient has been admitted to the critical care unit in myxedema coma. What patient
description does the nurse most expect?

om
A) Young man with abdominal trauma
B) Middle-aged man with skeletal trauma

.c
C) Middle-aged woman in summer

ep
D) Elderly woman during winter

pr
16. A patient has been diagnosed with primary hypoadrenalism (Addisons disease). What
usually destroys the adrenal gland tissues?
st
te
A) Mycobacterium tuberculosis
B) Septic shock hemorrhage
ng

C) Autoimmune antibodies
si

D) Metastatic malignancies
ur

17. A patient has symptomatic adrenal insufficiency. What symptoms does the nurse most
yn

expect to find?
A) Weakness and fatigue
.m

B) Constipation and increased appetite


w

C) Bradycardia and bradypnea


w

D) Weight gain and hypertension


w

18. A patient is being evaluated for possible syndrome of inappropriate antidiuretic hormone
(SIADH). What laboratory abnormality would the nurse most expect to find?
A) Serum hyponatremia
B) Serum hypokalemia
C) Urine hyponatremia
D) Urine hypo-osmolarity
19. A patient has developed diabetes insipidus after suffering severe head trauma. What
symptoms does the nurse expect to find?
A) Urine output less than 30 mL/hr
B) Urine specific gravity 1.001 to 1.005
C) Serum osmolality below 275 mOsm/kg
D) Serum sodium less than 135 mEq/L
20. A patient in intensive care after a severe head injury has developed diabetes insipidus.
What nursing assessment is the most important?

om
A) Vital signs
B) Cardiac pattern

.c
C) Fluid balance

ep
D) Level of consciousness

pr
21. For a patient with severe diabetic ketoacidosis, what treatment does the nurse expect to
initiate?
st
te
A) Large-volume intravenous crystalloid infusion
B) Discontinuation of exogenous insulin
ng

C) Restriction of nutritional protein and fat


si

D) Intravenous diuretic therapy


ur

22. A patient in diabetic ketoacidosis (DKA) is to receive insulin to control blood sugar.
yn

What type of insulin does the nurse expect to administer?


A) Intravenous drip regular
.m

B) Subcutaneous regular
w

C) Intravenous Lantis
w

D) Subcutaneous Lantis
w

23. A patient with type 1 diabetes mellitus is being discharged home after an episode of
diabetic ketoacidosis (DKA). What should the nurse most emphasize in discharge
teaching?
A) Insulin therapy may be discontinued after several months.
B) The patient needs supplemental insulin even when not eating.
C) Blood sugar is best regulated by use of long-lasting insulin preparations.
D) Blood sugar can be regulated by diet if the patient is very careful.
24. A 40-year-old insulin-dependent diabetic is becoming increasingly stuporous. He is pale,
diaphoretic, and tachycardic. What is the most important strategy the nurse expects to be
included in the initial treatment?
A) Low-dose insulin infusion
B) Bolus of high-dose insulin
C) Bolus of 50% dextrose
D) Fluid replacement
25. A middle-aged patient with diabetes mellitus type 2 is being treated for hyperosmolar

om
hyperglycemic syndrome, which developed from an infection. During discharge teaching,
the nurse emphasizes what concept?

.c
A) Reduce use of insulin when ill.

ep
B) Exercise moderately.
C) Increase frequency of glucose testing when ill.

pr
D) Increase amount of carbohydrates in diet.
st
te
Answer Key
ng

1. A
si

2. A
ur

3. A, B, C, D, E
yn

4. A
5. A
.m

6. A
w

7. A
w

8. A
w

9. B
10. C
11. D
12. D
13. C
14. A
15. D
16. C
17. A
18. A
19. B
20. C
21. A

om
22. A
23. B

.c
24. C

ep
25. C

pr
Chapter 45 Anatomy and Physiology of the Hematological and Immune Systems
st
1. Normal red blood cell development requires specific nutrients for formation. Which of
te
the following aid red blood cell development? Select all that apply.
ng

A) Iron
si

B) Vitamin B12
ur

C) Folic acid
D) Vitamin K
yn

2. Describe the main difference between granulocytes and agranulocytes.


.m

A) Granulocytes are white blood cells, whereas agranulocytes are platelets.


w

B) Granulocytes are white blood cells, whereas agranulocytes are immature white
blood cells.
w

C) Granulocytes include monocytes, whereas agranulocytes include basophils.


w

D) Granulocytes contain lysosomal granules, whereas agranulocytes do not.


3. Hemostatic homeostasis is maintained through which three interdependent components?
A) Red blood cells, white blood cells, and platelets
B) Serum, blood coagulation factors, and blood cells
C) Blood vessels, blood pressure, and platelets
D) Blood vessels, platelets, and blood coagulation factors
4. Which of the following are involved in innate (general) immune defenses? Select all that
apply.
A) Skin
B) Antibody response to an antigen
C) Stomach acid
D) Urine flow
E) Maternal transfer of antibodies

om
5. A patient with a skin wound shows pus-colored exudate and redness. Which of the
following stages of the inflammatory response is demonstrated in this patient?
A) Vascular stage

.c
B) Cell exudate stage

ep
C) Tissue repair stage

pr
D) Production of antibody stage
st
6. A patient had a severe case of pneumonia for a few weeks. The patient became concerned
te
that 2 weeks of antibiotics were not helping and went to the doctor for a repeat checkup.
A sputum sample led to a lung biopsy, which revealed the causative organism to be
ng

Aspergillus, an opportunistic infection. What are some similarities between the patients
innate (general) and adaptive (acquired) immune systems? Select all that apply.
si

A) They have highly specialized, systemic cells.


ur

B) They work to eliminate, or prevent attack of self from non-self.


yn

C) They have cells that originate in the bone marrow.


D) They have humoral immunity and can make IgG.
.m

7. A patients white blood cell count is:


w

WBC: 6,000 cells/mm3


w
w

Segmented neutrophils: 30%

Lymphocytes: 70%

This patient has an infection, yet the WBC count is low. What does this indicate? Select
all that apply.
A) A low WBC count with an infection may indicate a fungal disease.
B) The neutrophil count may indicate depressed bone marrow.
C) The neutrophil count may indicate a stress response due to hospitalization.
D) A low WBC count may indicate early stages of leukemia
8. The immune response is complex and involves humoral and cell-mediated components.
Which of the following is considered the first stage of the immune response?
A) Helper T cells are activated with the help of interleukin.
B) Macrophages process antigens and give them to T cells in lymph tissue.
C) T cells produce lymphokines which stimulate antibody-producing B cells.

om
D) Macrophages remove cellular debris.
9. A patient weighing 70 kg lost approximately 3,500 mL of blood in a traumatic incident.
What signs and symptoms does the nurse expect to find?

.c
A) Vital signs within normal range due to compensation

ep
B) Skin warm and dry with capillary refill less than 3 seconds

pr
C) Alert and oriented to person, place, time, and bodily relationships
D) st
Peripheral cyanosis, cool skin, diaphoresis, and weak thready pulse
te
10. A patient has been malnourished for a long time. What essential component of plasma
does the nurse most expect to be low?
ng

A) Albumin
si

B) B lymphocytes
ur

C) Sodium
yn

D) Platelets
.m

11. A patient has a disease that interferes with the production of vitamin B12. What reduced
blood component does the nurse most expect to find?
w

A) Lymphocytes
w

B) Platelets
w

C) Neutrophils
D) Erythrocytes
12. The patient has been exposed to significant levels of carbon monoxide. What component
of blood does the nurse most need to examine for adverse effects?
A) Plasma volume
B) Neutrophil count
C) Hemoglobin saturation
D) Arterial plasma pH
13. A patient has an acute infection. What type of blood cell does the nurse expect to be
increased?
A) Erythrocytes
B) Neutrophils
C) Platelets
D) Basophils

om
14. A patient has damage to the endothelial surfaces of blood vessels. The nurse expects
which of the following as the most useful normal body reaction?
A) Vasodilation of damaged vessels

.c
B) Migration of basophils to damaged areas

ep
C) Attraction of platelets to exposed collagen

pr
D) Activation of B-lymphocyte antibodies
st
15. A patient has suffered a major trauma, resulting in activation of the intrinsic and extrinsic
te
coagulation pathways. What laboratory result would most likely interfere with normal
coagulation?
ng

A) Normal fibrin level


si

B) Normal platelet count


ur

C) Elevated phospholipids
yn

D) Critical hypocalcemia
16. A patient has undergone surgery that resulted in unexpected massive blood loss. The
.m

bodys normal systems compensate to prevent excessive blood loss through clotting.
Under normal circumstances, what is the most important body process that is activated
w

after the coagulation pathways are activated?


w

A) Cellular phagocytosis
w

B) Antigenantibody formation
C) Coagulation inhibition
D) Platelet attraction to collagen
17. The patient has acquired immunity to the measles virus. What is the central component of
this acquired immunity?
A) Presence of natural killer cells
B) Developed antibodies
C) Platelets and eosinophils
D) Bone marrow stem cells
18. A culture report shows that a patients oral pharynx is colonized with normal flora. What
treatment does the nurse expect?
A) None, since normal flora are protective
B) Antiviral treatment
C) The test should be repeated.

om
D) Antibiotics
19. The patient has received a solid organ graft and is receiving drug therapy to suppress cell-
mediated immunity. What portion of the immune system does this medication suppress?

.c
A) Erythrocytes

ep
B) Gastric acid production

pr
C) T lymphocytes
D) Platelets st
te
20. A patient has a bacterial infection that has stimulated his humoral immune response. In
which of the following does the nurse expect to find an increase?
ng

A) Neutrophils
si

B) T lymphocytes
ur

C) Immunoglobulins
yn

D) Platelet aggregation
.m

Answer Key
w

1. A, B, C
w

2. D
w

3. D
4. A, C, D
5. B
6. A, B, C
7. B, C
8. B
9. D
10. A
11. D
12. C
13. B
14. C
15. D

om
16. C
17. B

.c
18. A

ep
19. C

pr
20. C

st
Chapter 46 Patient Assessment: Hematological and Immune Systems
te
1. When obtaining a history for a patient who is being admitted for hematological or
ng

immune disorders, the nurse first inquires about the chief complaint and history of present
illness. The nurse next inquires about which of the following?
si

A) Medical history
ur

B) Surgical history
yn

C) Last pap smear


.m

D) Recent CD4 count


2. When performing a physical examination of a hematological or immunocompromised
w

patient, the critical care nurse focuses on which of the following major areas?
w

A) Skin, liver, spleen, lymph nodes


w

B) Skin, respiratory, cardiac, spleen


C) Skin, liver, cardiac, lymph nodes
D) Respiratory, kidney function, liver, spleen
3. When examining a patients eyes for complaints of visual changes, which of the following
indicates hyperviscosity from polycythemia or retinal infarcts?
A) Sickle cell anemia
B) Retinal hamartoma
C) Stage one hypertension
D) Iron deficiency anemia
4. A patients smear sample in the lab shows spherocytes and elliptocytes. This is due to
abnormally shaped red blood cells originating from red blood cell membrane defects.
What predisposed factor could cause this to occur?
A) Hemolytic anemia
B) Hemophilia B
C) von Willebrands disease

om
D) Acute lymphoblastic anemia
5. A nurse assessing the stat lab results realizes that an increase in the patients mean

.c
corpuscular volume could possibly be which of the following?

ep
A) Folate deficiency

pr
B) Sickle cell
C) Chronic disease
st
te
D) Endocarditis
6. The critical care nurse promotes iron intake in the hematologic or immunocompromised
ng

patient to prevent which of the following?


si

A) Iron deficiency anemia


ur

B) Acute cholecystitis
yn

C) Uremic frost
D) Hyperspleenism
.m

7. A patient presents with a history of a congenital bleeding disorder. Upon assessment the
patient states that he has frequent nosebleeds and notices several bruises. The following
w

tests are ordered: PT, PTT, and factors VIIIR, VIII and IX. The results show a deficiency
w

in VIIIR, which indicates which of the following?


w

A) von Willebrands disease


B) Hemophilia A
C) Hemophilia B
D) AIDS
8. A patient presents to the unit with pallor, dyspnea, and dizziness. The patient has a
history of fatigue. Lab test results show low counts in the TIBC and iron levels. Based on
this information, which of the following is the likely disorder for this patient?
A) Iron-deficient anemia
B) Aplastic anemia
C) Thalassemia
D) Megaloblastic anemia
9. Which of the following are important to assess in the immunocompromised patient?
Select all that apply.
A) Nutritional status

om
B) Body temperature
C) White blood cell count

.c
D) Skin assessment

ep
10. An immunocomprised patient presents with the following: chills, tachycardia, tachypnea,
and hypotension. The critical care nurse suspects which of the following?

pr
A) Early septic shock
B) Acute pancreatitis st
te
C) AIDS
ng

D) HIV
11. The patient is being evaluated for a hematologic disorder. While collecting the history,
si

what does the nurse expect the patient to complain of?


ur

A) Specific system-related deficits


yn

B) A particular pattern of symptoms


.m

C) Vague and unrelated symptoms


D) No medication history
w

12. The patient is being evaluated for oxygen deficit. What laboratory study will be most
w

helpful to the nurse in this evaluation?


w

A) Red blood cell count


B) Complete blood count
C) Complete blood count differential
D) Serum carbon dioxide combining power
13. The patient has iron deficiency anemia. What changes in red blood cell morphology
would the nurse expect to find?
A) Decreased mean corpuscular volume
B) Elevated mean corpuscular hemoglobin
C) Increased hemoglobin and hematocrit
D) Decreased total iron-binding capacity
14. The patient has been diagnosed with multiple myeloma. What abnormality on a
laboratory test would the nurse most expect?
A) Rouleaux formations on peripheral smear
B) Reduced hemoglobin and hematocrit

om
C) Nucleated red cells on peripheral smear
D) Spherocytes on peripheral smear

.c
15. The nurse is interpreting a patients complete blood count. What does the CBC give an
overall indication of?

ep
A) Coagulation cascade

pr
B) Cardiac output and index
C) Bone marrow health st
te
D) Overall immune status
ng

16. The patient is seriously ill and has developed a fever, a cough productive of thick, yellow
sputum, and respiratory insufficiency. What changes in the white blood cell differential
si

count does the nurse expect to find?


ur

A) Increased neutrophils and bands


yn

B) Increased eosinophils and blasts


C) Decreased neutrophils with bands
.m

D) Decreased lymphocytes and neutrophils


w

17. A patient has a severe bacterial infection. What white blood cell does the nurse expect of
w

be most active in antibody production?


w

A) T lymphocytes
B) Neutrophils
C) B lymphocytes
D) Eosinophils
18. A critically ill patient has an elevated platelet count. What potential complication does the
nurse assess for?
A) Dehydration
B) Thrombosis
C) Hepatic impairment
D) Disseminated intravascular coagulation
19. Intradermal skin testing using a variety of antigens can be done to evaluate cell-mediated
immunity. If the patient has a defect in cellular immunity, what test result does the nurse
expect?
A) Erythema and induration
B) Itching and pain

om
C) No change in skin assessment
D) Increased B lymphocytes

.c
20. Because of an immune disorder, the patient is to undergo evaluation of bone marrow

ep
function. For what test does the nurse prepare the patient?

pr
A) Computed tomography (CT)
B) Intradermal skin testing
st
te
C) Bone marrow aspiration
D) Magnetic resonance imaging (MRI)
ng

21. A female patient is in intensive care recovering from a severe illness and has these
si

laboratory results: total white blood cells 2,000 cells/mm3, neutrophils 40%, lymphocytes
35%, monocytes 11%, eosinophils 4%, basophils 0%, red blood cell count 4.2 106
ur

cells/mm3, hemoglobin 11.7 g/dL, hematocrit 38%, serum sodium 140 mEq/L, serum
potassium 4.0 mEq/L. Based on the laboratory results, what is the highest-priority nursing
yn

action?
.m

A) Monitor cardiac rhythm closely.


B) Measure intake and output carefully.
w

C) Institute protective isolation.


w

D) Obtain an order for antibiotic therapy.


w

22. The patient has been diagnosed with severely compromised immune function. What
nursing intervention is most important?
A) Antibiotic therapy
B) Adequate protein
C) Coughing and deep breathing
D) Restricted visits from family
Answer Key

1. A
2. A
3. A
4. A
5. A

om
6. A
7. A

.c
8. A

ep
9. A, B, C
10. A

pr
11. C
12. A
st
te
13. A
ng

14. A
si

15. C
ur

16. A
yn

17. C
18. B
.m

19. C
w

20. C
w

21. C
w

22. B

Chapter 47 Organ and Hematopoietic Stem Cell Transplantation

1. Selecting the ideal candidate for transplantation is an intricate process. To evaluate a


patients suitability for transplantation, a comprehensive analysis on the patient is
performed that includes which of the following types of assessments? Select all that
apply.
A) Multisystem
B) Physiological
C) Psychosocial
D) Single donor system
2. Which of the following general assessment criteria guide the selection for
transplantation? Select all that apply.
A) Age
B) Presence of infection

om
C) Presence of malignancy
D) Health insurance

.c
3. Which of the following organ-specific criteria for transplantation are taken into
consideration?

ep
A) ABO typing

pr
B) Transfusion history
C) Gynecological examination st
te
D) Eye examination
ng

4. The critical care nurse knows that determination of compatibility in transplantation


involves the evaluation of two major antigen systems. A mismatch in compatibility may
si

cause an immediate reaction leading to organ loss. The primary determinant for solid
organ transplantation is which of the following?
ur

A) ABO grouping
yn

B) HBO grouping
.m

C) A and B antigen matching


D) HLA grouping
w
w

5. Which of the following are paramount procedures that the critical care nurse ensures are
complete during the preoperative phase for the patient undergoing organ transplantation?
w

A) ECG
B) Complete dialysis within 48 hours
C) CT of the head without contrast
D) Venogram and arteriogram
6. The nurse assesses a kidney transplant patient in the ICU immediately following surgery.
Which of the following assessments are paramount in the care of the patient? Select all
that apply.
A) Vital signs
B) Central venous pressure
C) Patency of urinary catheters
D) Blood sugars
7. During the immediate postoperative phase, the critical care nurse focuses on
hemodynamic stability. Blood products should be leukocyte reduced to avoid
introduction of CMV. CMV causes which of the following?
A) Death

om
B) Hyperkalemia
C) Shift of oxyhemoglobin curve to the right

.c
D) Immunoadsorbent assays

ep
8. Some degree of postoperative renal dysfunction is common due either to hepatorenal

pr
syndrome or hypotension during surgery. In addition, some immunosuppressive
medications are nephrotoxic. This can affect which of the following?
A) Fluid and electrolyte balance
st
te
B) Increased cardiac output
ng

C) Amount of dialysis treatments


si

D) Retransplantation
ur

9. A patient 30 days postoperative after allogeneic hematpoietic stem cell transplant presents
with adenovirus and Candida infection. The critical care nurse suspects which of the
yn

following?
A) Acute graft-versus-host disease
.m

B) Chronic graft-versus-host disease


w

C) Neutropenia
w

D) Stem cell rejection


w

10. Long-term care focuses on monitoring the patients progress and adherence to the health
care regimen. In solid organ transplant recipients, a major cause of graft loss in the long
term is failure of patients to adhere to which of the following?
A) Medication regimen
B) Dietary requirements
C) Follow-up appointments
D) Financial requirements
11. A patient is being evaluated for a solid organ transplant. If the patient is found to be
eligible for a transplant, which of the following would the nurse expect to find?
A) Age less than 65 years
B) Presence of active inflammatory process
C) End-stage organ disease refractory to other treatments
D) Able to reach full life span without transplant
12. A patient with end-stage organ failure is being evaluated for an organ transplant. What
factor is most likely to be a contraindication for transplant?

om
A) Age 30 to 55 years
B) Absence of acute or chronic infection

.c
C) Cannot afford the cost of medications after transplant

ep
D) Is receiving maximum doses of organ support medications

pr
13. The patient is being evaluated for compatibility for a solid organ transplant. What
st
congruency between donor and recipient is the primary requirement?
te
A) B and DR locus
B) HLA matching
ng

C) ABO blood grouping


si

D) White blood cell (WBC) match


ur

14. During evaluation for compatibility with the donor, a potential organ transplant recipient
yn

undergoes HLA matching. When explaining the importance of this test to the patient,
what is the best rationale for the nurse to use?
.m

A) This is simply a routine part of compatibility testing for all organs.


B) The higher the number of matching antigens, the lower the risk of rejection.
w
w

C) This test is used to ensure that the donor does not have an active infection.
w

D) HLA testing is part of the ongoing research into organ transplant success.
15. A patient is awaiting a stem cell transplant. What donor is most likely to have the best
HLA and MLC matching?
A) Not related, with same ABO group
B) Sibling of same gender
C) Any blood relative
D) Nonrelated donor of either gender
16. A patient has experienced brain death after a head injury and the family has consented to
organ donation. In this situation, who does the nurse now recognize as managing the care
of that donor?
A) Attending physician
B) Intensive care physician team
C) Registered nurse from organ procurement organization
D) Pathophysiologist from the medical examiners office
17. The nurse is caring for a patient who has just received a kidney transplant. What nursing

om
assessment finding would be most indicative of development of urinary drainage
blockage?

.c
A) Swelling over the graft site

ep
B) Decrease in serum creatinine
C) Sudden drop in hourly urine drainage

pr
D) Increase in serum sodium
st
18. The nurse is caring for a patient who has received a liver transplant. The patients blood
te
pressure is dropping and there is increased bloody drainage from the Jackson-Pratt drains
at the abdominal incision. What is the most important nursing action?
ng

A) Administer intravenous fluid bolus.


si

B) Continue close observation and monitoring.


ur

C) Notify the surgeon.


yn

D) Milk the Jackson-Pratt drainage tubes.


19. A patient with end-stage liver failure has received a liver transplant. During the
.m

postoperative care phase, the nurse monitors the patients oxygen status closely. What
condition may interfere with the use of peripheral pulse oximetry in this patient?
w

A) Metabolic acidosis
w

B) Severe jaundice
w

C) Normal body temperature


D) Clubbing of fingernails
20. After a liver transplant, what nursing assessment best indicates that the transplanted liver
is functioning?
A) Increased INR ratio
B) Increased aminotransferase levels
C) Unstable serum glucose levels
D) Increasing bile drainage
21. A patient has had a heart transplant using the standard atrial cuff technique. Based on the
use of this surgical technique, what cardiac monitor reading does the nurse most expect?
A) Third-degree heart block with ventricular rate 80
B) Two P waves for each QRS complex with one in disassociation
C) Two independently rhythmic QRS complexes

om
D) Ventricular pacing stimulus followed by a QRS complex
22. An adult patient with a heart transplant develops severe bradycardia. The patient also had
worsening hypotension and a deteriorating level of consciousness. What is the priority

.c
nursing action?

ep
A) Administer intravenous atropine.

pr
B) Administer intravenous epinephrine.
C) Apply a transcutaneous pacemaker.
st
te
D) Place the patient in reverse Trendelenburg position.
23. A patient who received a kidney transplant 2 months ago is having an episode of acute
ng

rejection. The patient expresses fear that his new kidney will not survive. What is the best
rationale for the nurses response?
si

A) Cytotoxic T cells damage the donor organ by secreting lysosomal enzymes.


ur

B) Acute rejection is the most common type of rejection.


yn

C) Acute rejection is the type most likely to respond to immune suppression.


.m

D) Acute rejection occurs when donor organ antigens trigger helper T cells.
24. A patient who has received a solid organ transplant is experiencing chronic rejection.
w

What symptoms does the nurse most expect?


w

A) Lack of T-cell activity or increase


w

B) Evidence of deteriorating organ function


C) Evidence of immune suppression
D) Negative antigenantibody reactions
25. A patient who is immunosuppressed after organ transplant has acquired an infection.
What symptoms does the nurse most expect?
A) Fever above 101 F
B) Increase in the percentage of WBC bands
C) Subnormal core body temperature
D) Decline in absolute neutrophil count
26. A patient who has received an allogenic stem cell transplant has developed graft-versus-
host disease. What symptoms does the nurse most expect?
A) Diminished lymphocytes
B) Elevated liver function tests

om
C) Absence of active T cells
D) Very high fever

.c
ep
Answer Key

pr
1. A, B, C
2. A, B, C
3. A, B, C
st
te
4. A
ng

5. A
si

6. A, B, C
ur

7. A
yn

8. A
.m

9. A
10. A
w

11. C
w

12. C
w

13. C
14. B
15. B
16. C
17. C
18. C
19. B
20. D
21. B
22. C
23. C
24. B
25. B

om
26. B

.c
Chapter 48 Common Immunological Disorders

ep
1. Why is it important for critical care nurses to be aware of antiretroviral therapy (ART) for
AIDS patients? Select all that apply.

pr
A) To recognize life-threatening toxicities associated with this class of drugs
B) st
To keep the patient from taking these drugs
te
C) To avoid drug interactions between ARTs and other classes of drugs that are
ng

common and potentially life-threatening


D) To avoid promoting ART drug resistance
si

2. A patient is admitted to the critical care unit with pneumocystis pneumonia. Which of the
ur

following will the nurse likely need to manage for this patient? Select all that apply.
yn

A) Intravenous (IV) trimethoprim and sulfamethoxazole (Bactrim, Septra) and


corticosteroids
.m

B) Dialysis
w

C) Bloodborne pathogen isolation


w

D) Ultrasound
w

3. A patient is admitted to the ICU to rule out necrotizing enterocolitis. Management of this
patient may include which of the following initial therapies?
A) Vasopressin
B) Prophylactic hematopoietic growth factors
C) Regular diet
D) NG feedings
4. A patient is admitted to the ICU with the following symptoms: weak pulse, distant heart
sounds, pulsus paradoxis, and distended neck veins. What is the likely diagnosis for this
patient?
A) Cardiac tamponade
B) Pneumocystis pneumonia
C) Pleural effusion
D) Cardiac artery rupture
5. A 48-year-old patient is admitted with hepatic veno-occlusive disease. Which of the

om
following are likely assessments and interventions for this patient? Select all that apply.
A) Observe and assess for dehydration.

.c
B) Obtain serum bilirubin.

ep
C) Prepare for a liver biopsy.

pr
D) Prepare for a surgical intervention.

st
6. Radiology results come back on a 44-year-old male cancer patient with a significant
pleural effusion. He is breathing at a rate of 60 breaths per minute and has already had
te
thoracentesis in the past to remove fluid. Current intervention should include which of the
following?
ng

A) IV antibiotics
si

B) IV antiviral agents
ur

C) Additional thoracentesis to reduce the fluid again.


yn

D) Bag/mask ventilation
7. A nurse is caring for a patient with spinal cord compression from a tumor. The family
.m

asks the nurse why this happens. Which of the following is the nurses best response?
w

A) There is not enough data to answer that question.


w

B) Tumors arising within the epidural space through vertebral or lymphatic spread may
cause spinal cord compression.
w

C) More than 75% of cases are secondary to small cell or squamous cell lung cancers,
and 10% to 15% are secondary to mediastinal lymphomas.
D) This occurs because of a large amount of fluid in the spinal space.
8. A nurse is caring for a patient with a tracheobronchial obstruction. The nurse should
assess the patient frequently and watch for which of the following major complications?
A) Pleural effusion and tachycardia
B) Total airway occlusion and hemorrhage
C) Total airway occlusion and vomiting
D) Hemorrhage and decreased level of consciousness
9. A nurse is caring for a patient with a diagnosis of hypocalcemia. This patient likely has a
corrected serum calcium level above which of the following?
A) 11 mg/dL
B) 5 mg/dL

om
C) 25 mg/dL
D) 2 mg/dL

.c
10. Which of the following are included in the management of a patient with hypocalcemia?
Select all that apply.

ep
A) IV fluids for hydration

pr
B) Medication to enhance renal excretion of calcium
C) Dialysis st
te
D) Glucose
ng

11. A patient has been diagnosed with human immunodeficiency virus (HIV). HIV results in
dysfunction of what specific portion of the immune system?
si

A) T lymphocytes
ur

B) B lymphocytes
yn

C) T4 helper cells
.m

D) IgG antibodies
12. When caring for an HIV-positive patient, what isolation technique should the nurse
w

implement?
w

A) Droplet
w

B) Aerosol
C) Contact
D) Standard
13. The nurse is caring for a patient with active acquired immune deficiency syndrome
(AIDS) in an intensive care unit. What is the most important component of care for this
patient?
A) Psychological support
B) Infection control measures
C) Prophylactic antibiotics
D) Neutropenic precautions
14. An AIDS patient is to receive zidovudine (AZT, ZDV, Retrovir). What is the most
serious side effect that the nurse should include in teaching?
A) Nausea and vomiting
B) Bone marrow suppression

om
C) Bleeding disorders
D) General malaise

.c
15. The nurse is caring for an AIDS patient who is being offered therapy with potent
combination antiretroviral therapy (ART). The patient is concerned about the many

ep
possible adverse effects of ART medications. What statement by the nurse most
accurately represents the rationale for the use of ART?

pr
A) It suppresses and maintains the viral load at nondetectable levels.
B) It will ultimately cure AIDS.
st
te
C) Therapy is based on research study results.
ng

D) Effects are similar in men, women, and children.


si

16. After receiving a bone marrow transplant, a cancer patient suffers a cardiopulmonary
arrest and needs mechanical ventilation. The family asks the nurse about the patients
ur

chances of survival. What is the most accurate information for the nurse to base the
answer on?
yn

A) Cardiopulmonary arrest and mechanical ventilation are associated with low survival
.m

rates in patients with cancer.


B) The hospital has a generally low incidence of death after cardiopulmonary arrest
w

compared to national averages.


w

C) Every patient and every situation must be evaluated individually, and no prediction
w

of survival is possible at this time.


D) The family should be referred to the physician and the chaplain to discuss their
concerns about survival.
17. A patient who has received a bone marrow transplant is suspected to have either
engraftment syndrome or sepsis and is undergoing evaluation. If the complication turns
out to be engraftment syndrome, what is the most likely differentiating criterion?
A) High fever, especially in the evening
B) Pruritic total body rash
C) Leukocytes less than 2,500/mm3
D) Dyspnea with pulmonary infiltrates
18. A patient with leukemia has been admitted with possible leukostasis. What sign or
symptom does the nurse most expect?
A) Critical neutropenia
B) High and persistent fever
C) SpO2 82% to 90%

om
D) Diffuse erythematous rash
19. The patient has been diagnosed with leukostasis. What definitive medical treatment does
the nurse anticipate?

.c
A) Supplemental oxygen

ep
B) Magnetic resonance imaging (MRI)

pr
C) Serial arterial blood gases
D) Therapeutic leukapheresis st
te
20. A patient with acute leukemia is also anemic and is to receive packed red blood cells.
What is the most important nursing action when giving the blood?
ng

A) Give intravenous diuretics before the blood.


si

B) Administer the blood with normal saline volume.


ur

C) Ensure administration of bronchodilators.


yn

D) Prepare for continuous renal replacement therapy (CRRT).


.m

21. A patient with cancer has developed probable cardiac tamponade. The patient most likely
presents with which of the following symptoms?
w

A) Flat jugular veins when erect


w

B) Muffled or distant heart sounds


w

C) Bradycardia and heart block


D) Generalized erythematous rash
22. A patient with thyroid cancer is at high risk for rupture of a carotid artery and has had a
carotid artery stent placed. The nurse notices a trickle of blood from the surgical site.
What is the first nursing action?
A) Application of a vascular clamp after digital pressure
B) Insertion of two large-bore intravenous lines
C) Constant digital pressure until reaching the operating room
D) Immediate notification of the surgeon and oncologist
23. The patient has developed a large pleural effusion as a complication from lung cancer.
The patients life expectancy is short. What medical treatment does the nurse anticipate?
A) Observation
B) Thoracentesis
C) Diuretic therapy

om
D) Pneumonectomy
24. A critically ill cancer patient has severe hypercalcemia. What treatment should the nurse
initiate first?

.c
A) Administer high doses of phosphorus.

ep
B) Withhold antineoplastics.

pr
C) Give high-volume intravenous saline.
D) Maintain complete bed rest. st
te
ng

Answer Key
si

1. A, C, D
ur

2. A, C
3. B
yn

4. A
.m

5. B, C
w

6. C
w

7. B
w

8. B
9. A
10. A, B, C
11. C
12. D
13. B
14. B
15. A
16. A
17. C
18. C
19. D
20. B

om
21. B
22. C

.c
23. B

ep
24. C

pr
Chapter 49 Common Hematological Disorders
st
1. Secondary polycythemia is often the result of increased RBC production that can develop
te
as a result of which of the following?
ng

A) Chronic chemotherapy
si

B) Obesity hypoventilation system


ur

C) Myocardial infarction
D) Sleep apnea
yn

2. The nurse completes a physical assessment of a 40-year-old female patient who presents
.m

with a history of anemia. What is the most important aspect of the assessment to
determine if the patient has hemodilution?
w

A) A stool guiac test


w

B) BUN and creatinine


w

C) Intake and output


D) Bilirubin
3. A 22-year-old male is in the ICU with sepsis. The physicians are trying to rule out acute
DIC. The nurse performs a primary assessment of the patient at the beginning of shift.
Which of the following items would be part of the initial assessment for this patient?
Select all that apply.
A) Check for signs of bleeding in the gums and/or mouth.
B) Check pupils for reaction.
C) Check IV sites or any areas where the patient may have had labs drawn for bleeding.
D) Check for altered level of consciousness.
4. A 4-year-old female patient who is accompanied by her mother presents with bruising
and petechiae all over her legs and trunk. The diagnosis is immune thrombocytopenia
purpura (ITP). The mother asks about the cause of the bruising. What is the nurses best
response to this question?
A) Dont worry. She probably just fell on the playground.

om
B) She is presenting with thrombocytopenia due to an acute episode of platelet
consumption. What medications is your child taking and has she recently received a
platelet transfusion?

.c
C) Its OK. This disease frequently is accompanied by bruising.

ep
D) The bruises indicate a problem with white blood cells.

pr
5. A nurse is doing an assessment of a 50-year-old female. She complains of abdominal pain
and the nurse suspects that the patient may be bleeding into her abdomen because of a
st
complication with a coagulation disorder. The nurse notifies the physician. What is the
te
nurses greatest concern about this patient?
ng

A) Hypotension, hypovolemic shock


B) Kidney disorder
si

C) Ectopic pregnancy
ur

D) Infection
yn

6. A nurse is caring for a patient with severe aplastic anemia. The patient was otherwise
healthy up until about 2 weeks ago and just returned from a trip to Mexico where she
.m

received some medication for a dog bite. The patient asks when she will be able to go
home and how serious this is. What is the nurses best response? Select all that apply.
w

A) You may have a serious adverse event from the medication you received in Mexico,
w

which caused your bone marrow to alter or stop producing red blood cells.
w

B) The physicians may be talking to you about a bone marrow transplant as that is the
best treatment for your diagnosis.
C) This is something that is easily treated with a blood transfusion.
D) You will need to get a white cell transfusion.
7. A nurse is working in the emergency department when a 17-year-old patient presents in
acute sickle cell crisis. He tells the nurse he is just getting over the flu and has been
vomiting for three days. What is the nurses highest priority for this patient?
A) Start an IV and give fluids to hydrate the patient.
B) Give morphine before completing the assessment because he is in pain.
C) Give the patient some food because he has been vomiting.
D) Leave the patient alone because he is in pain and doesnt want to be bothered.
8. A nurse is caring for a 29-year-old female who has labs drawn. The results come back
indicating that the neutrophil count is below 1,500 cells/mm3. What is the most likely
diagnosis for this patient?
A) Aplastic anemia

om
B) Chronic ideopathic thrombocytopenia
C) Sickle cell disease

.c
D) Leukopenia

ep
9. Further testing is being done to determine if a patient has Hodgkins lymphoma or non-

pr
Hodgkins lymphoma. Which test result would be conclusive in this case?
A) Elevated platelet count
st
te
B) White count of 10,000
C) Presence of abnormal Reed-Sternberg cells in biopsied tissue
ng

D) Fever and a white count of 0


si

10. A patient is admitted to the ICU with a history of general malaise for two weeks. A nurse
ur

does the initial assessment and suspects leukemia. What are the signs and symptoms that
will identify if this patient has leukemia? Select all that apply.
yn

A) Anemia, general malaise, bleeding


.m

B) Tachycardia, hypotension and shortness of breath


C) Hypertension, very high fever, and low heart rate
w
w

D) Bone pain, headache, vomiting, papilledema


w

11. A patient in a critical care unit who is intubated and on mechanical ventilation has
developed anemia. What nursing action has the highest priority to prevent this
complication?
A) Monitor hemoglobin and hematocrit every 4 hours.
B) Increase oral iron in diet.
C) Reduce frequency and amount of blood samples drawn.
D) Draw blood sample for type and cross-match.
12. A trauma patient is found to be profoundly anemic. What is the nurses primary focus in
planning care?
A) Draw laboratory sample for type and cross-match for blood replacement.
B) Initiate therapy with erythropoietin and iron.
C) Contact family to begin solicitation for blood donors.
D) Assess for and collaboratively treat the source of blood loss.
13. A patient admitted to critical care is found to have an absolute neutrophil count of 1,235
cells/mm3. What is the most important nursing intervention?

om
A) Institute strict isolation to prevent disease transmission from the patient.
B) Evaluate the patient for bone marrow transplantation for replacement.

.c
C) Institute reverse isolation to prevent disease transmission to the patient.

ep
D) Administer hematopoietic growth factor stimulation medications.

pr
14. A critically ill patient has an absolute neutrophil count of 1,000 cells/mm3. The nurse
st
assesses a single patient temperature of 101F; subsequent temperatures were normal, and
the patient reports no other new symptoms. What is the best nursing action?
te
A) Assume that the elevated temperature was erroneous.
ng

B) Evaluate for acute infection.


si

C) Assess vital signs more frequently.


ur

D) Document the temperature results as usual.


yn

15. A critically ill patient has been diagnosed with heparin-induced thrombocytopenia (HIT).
What symptoms would the nurse expect?
.m

A) Gastric aspirate with positive guaiac test


B) Oozing of blood around intravenous sites
w
w

C) Sudden severe hypoxia and lateral chest pain


w

D) Red blood cells in urine


16. A critical care patient has been receiving a continuous heparin drip for treatment of
possible deep vein thrombosis for 4 days. The patient now has developed symptoms
consistent with heparin-induced thrombocytopenia (HIT). What change in therapy does
the nurse anticipate?
A) None, as the patient needs anticoagulation for deep vein thrombosis
B) Change of heparin to subcutaneous low-molecular-weight heparin
C) Discontinuation of all heparin and heparin-coated devices
D) Replacement of platelets by transfusion and addition of aspirin therapy
17. A critically ill patient with a severe burn has developed disseminated intravascular
coagulation (DIC). What is the precipitating event for this complication?
A) Reaction to medications such as heparin or antineoplastic agents
B) Heat and direct damage of blood cells and vessels
C) Autoimmune disease causing destruction or clumping of platelets
D) Blood and fluid loss from the burn causing hemoconcentration

om
18. A critically ill patient has been diagnosed with disseminated intravascular coagulation
(DIC). What pattern of abnormal laboratory results does the nurse expect? Select all that
apply.

.c
A) Low absolute neutrophil count and red blood cell count

ep
B) High prothrombin time (PT) and partial thromboplastin time (PTT)

pr
C) Increased fibrin degradation products and presence of D-dimers
D) Decreased fibrinogen and thrombocytes
st
te
E) Increased fibrinogen and fibrin degradation products
F) Decreased total white cell count and hematocrit
ng

19. A critically ill patient is at risk for developing disseminated intravascular coagulation
si

(DIC). What presenting symptoms would the nurse expect if this occurs?
ur

A) Elevated core temperature and neutrophil count


yn

B) Sudden increase in dyspnea and hemoptysis


C) Continued need for opioid analgesia after trauma
.m

D) Increase in purulent pulmonary secretions and cough


w
w

Answer Key
w

1. B
2. C
3. A, C, D
4. B
5. A
6. A
7. A
8. D
9. C
10. A, D
11. C
12. D
13. C

om
14. B
15. C

.c
16. C

ep
17. B

pr
18. B, C, D
19. B st
te
Chapter 50 Anatomy and Physiology of the Integumentary System
ng

1. The dermis is composed of which of the following layers? Select all that apply.
si

A) Epidermis
ur

B) Papillary dermis
yn

C) Collagen
D) Hypodermis
.m

E) Subcutaneous tissue
w

F) Reticular dermis
w

2. The nurse is caring for a patient who has nerve damage of the dermal layer of the skin.
w

Which of the following are sensory receptors included in the dermal layer? Select all that
apply.
A) Heat
B) Cold
C) Inflammation
D) Pain
E) Touch
F) Pressure
G) Light
H) Numbness
3. How does the epidermal layer of the integument help to maintain homeostasis? Select all
that apply.
A) It kills bacteria.
B) It restricts water loss.

om
C) It allows all bacteria to escape.
D) It protects against microbes.

.c
E) It protects against radiation.

ep
F) It protects against ultraviolet light exposure.
G) It filters microbes.

pr
4. The nurse is caring for a patient in the ICU who became septic from a sebaceous cyst.
st
During a patient education session the nurse is explaining to the patient that the sebaceous
te
glands secrete sebum through the hair follicles. Sebum consists of which of the
following? Select all that apply.
ng

A) Mucus
si

B) Wax
ur

C) Serum
yn

D) Cholesterol
E) Oil
.m

F) Triglycerides
w

G) Sweat
w

H) Bacteria
w

5. Which layer of the skin contains blood vessels that are integral to the regulation of body
temperature and blood pressure?
A) Subcutaneous
B) Dermis
C) Epidermis
D) Endothelial
6. Which of the following are appendages of the skin contributing to homeostatic function
primarily by preventing heat loss? Select all that apply.
A) Hair
B) Teeth
C) Toes
D) Nails
E) Fingers

om
F) Glands
7. Which of the following protect underlying structures from invasion by microbes and
other foreign substances? Select all that apply.

.c
A) Subcutaneous layer

ep
B) Dermis

pr
C) Epidermis
D) Outer layer st
te
E) Pigment
ng

F) Appendages
8. Which of the following is produced when intravascular hydrostatic pressure increases
si

above the hydrostatic pressure within the dermal layer?


ur

A) Keratin
yn

B) Water
.m

C) Edema
D) Insult
w

9. Which of the following aids in the bodys regulation of water loss?


w

A) Dermis
w

B) Cornified, external layer of the epidermis


C) Epidermis
D) The innermost sublayer
10. A patient in the ICU is now prone to infection due to the fact that her skin has received
insult during her hospital stay. Which of the following is an insult to the skin? Select all
that apply.
A) Applying a barrier to the skin
B) Inserting an IV catheter in the lower arm
C) Inserting a urinary catheter
D) Persistent pressure that leads to reduced blood flow
E) Lack of sleep
F) Being nutritionally compromised
11. After a severe sunburn, the person experiences sloughing of the external layer of the
epidermis. What is the major effect of this sloughing?

om
A) Cosmetic
B) Infective

.c
C) Dehydrating

ep
D) Febrile
12. The nurse is caring for a patient with extreme heat exposure. The nurse explains to the

pr
patient and family that normally cutaneous blood vessels dilate to help reduce core body
st
temperature. The nurse would be most accurate in stating that these vessels are located in
what layer of the skin?
te
A) Epidermis
ng

B) Papillary dermis
si

C) Reticular dermis
ur

D) Hypodermis
yn

13. The nurse is caring for an elderly patient who complains constantly of being cold. This
situation is secondary to thinning of what layer of the skin?
.m

A) Epidermis
B) Papillary dermis
w
w

C) Reticular dermis
w

D) Hypodermis
14. A patient is admitted with heat exposure exhaustion and has a body temperature of 107F.
The patient has flushed dry skin and is not sweating. What portion of the skin or skin
appendages has this situation interfered with the function of?
A) Apocrine sweat glands
B) Hypodermis
C) Epidermis
D) Eccrine sweat glands
15. A neighbor asks the nurses opinion about an advertisement that states that a particular
soap will kill skin bacteria that cause body odor. What is the best rationale for the nurses
response?
A) All antimicrobial soaps have similar bactericidal actions.
B) The choice of soap brands is a personal decision.
C) Body odor comes primarily from the apocrine sweat glands.
D) The manufacturers information is regulated by federal law.

om
16. The nurse is assessing the patients fingernails and finds that the angle between the
proximal nail fold and the nail plate is approximately 165 degrees. What is the nurses

.c
diagnosis?

ep
A) Normal
B) Requires further evaluation

pr
C) Individual variation
D) Abnormally low
st
te
17. The patient has experienced large epidermal skin sloughing from a medication reaction.
ng

The dermal levels of the skin are exposed for over 40% of the patients body. What is the
most serious adverse reaction that the nurse should expect?
si

A) Excess fluid volume loss


ur

B) Elevated body temperature


yn

C) Numbness and paresthesia


D) Disturbance of body image
.m

18. The nurse is caring for an elderly critically ill patient. Because of the normal aging
w

changes in the skin, what is a nursing priority?


w

A) Adherence to hand hygiene protocols


w

B) Increased frequency of vital sign monitoring


C) Explanations of procedure rationales
D) Move patient without sliding on linen
19. The nurse is caring for a critically ill patient who has lost a large portion of skin. During
the patients bath, the nurse raises the temperature in the room. What lost function of the
skin is the nurse compensating for?
A) Protection from microbes
B) Protection from water loss
C) Body heat regulation
D) Organ cushioning
20. The patient has a very large burn involving the deep dermis and is being cared for in a
reverse isolation environment. The family is very annoyed by the need to don gowns and
masks to visit the patient. What is the best rationale for this rule?
A) It is the policy of the unit and hospital.
B) It protects the family from infection.

om
C) The patient is at increased risk for life-threatening infection.
D) It protects the family from odors.

.c
ep
Answer Key

pr
1. B, F
2. A, B, D, E, F st
te
3. B, D, F
ng

4. B, D, F
si

5. B
ur

6. A, D, F
7. C, D
yn

8. C
.m

9. B
w

10. B, C, D, F
w

11. A
w

12. C
13. D
14. D
15. C
16. A
17. A
18. D
19. C
20. C

Chapter 51 Patient Assessment: Integumentary System

1. A nurse is completing an integumentary assessment on a patient. Which of the following


specific techniques is part of this assessment? Select all that apply.
A) Auscultation

om
B) Tapping

.c
C) Palpation

ep
D) Comparison
E) Identification

pr
F) Inspection
st
2. A patient is receiving education related to skin cancer. The nurse explains to the patient
te
that skin lesions are variously described by which of the following? Select all that apply.
ng

A) Odor
B) Color
si

C) Texture
ur

D) Shape
yn

E) Cause
.m

F) General appearance
3. A nurse compares normal to abnormal vascular findings during a skin assessment. Which
w

of the following are considered normal vascular findings? Select all that apply.
w

A) Purpura
w

B) Cherry angioma
C) Spider angioma
D) Nevus flammeus
E) Urticaria
F) Capillary hemangioma
4. Skin color is determined by the presence of which of the following pigments? Select all
that apply.
A) Carboxyhemoglobin
B) Deoxyhemoglobin
C) Melanin
D) Pallor
E) Hemoglobin
F) Carotene
5. Which of the following are characteristic of nonpitting edema? Select all that apply.

om
A) Redness
B) Fluid leakage

.c
C) Tenderness

ep
D) Dryness

pr
E) Scaling
F) Warmth st
te
6. Which of the following provide information about the health of the skin and may yield
information about the patients fluid volume balance? Select all that apply.
ng

A) History
si

B) Mobility
ur

C) Temperature
yn

D) Color
E) Turgor
.m

F) Moisture
w

7. Erythema manifests as a reddish tone in light-skinned people and a deeper brown or


w

purple tone in dark-skinned people. Erythema is associated with which of the following
disorders and diseases? Select all that apply.
w

A) Liver disease
B) Cellulitis
C) Lung disease
D) Inflammation
E) Tissue trauma
F) Surgical wounds
8. A nurse is providing education to a patient with diabetes as it relates to yeast (candidiasis)
infections. In order to assess patient understanding, the nurse asks the patient to describe
symptoms of a yeast infection. Which of the following may the patient include in the
description? Select all that apply.
A) Red sores on oral mucosa
B) White patches on the tongue and/or oral mucosa
C) Red scaly rash on abdomen
D) Whitish pseudomembrane, and macropapular lesions in the groin

om
E) Erythema under the breasts
F) Petechial rash under abdominal folds and breasts

.c
9. A nurse is inspecting the patients skin for signs of melanoma. Which of the following

ep
may the nurse find during the assessment if the patient exhibits signs of melanoma?
Select all that apply.

pr
A) Pale red rash
B) st
Lesions that are either dark brown or black in color
te
C) Lesions with irregular borders
ng

D) Raised lesions with hair growing from them


E) Lesions greater than 6 mm
si

F) Lesions greater than 3 mm


ur

10. A Caucasian patient and an African-American patient are exhibiting signs of jaundice.
yn

Which of the following signs did the African-American patient exhibit? Select all that
apply.
.m

A) Yellow tone to skin


w

B) Yellow-greenish color of soles of feet


w

C) Yellow color of hands


w

D) Yellow-greenish color of palms of hands


E) Yellow-greenish color of skin
F) Yellow-greenish color of hair
11. The nurse is examining the skin of a critically ill patient. What technique should the
nurse?
A) Inspect the appearance of the skin every 12 hours.
B) Inspect only the anterior body skin every 12 hours.
C) Auscultate for bruits over skin lesions when found.
D) Percuss the borders of skin lesions when found.
12. A critically ill patient with a Hispanic and American Indian ethnic background is in
hemorrhagic shock. The nurse notices that the patients skin is yellowish brown and the
conjunctiva, oral mucosa, and nail beds are ashen gray. What variation in skin color does
the nurse document?
A) Pallor
B) Cyanosis

om
C) Jaundice
D) Erythema

.c
13. The patient is in decompensated shock. What abnormal variation in peripheral skin color

ep
does the nurse expect to find?
A) Pallor

pr
B) Cyanosis
C) Erythema
st
te
D) Jaundice
ng

14. The patient has a large dark-red area involving the left lower face and jaw. There is no
swelling or pain or history of trauma to the face or jaw. The patient states this mark has
si

been present as long as she can remember. What is the most appropriate nursing action?
ur

A) Obtain radiographs of the head and neck.


yn

B) Report symptoms to adult protective services.


C) Document findings in permanent record.
.m

D) Obtain clotting panel laboratory studies.


w

15. An elderly patient is admitted to the emergency department for treatment of an acute
w

urinary tract infection with possible sepsis. During the admission examination, the nurse
notices multiple ecchymoses of varying ages over both ulnar edges. The patient denies
w

falling and seems reluctant to answer any questions about the ecchymoses. The patients
daughter states that the patient falls down often, and she appears impatient with her
mother. The patients gait and balance are normal, and the patient is oriented and
otherwise cooperative. What is the best nursing action?
A) Obtain x-rays of the forearms.
B) Evaluate serum electrolyte values.
C) Report the situation to protective services.
D) Apply a vest restraint to the patient.
16. The patient is receiving the second dose of an intravenous antibiotic and complains of
urticaria that is rapidly spreading all over the his body. What is the most appropriate
nursing action?
A) Slow the medication administration rate.
B) Notify the physician if symptoms worsen.
C) Monitor vital signs frequently.
D) Discontinue the medication at once.

om
17. A patient is being treated for sepsis with several intravenous broad-spectrum antibiotics.
After a week of therapy, the nurse notices a rash in the groin that is erythematous, with

.c
white papules and pustules. There is also an odor similar to bread rising. What condition
does the nurse suspect?

ep
A) Urticaria from drug allergy

pr
B) Hepatic frost from liver failure
C) Candidiasis from yeast overgrowth st
te
D) Atypical rash from venereal disease
ng

18. The patient has systemic inflammatory response syndrome (SIRS) and very severe third
spacing of fluid. During routine assessment, the nurse finds that the patients skin appears
si

generally swollen, red, and shiny, and the nurse is unable to depress the surface of the
skin. What type or degree of edema is present?
ur

A) Nonpitting
yn

B) 2+
.m

C) 3+
D) 4+
w

19. During routine assessment of a critically ill patient, the nurse is able to indent the skin on
w

the dorsal surface of the foot 4 mm, and the skin rebounds in a few seconds. What degree
w

or type of edema is present?


A) Nonpitting edema
B) 2+ pitting edema
C) 3+ pitting edema
D) 4+ pitting edema
20. A critically ill patient is on mechanical ventilation through a tracheostomy and is
receiving enteral nutrition through a gastrostomy tube. External fixation devices are in
place for multiple fractures. Because of persistent hemodynamic instability, the patient
spends a great deal of time in the supine position. What areas of the patients body are
most likely to develop pressure ulceration?
A) Greater trochanter
B) Dorsal feet and toes
C) Ischial areas
D) Occipital areas
21. The nurse completes an assessment using the Braden scale on a critically ill patient. The

om
patients score is 9. What is the best nursing action?
A) Increase frequency of assessment to every 4 hours.

.c
B) Consult skin care specialist nurse for recommendations.

ep
C) Turn patient every 2 hours around the clock.

pr
D) Evaluate patients nutritional status.

st
22. The patient reports changes in a mole that are consistent with the development of
melanoma. What mole characteristic would the nurse least expect to assess?
te
A) Irregular borders
ng

B) Dark brown color


si

C) Located on upper arm


ur

D) Asymmetric shape
yn

Answer Key
.m

1. C, F
w

2. B, D, E, F
w

3. B, D, F
w

4. B, C, E, F
5. A, C, F
6. B, E
7. B, D, F
8. B, D, E
9. B, C, E
10. B, D
11. A
12. B
13. B
14. C
15. C
16. D

om
17. C
18. A

.c
19. B

ep
20. D

pr
21. B
22. C st
te
Chapter 52 Patient Management: Integumentary System
ng

1. A nurse is caring for a patient in the ICU. During assessment of the skin she notices a
si

wound on the patients coccyx that is covered in a brown leathery tissue-like material.
Which of the following is the patient exhibiting?
ur

A) Stage III pressure ulcer


yn

B) Unstageable pressure ulcer


.m

C) Abrasion
D) Stage II pressure ulcer
w

E) Suspected deep tissue damage


w

2. Which of the following is the final phase of wound healing?


w

A) Proliferative phase
B) Maturation phase
C) Inflammatory phase
D) Epitheliazation phase
E) Granulization phase
3. A nurse performs a visual assessment of a wound during the inflammatory phase of
healing. Which of the following observations does the nurse make? Select all that apply.
A) Bleeding
B) Erythema
C) Edema
D) Scarring
E) Itching

om
F) Pain
4. Which of the following are precipitating factors that can cause a chronic wound to fail to
follow a sequential healing process? Select all that apply.

.c
A) Rest

ep
B) Diabetes

pr
C) Infection
D) Mobility st
te
E) Malnutrition
ng

F) Peripheral vascular disease


5. A nurse is caring for a patient with a skin tear. Which of the following is included in this
si

care? Select all that apply.


ur

A) Clean the wound with normal saline.


yn

B) Clean the wound with hydrogel.


.m

C) Cover the wound with a self-adherent dressing.


D) Tape the dressing in place to avoid displacement.
w

E) Apply hydrogel before cleansing the wound.


w

F) Wrap the wound with a nonadherent dressing.


w

6. The nurse is providing education to a patient related to wound healing. Which specific
nutritional components that are needed by the body to heal will the nurse include in the
teaching? Select all that apply.
A) Sugar
B) Carbohydrates
C) Protein
D) Water
E) Fats
F) Minerals
7. Which of the following is indicative of tertiary intention?
A) During this time the wound is packed or irrigated to remove exudate and cellular
debris.
B) The edges of the wound are drawn together shortening the time required for the
wound to heal.
C) The wound is closed immediately to prevent infection.

om
D) It is associated with a decreased risk for infection and minimal scarring.
E) The wound is cleansed and then covered with a non-adhesive dressing.

.c
F) The wound is left open to air and cleansed daily with normal saline.

ep
8. The nurse is preparing to cleanse a pressure wound on a patient. Which of the following

pr
solutions are the nurses best choices for cleansing the wound? Select all that apply.
A) Hydrogen peroxide
st
te
B) Normal saline
C) Acetic acid
ng

D) Provodone-iodine
si

E) Commercial wound cleansers


ur

F) Dakins solution
yn

9. A nurse is caring for a patient with multiple chronic wounds over her body. Which of the
following is characteristic of a chronic wound?
.m

A) Caused by surgery or trauma


w

B) Follows a disorderly healing process


w

C) Follows an orderly healing process


w

D) Only occurs with patients who have diabetes


10. A nurse is caring for patients exhibiting stage III and IV pressure ulcers. Which of the
following dressings is the nurse most likely to use on these patients? Select all that apply.
A) Nonadhesive dressing
B) Calcium alginate dressing
C) Dry dressing
D) Wet saline dressing
E) Hydrofiber dressing
F) Rope dressing
11. The patient has a wound from peripheral vascular disease that is not healing in an orderly,
sequential manner, and the patient has diminished functional integrity. How does the
nurse label this wound?
A) Acute
B) Chronic
C) Nonhealing

om
D) Partial thickness
12. The patient has a peripheral vascular ulcer that involves loss of the epidermis, dermis, and

.c
subcutaneous tissue. Muscle and tendons are visible at the deepest part of the wound.

ep
How does the nurse classify this wound?
A) Stage II

pr
B) Stage III
C) Superficial
st
te
D) Full thickness
ng

13. The patient has a pressure ulcer that was classified as a stage III ulcer. With care, the
ulcer has partially healed and now appears as an area of nonblanchable erythema with
si

mild edema. What is the most appropriate nursing description?


ur

A) Stage I ulcer
yn

B) Stage III ulcer, healing


C) Stage III to I ulcer
.m

D) Stage III ulcer, exacerbated


w

14. The nurse is caring for a patient with a traumatic wound that is 4 days old. The wound is
w

erythematous, edematous, and painful. The nurse assesses the wound as being in what
phase of healing?
w

A) Proliferation
B) Epithelialization
C) Inflammatory
D) Maturation
15. An elderly patient has a wound that is slow to heal. What normal aging characteristic is
most likely to be a factor in slow wound healing?
A) Inefficient immune system
B) Slower reflexes and reaction time
C) Diminished pulmonary compliance
D) Lengthened recall time for events
16. The patient has a surgical wound with the edges approximated by sutures. By what
method does the nurse expect this wound to heal?
A) Naturally

om
B) Primary intention
C) Secondary intention

.c
D) Tertiary intention

ep
17. The patient has a pressure ulcer that is being allowed to heal by secondary intention.
What is the most significant complication of this type of wound healing?

pr
A) Venous stasis
B) Arterial insufficiency st
te
C) Scar formation
ng

D) Wound infection
18. The nurse is caring for a patient with a traumatic wound. What is the best way to describe
si

the size of the wound?


ur

A) Actual measurements in three dimensions


yn

B) Compare to size of a common object


.m

C) State percentage of body affected


D) Diameter at widest part in centimeters
w

19. The nurse is preparing to clean a deep wound. What cleaning solution is the safest for all
w

wounds?
w

A) Normal saline
B) Povidone-iodine
C) Dakins solution
D) Hydrogen peroxide
20. The patients wound is being treated with vacuum-assisted wound closure (negative
pressure therapy, or VAC). The nurse notices that the edge of the dressing is loose and
torn, the foam in the wound has expanded, and wound drainage is leaking through the
loose edge. What is the best nursing action?
A) Reinforce dressing with additional foam packing.
B) Reinforce transparent adhesive dressing.
C) Discontinue suction on the wound to prevent maceration.
D) Completely redress the wound with new materials.
21. The patient has a large, deep wound from trauma that is being treated with constantly
moist saline and gauze packing. Before cleaning and redressing the wound, what is the

om
most important nursing intervention?
A) Gather necessary supplies.

.c
B) Read previous documentation.

ep
C) Premedicate for pain.

pr
D) Explain procedure to patient.

st
22. A critically ill patient with a large, deep, open traumatic wound has diminished serum
albumin and total protein, diminished lymphocytes, and low iron levels. The patient has a
te
history of alcohol abuse and is currently being supported with enteral nutrition, but the
feeding is being held for excess residual volumes. What impact would the patients
ng

situation have on wound healing?


si

A) Diminished healing with increased risk of infection


ur

B) Improved healing secondary to the enteral nutrition


C) Healing by primary intention has the lowest risk.
yn

D) Normal healing unless wound is infected


.m

Answer Key
w
w

1. B
w

2. B
3. B, C, F
4. B, C, E, F
5. A, E, F
6. B, C, D, F
7. A
8. B, E
9. B
10. B, E
11. B
12. D
13. B
14. C

om
15. A
16. B

.c
17. D

ep
18. A

pr
19. A
20. D st
te
21. C
ng

22. A
si

Chapter 53 Burns and Common Integumentary Disorders


ur

1. A patient has had an alkali splash causing burns to the right upper extremity and the right
side of the face. Which intervention by the nurse is a priority?
yn

A) Obtain a health history.


.m

B) Irrigate with water to the burned area.


w

C) Apply an antibiotic cream.


w

D) Start an intravenous line (IV) of normal saline at 10 mL/hr.


w

2. A patient arrived in the emergency department (ED) after being found by a family
member unconscious in the garage with the car running. High flow oxygen at 100% is
administered in the ED. The nurse knows that which of the following diagnostic tests
would be the best to determine that the present therapy is effective?
A) Pulse oximetry
B) Chest x-ray
C) Serial carboxyhemoglobin levels
D) Hemoglobin and hematocrit
3. A patient has been treated for severe burns over 36% of the body. What assessment data
obtained by the nurse indicates the patient may be experiencing septic shock? Select all
that apply.
A) White blood count (WBC) 7,200
B) Blood pressure 60/40
C) PaO2 72
D) Urine output <10 mL/hour
E) Blood pressure 140/76

om
4. The nurse is caring for a patient 24 hours after fluid resuscitation for burns over 50% of
the body. Which assessment data obtained by the nurse indicates complications from the

.c
fluid resuscitation?

ep
A) Absent peripheral pulses
B) Crackles in lung fields

pr
C) Sinus bradycardia
D) 1+ pitting edema of the feet
st
te
5. The nurse determines that the patient is at the end of the resuscitation phase of the burn
ng

injury. What assessment data has the nurse obtained that is a primary indicator of this
phase?
si

A) Pa02 of 90
ur

B) A reddish-brown color of urine


yn

C) Urine output of >30 mL/hr


D) Urine output of <20 mL/hr
.m

6. The patient has received a nebulizer treatment with a bronchodilator for the treatment of
w

bronchospasm related to an inhalation injury. What intervention by the nurse can


effectively evaluate the effectiveness of this treatment?
w

A) Assess breath sounds.


w

B) Monitor for cardiac dysrhythmias.


C) Assess heart rate.
D) Assess for jugular vein distention.
7. The nurse is assessing the patient with a circumferential burn to the left upper extremity.
The nurse anticipates the performance of an escharotomy with which of the following
assessment findings? Select all that apply.
A) Absent distal radial pulses
B) Progressive diminution of ultrasound signal
C) Limited range of motion
D) Decrease in capillary refill
E) Pink nail beds
8. A patient with third degree burns on both lower extremities from a house fire has been in
the hospital for two weeks. The patient states, I do not feel like walking today. I am going
to stay in bed. What is the best response by the nurse?

om
A) I understand. I have days like that too. Go ahead and stay in bed.
B) I am going to tell the doctor that you were not compliant with the order to get out of

.c
bed.

ep
C) I will allow you to wait until the afternoon, but then you must get up.

pr
D) It is important for your recovery to walk every day. Lets walk in the hallway for 10
minutes and then get a snack.
st
9. A patient is in the ICU with lesions of the chest, back, and extremities. The physician
te
suspects toxic epidermal necrolysis (TEN). What statement by the patient would alert the
nurse to the precipitating factor?
ng

A) I started taking an antibiotic for a respiratory infection.


si

B) I was working in the garden planting flowers.


ur

C) I have been treated for basal cell carcinoma of my nose.


yn

D) I had a flu shot one month ago.


10. A patient is admitted into the emergency department with the complaint, I burned my
.m

hand. I didnt realize the stove top was still so hot. The palmar surface of the hand is
blistered, red, and blanches. What type of burn does the nurse document this patient has?
w

A) Superficial
w

B) Deep partial thickness


w

C) Superficial partial thickness


D) Full thickness
11. The patient has an acute burn injury. What might be the cause of the burn trauma? Select
all that apply.
A) Pot of boiling water pulled off a stove
B) Crockpot full of beans spilled
C) House or room fire
D) Absence of electrical power
E) Exposure to anhydrous ammonia
F) Contact with white phosphorus
12. The patient has a superficial partial-thickness (second-degree) burn. What characteristic
unique to this type of burn does the nurse expect to find?
A) Bright red color

om
B) Surface moist and supple
C) Fluid-filled blisters

.c
D) High pain level

ep
13. An adult has a thermal burn injury involving all of one leg and the entire posterior trunk.
There are also three scattered areas on the anterior trunk, each approximately the size of

pr
the patients palm. As part of initial assessment, the nurse calculates the size of the burn
using a combination of the Rule of Nines and the Rule of Palm. What is the result of this
st
calculation in percentage of total body surface area (TBSA)?
te
14. The underlying pathophysiology of a burn trauma centers around what?
ng

A) Location of burn damage


B) Zone of coagulation
si

C) Maximum exposure temperature


ur

D) Extent of cellular injury and death


yn

15. A patient has a large thermal burn injury and is receiving large amounts of intravenous
crystalloid fluid as part of initial therapy. What is the best explanation of the purpose of
.m

this therapy?
w

A) It is part of the protocol for initial burn management.


w

B) Burn trauma cell damage causes external fluid loss.


w

C) Development of third spacing reduces renal perfusion.


D) Crystalloid intravenous fluids are less expensive.
16. A patient with a large thermal burn injury is found to have depressed cardiac contractility
and a resultant low cardiac index. What is the underlying, primary, physiologic rationale
for this situation?
A) Inflammation-triggered release of nitric oxide
B) Reduction of platelet aggregation and adhesion
C) Decreased intravascular volume from third spacing
D) Peripheral vasodilatation from inflammation
17. A patient has a large burn injury that occurred in an enclosed space. On initial
assessment, the patient is found to have erythema and blistering of the mouth and
pharynx, hoarse speech, and tachypnea. What immediate therapy addressing these
symptoms does the nurse anticipate?
A) Intravenous fluid resuscitation
B) Prophylactic antimicrobial therapy

om
C) Application of topical burn medications
D) Endotracheal intubation

.c
18. A patient with a large thermal burn has been admitted to the burn intensive care unit.

ep
Because the leading cause of death in burned patients after the initial care period is
infection, what is the nurses priority action?

pr
A) Ensure compliance with hand hygiene protocols by all health care team members.
B) st
Limit visits of legal next of kin to very brief periods of time.
te
C) Adhere to clean aseptic principles during wound care and dressing changes.
ng

D) Collect environmental cultures and wound cultures as scheduled.


19. In a motor vehicle crash, a patient suffers a skull fracture, possible cervical spine injury,
si

multiple extremity fractures, and a large thermal burn. On initial admission to the
ur

emergency department, what is the nursing priority of care?


A) Intravenous fluid resuscitation
yn

B) Protection from infection


.m

C) Assessment of extent of burns


w

D) Protection of airway and cervical spine


w

20. A patient weighing 65 kg has a deep partial-thickness burn that totals 45% total body
surface area (TBSA) and is ordered to receive fluid resuscitation with volumes calculated
w

by the maximum amount from the American Burn Association consensus formula. The
burn occurred at 0800 and intravenous fluids are initiated at 1000. For the first 8 hours
after the burn, the nurse plans to administer what amount of intravenous lactated Ringers
solution in mL/hr?
21. An adult patient with a large thermal burn has been receiving intravenous fluid
resuscitation calculated by the maximum amount according to the American Burn
Association consensus formula. What symptom or result would indicate the onset of a
complication secondary to rapid fluid resuscitation?
A) Urine output 30 to 50 mL/hr
B) Serum sodium 135 to 140 mEq/L
C) Weight gain of 12% since admission
D) New coarse crackles auscultated at lung bases
22. The patient has a large burn wound that is mostly a full-thickness (third-degree) injury.
Wound care included surgical excision and grafting with a variety of materials. The
patient and family ask the nurse to explain why repeated surgical procedures are
necessary. What is the best explanation the nurse can give the family?

om
A) Because of pain and surgical shock, the repair is done in stages.
B) The surgeon is the appropriate person to answer this question.

.c
C) Successful autografting requires preparation of the wound bed.

ep
D) Lack of patient compliance has reduced the effectiveness of autografting.

pr
23. A patient with a healing burn wound involving several joints is being evaluated at a
patient care conference. What would indicate an effective collaborative plan of care and
functioning of the health care team? st
te
A) Conference discussion focuses on patient and family goal achievement.
ng

B) Complication of contractures is blamed on inadequate attention by physical therapy.


C) Lack of adequate nutritional support is explained by poor patient compliance.
si

D) Family and patient have filed complaints of uncaring behaviors by physician.


ur

24. The patient has been admitted with toxic epidermal necrolysis (TENS) involving 40%
yn

total body surface area (TBSA), including the mouth, trunk, and lower extremities. What
therapy does the nurse anticipate initiating?
.m

A) Intravenous antimicrobial therapy


w

B) Vigorous physical therapy and walking


w

C) Topical corticosteroid therapy


w

D) Intravenous fluid and electrolyte replacement

Answer Key

1. B
2. C
3. B, C, D
4. B
5. D
6. A
7. A, B, D
8. D
9. A
10. C

om
11. A, B, C, E, F
12. C

.c
13. 39%

ep
14. D

pr
15. C
16. A st
te
17. D
ng

18. A
19. D
si

20. 975
ur

21. D
yn

22. C
.m

23. A
24. D
w
w

Chapter 54 Shock, Systemic Inflammatory Response Syndrome/Multiple Organ


w

Dysfunction

1. What would the nurse identify as the primary purpose for the administration of
intravenous (IV) crystalloid fluids in the patient with hypovolemic shock?
A) Decrease myocardial oxygen demand.
B) Maximize oxygen-carrying capability.
C) Increase capillary permeability.
D) Restore circulating volume.
2. The nurse is administering an intravenous antibiotic infusion over 30 minutes for a
patient with cellulitis of the left lower extremity. The patient states, I am itching all over
and am having trouble swallowing. What priority interventions by the nurse are necessary
for this patient? Select all that apply.
A) Stop the antibiotic infusion.
B) Administer subcutaneous epinephrine.
C) Administer diphenhydramine (Benadryl) IV.
D) Switch to amoxicillin by mouth.

om
E) Administer Ativan for the patients anxiety.
3. A nursing assessment of a patient with hypovolemic shock is most likely to reveal what

.c
assessment findings? Select all that apply.

ep
A) Tachycardia
B) Oliguria

pr
C) Disoriented to time and place
D) Diuresis
st
te
E) Bradycardia
ng

F) Hypotension
si

4. The nurse is assigned to the care of a patient in the ICU who is in cardiogenic shock.
What priority nursing intervention is necessary to conserve myocardial energy and
ur

decrease workload of the heart?


yn

A) Lactated Ringers at 150 mL/hr


B) Morphine sulfate 4 mg IV
.m

C) Furosemide (Lasix) 80 mg IV
w

D) Epinephrine 1:1,000, 0.3 mL IV


w

5. The nurse in the ICU is assigned to care for a patient with septic shock. What nursing
w

interventions are necessary to prevent malnutrition and optimize cellular function in this
patient?
A) Administration of crystalloid solutions IV
B) High calorie, low protein diet
C) Enteral feedings
D) Administration of multivitamins in the IV fluid
6. In developing the discharge plan for a patient who was treated in the hospital for
anaphylactic shock related to a nonsteroidal anti-inflammatory (NSAID) allergy, what
would be the most important information for the nurse to include?
A) Adhere to dietary restrictions.
B) Follow up in one month with the physician.
C) Check labels of over-the-counter medications prior to taking.
D) Have blood pressure checked on a regular basis.
7. The nurse is assigned to a patient in the ICU who is on a ventilator for exacerbation of

om
chronic obstructive pulmonary disease. What intervention by the nurse can prevent the
development of multiple organ dysfunction syndrome?

.c
A) Suctioning the patient every 2 hours

ep
B) Enteral feedings
C) Oral care every 2 hours

pr
D) Administration of total parenteral nutrition
st
8. The nurse understands that which of the following patients in the hospital is at the
te
greatest risk for cardiogenic shock?
ng

A) The 76-year-old male patient with a history of diabetes mellitus and previous
myocardial infarction (MI)
si

B) The 42-year-old male who has mitral valve prolapse with a left ventricular ejection
ur

fraction of 65%
C) The 52-year-old female with a recent small anteroseptal wall MI
yn

D) The 84-year-old female with hypertension


.m

9. The patient in the ICU is being treated for left lower lobe pneumonia. What assessment
findings by the nurse may indicate that the patient is developing systemic inflammatory
w

response syndrome (SIRS)? Select all that apply.


w

A) White blood cell count of 24,000/mm3


w

B) Respiratory rate of 24
C) Blood pressure of 100/60
D) Heart rate 96
E) Atrial fibrillation
10. The nurse is caring for a patient with hypovolemic shock who has had 6 units of packed
red blood cells. Which of the following values would alert the nurse to a complication
related to the administration of blood?
A) Potassium level of 6.0
B) Hemoglobin of 13
C) Sodium level of 134
D) pH 7.37
11. A patient in the critical care unit has developed shock. What symptom or symptom group
does the nurse expect to assess in any type of shock?
A) Tissue hypoxia

om
B) Massive vasodilation
C) Extreme blood loss

.c
D) Presence of enterotoxins

ep
12. A critically ill patient has developed shock. What nursing assessment result indicates a
normal compensatory mechanism?

pr
A) Reduction of respiratory depth
B) st
Increase in systemic vascular resistance (SVR)
te
C) Decrease in circulating catecholamines
ng

D) Increased stimulation of baroreceptors


13. The patient is in hypovolemic shock from traumatic massive blood loss and is tachypneic
si

and tachycardic, with cool, clammy skin and weak and thready pulses. What additional
ur

assessment parameter would the nurse be least likely to find during stage one or early
compensated shock?
yn

A) Hypotension
.m

B) Increased urine output


C) Estimated blood loss greater than 30%
w
w

D) Mild altered mental status


w

14. A patient is being treated for severe hypovolemic shock. Based on the primary treatment
goal, what nursing intervention has the highest priority?
A) Frequent measurement of vital signs
B) Management of mechanical ventilation
C) Rapid intravenous fluid administration
D) Insertion of urinary drainage catheter
15. The patient has developed cardiogenic shock and is decompensating. What pattern of
hemodynamic alterations does the nurse expect to find?
A) High preload, high afterload, low cardiac index, tachycardia
B) Low preload, low afterload, high cardiac index, bradycardia
C) Low preload, high afterload, high cardiac index, tachycardia
D) High preload, low afterload, high cardiac index, tachycardia
16. The patient is in decompensated cardiogenic shock. What collaborative intervention best
addresses the central cause of cardiogenic shock?

om
A) Mechanical ventilation
B) Hemodynamic monitoring

.c
C) Pharmacologic sedation

ep
D) Intravenous nitrate infusion

pr
17. For a patient in cardiogenic shock, the physician has ordered an intravenous continuous
st
infusion of dobutamine hydrochloride. What nursing assessment result demonstrates
achievement of therapeutic goals?
te
A) Blood pressure 120/70 mm Hg
ng

B) Urine output 30 to 40 mL/hr


si

C) Arterial oxygen saturation 60%


ur

D) Heart rate 110 to 120 bpm


yn

18. A patient is in shock and is exhibiting low blood pressure, low systemic vascular
resistance (SVR), peripheral edema, pulmonary wheezing, tachycardia, and nausea and
vomiting. What precipitating event does the nurse expect for this group of symptoms?
.m

A) Acute myocardial infarction


w

B) Bacterial infectious illness


w

C) Recent seafood meal


w

D) Massive fluid loss


19. The patient has been diagnosed with shock secondary to an antigenantibody reaction.
What collaborative intravenous intervention has the highest priority?
A) Dobutamine
B) Red blood cells
C) Antimicrobials
D) Epinephrine
20. A critically ill patient has developed septic shock. What pattern of hemodynamic values
does the nurse expect to find?
A) Low preload, high afterload, low cardiac index, tachycardia
B) Low preload and afterload, high cardiac index, tachycardia
C) High preload and afterload, low cardiac index, tachycardia
D) Normal preload, low afterload, normal cardiac index, bradycardia

om
21. A leading cause of death in critically ill patients is sepsis and septic shock. What nursing
intervention is most directed toward preventing this life-threatening complication?
A) Strict adherence to hand hygiene protocols

.c
B) Prompt initiation of isolation protocols

ep
C) Patient and family preventive teaching

pr
D) Sterile technique for care of intravenous sites
st
22. A patient has been diagnosed with septic shock and is receiving intravenous fluid
te
resuscitation along with other therapies. What nursing assessment best indicates
improvement in tissue perfusion?
ng

A) Mean arterial pressure 65 to 70 mm Hg


si

B) SvO2 80% to 90%


ur

C) Skin warm and dry


yn

D) Arterial bicarbonate ion 22 to 24 mEq/L


23. A critically ill patient who is mechanically ventilated and has developed shock is in need
.m

of nutritional support. What route is preferred for this patient?


A) Oral
w
w

B) Enteral
w

C) Parenteral
D) Variable
24. A patient in shock has developed systemic inflammatory response syndrome (SIRS).
What is the most likely type of shock resulting in SIRS?
A) Hypovolemic
B) Septic
C) Cardiogenic
D) Any shock
25. A critically ill patient has developed multiple organ dysfunction syndrome (MODS).
What should the nursing goal for management of the patient with impending MODS
center on?
A) Early normalization of SvO2 and acidbase balance
B) Use of intravenous drotrecogin alfa (Xigris)
C) Specific organ system support
D) General intensive nursing care

om
Answer Key

.c
ep
1. D
2. A, B, C

pr
3. A, B, C, F
4. B st
te
5. C
ng

6. C
si

7. C
ur

8. A
9. A, B, D
yn

10. A
.m

11. A
w

12. B
w

13. A
w

14. C
15. A
16. D
17. B
18. C
19. D
20. B
21. A
22. D
23. B
24. D
25. A

om
Chapter 55 Trauma

1. A patient sustained injuries in a motor vehicle accident and is in the Emergency

.c
Department. A CT scan of the head and neck have been ordered. What part of the survey
is this?

ep
A) Primary

pr
B) Secondary
C) Tertiary st
te
D) Initial
ng

2. A patient who is in the Emergency Department was attacked in a parking lot and suffered
several stab wounds to various areas on the chest and abdomen; BP 100/60, heart rate
108, respiratory rate 20, pulse oximetry 98%. In order to counteract the blood loss and
si

restore circulating volume for this patient, what priority intervention will the nurse
ur

perform?
yn

A) Start lactated Ringers at 150 mL/hr.


B) Start dopamine at 5 mcg/kg/min.
.m

C) Start an albumin infusion wide open.


w

D) Start a unit of uncrossmatched blood.


w

3. The nurse is assigned to a patient in the Emergency Department who exhibits paradoxical
w

chest movement. What intervention by the nurse can help improve oxygenation in this
patient?
A) Elevate the head of the bed 30 degrees.
B) Splint the chest with 3-inch surgical tape.
C) Turn the patient with the injured side down.
D) Place the patient in the prone position.
4. A patient has been involved in a motor vehicle accident. The patient, who was driving,
was unrestrained by a seat belt when hitting the car in front of him. The patient is
complaining of midsternal pain, restlessness, and difficulty breathing. What is the priority
nursing diagnosis for this patient?
A) Anxiety
B) Impaired gas exchange
C) Impaired circulation
D) Pain

om
5. A patient has suffered a mild pulmonary contusion from a jet ski accident. What nursing
interventions are appropriate for this patient? Select all that apply.

.c
A) Maintenance of chest tubes

ep
B) Frequent pulse oximetry monitoring
C) Assessment of lung sounds every 2 hours

pr
D) Continuous epidural analgesia
E) Maintainance of ventilatory support
st
te
6. A patient sustained an injury to the right arm after falling off a motorcycle. The patient is
ng

complaining of severe pain and is unable to feel the fingers of the right hand. Radial pulse
is absent. What is the priority intervention by the nurse?
si

A) Elevate the right arm above the level of the heart.


ur

B) Notify the physician.


yn

C) Apply ice packs to the affected area.


D) Place the patient in Trendelenburg position.
.m

7. The nurse is caring for a patient with deep vein thrombosis of the left lower extremity.
w

The patient exhibits a decrease in pulse oximetry readings from 98% to 86%, shortness of
breath with a respiratory rate of 34, and is now disoriented to place. The nurse recognizes
w

that these findings are caused by what complication?


w

A) Pulmonary edema
B) Cardiac tamponade
C) Pulmonary embolus
D) Tension pneumothorax
8. The nurse is assigned to care for a patient who was admitted 2 days previous after a four-
wheeler accident. The patient sustained a closed fracture to the left femur and had an
open reduction with internal fixation the same day. What is a priority for the nurse to
assess for this patient?
A) White blood count
B) Urinary output
C) Cardiac output
D) Pulse oximetry
9. A patient has been brought into the Emergency Department via ambulance with
resuscitation efforts being performed. It is unlikely that the patient will survive the severe

om
injuries sustained. Two adult children of the patient are present and are requesting to be
with the patient at this time. What is the best response by the nurse?

.c
A) I dont think you should see your loved one like this. Wouldnt you rather remember
him the way he was?

ep
B) Our hospital doesnt allow more than one family member in with a patient. One of

pr
you can come in and one of you will have to wait in the waiting area.
C) You may come in with your parent and I will have someone stay with you to explain
what is happening. st
te
D) I have been through this many times and I promise you, it is a sight that you dont
ng

want to remember.
10. The nurse is caring for the patient with chest tubes. Which observation by the nurse is a
si

priority concern?
ur

A) 250 mL/hr of blood in drainage collection system


yn

B) Pulse oximetry of 94%


C) Blood pressure of 104/62
.m

D) 30 mL/hr of urine output


w

11. A patient is admitted to the emergency department after he was hit by a car. The car was
w

going about 30 mph and was braking at the time of impact. The patient was struck just
above the right knee, fell forward over the hood of the car, striking his anterior chest, and
w

then slipped off the hood of the car and hit the pavement head first. Based on the
mechanism of injury and transfer of force, what injuries does the nurse most expect?
Select all that apply.
A) Fracture of left femur and damage to left knee
B) Fractures of thoracic and lumbar spine
C) Fractured ribs and cardiac and lung contusion
D) Bilateral radial and humerus fractures
E) Closed head injury and cervical spine fracture
F) Bilateral clavicle and scapular fractures
12. The patient has received a gunshot wound. To help predict the amount of damage, what
information does the nurse collect?
A) Location of the shooting
B) Information about the shooter
C) Type of weapon and caliber of bullet

om
D) Whether the injury involved a felony
13. A patient was in a serious motor vehicle crash. At the scene, what is the highest priority
of care?

.c
A) Extrication from the vehicle

ep
B) Cervical spine protection

pr
C) Establishing two large-bore intravenous lines
D) Collecting information about the crash st
te
14. On initial admission of a trauma victim to the emergency department, the nurse completes
a primary survey. The patient is awake and tachypneic, is using accessory muscles of
ng

respiration, has unequal chest expansion, and is very anxious. There are absent breath
sounds on the right and cyanosis on 100% oxygen, and the trachea is deviated to the left.
si

What action takes the highest priority during the primary survey?
ur

A) Jaw thrust maneuver


yn

B) Suctioning the oral pharynx


C) Chest tube insertion
.m

D) Assisting ventilation with bag-mask device


w

15. A patient has been admitted to the emergency department after being in a severe motor
w

vehicle crash. The patient was a passenger and had a lap and seat belt in place. The
patient is lethargic and moaning. Initial exposure and head-to-toe examination reveals
w

scattered minor abrasions and contusions and bruising over the upper abdomen. The
patient moans more when the abdomen is palpated, and the abdomen is rigid. Heart rate is
110, capillary refill is greater than 4 seconds, and blood pressure is 140/88 mm Hg. What
is the nursing priority of care?
A) Administer intravenous opioid for pain.
B) Increase rate of intravenous crystalloid.
C) Obtain CT of the abdomen.
D) Prepare for immediate endotracheal intubation.
16. As part of a major trauma, a patient has suffered a flail chest injury. What hallmark sign
of flail chest does the nurse expect to find?
A) Flail segment elevation during inhalation
B) Evidence of rib fractures on chest radiograph
C) Flail segment depression during inhalation
D) Hypoxemia evident on arterial blood gases

om
17. A patient is admitted to the CCU after experiencing blunt trauma to the chest. Among
other injuries, the patient has a flail chest on the left and several extremity fractures.
About 12 hours after admission, the patient is tachypneic and complaining of shortness of

.c
breath. Breath sounds are present bilaterally with scattered fine crackles. Chest
radiograph shows an ill-defined, patchy, ground-glass area of density on the left. If the

ep
patient has a pulmonary contusion, what is the nursing priority?

pr
A) Monitor pulse oximetry and arterial blood gases closely.
B) Place an oral endotracheal tube immediately.
C)
st
Increase the amounts of intravenous crystalloid administration.
te
D) Obtain sputum culture and sensitivity and Gram stain.
ng

18. A patient has suffered severe blunt trauma to the abdomen with bruising, diffuse pain,
si

guarding, and rigidity evident. Damage to which structure is most likely?


ur

A) Stomach
B) Bladder
yn

C) Large intestine
.m

D) Liver
w

19. As part of a multiple trauma injury, the patient has suffered a closed fracture of the
radius. What nursing assessment finding indicates a significant complication warranting
w

immediate treatment?
w

A) Swelling and pain over the fracture


B) Loss of pulses distal to the fracture
C) Ecchymosis over the fracture
D) Deformity of forearm
20. A patient has experienced multiple fractures, including pelvic and long bone fractures.
After 72 hours, the patient complains of tachypnea and dyspnea and is found to have
cyanosis, tachycardia, confusion, and fever. Laboratory analysis reveals a normal
complete blood count except for thrombocytopenia and progressive respiratory
insufficiency. What is the nursing care priority?
A) Administer oxygen and monitor pulse oximetry.
B) Initiate low-molecular-weight heparin therapy.
C) Obtain cultures of all body substances.
D) Initiate fall and seizure precautions.
21. During a motor vehicle accident, a patient sustained blunt trauma to the head and face,
resulting in hairline skull fracture and a LeFort III maxillofacial fracture. The patient also

om
has bruising across the chest and upper abdomen and multiple small superficial bleeding
abrasions and lacerations. On admission to the emergency department, what is the nursing
care priority?

.c
A) Apply direct pressure to bleeding areas.

ep
B) Assess neurologic status.

pr
C) Perform endotracheal intubation.
D) Administer tetanus booster immunization. st
te
22. As part of a multiple trauma injury, a patient developed hemorrhagic hypovolemic shock,
necessitating fluid resuscitation with massive amounts of intravenous crystalloid fluids
ng

and blood products as well as extensive surgical repair under general anesthesia. Twenty-
four hours later, the patient develops hypoxia unresponsive to oxygen therapy and diffuse
si

white, ground-glass infiltrates of the lung fields on a chest radiograph. Development of


this complication has what effect on the patients recovery?
ur

A) Significantly greater chance of death


yn

B) No change in outcome expectations


.m

C) Outcome depends on treatment.


w

D) Lower chance of death


w

Answer Key
w

1. B
2. A
3. C
4. B
5. B, C, D
6. B
7. C
8. D
9. C
10. A
11. A, C, E
12. C

om
13. B
14. C

.c
15. B

ep
16. C

pr
17. A
18. D st
te
19. B
ng

20. A
21. C
si

22. A
ur
yn

Chapter 56 Drug Overdose and Poisoning

1. An elderly patient is being treated after taking too much cardiac medication. The patient
.m

states, I didnt mean to do it. I cant see as well as I used to and cant see the writing on the
medication labels. What intervention by the discharge planning nurse would help prevent
w

this from occurring again?


w

A) Transfer the patient to the nursing home where she will have her medications
w

administered to her.
B) Call the patients family and tell them they must administer the medications to the
patient when they are scheduled.
C) Make a home health referral for evaluation of resources and medication dispensing.
D) Encourage the health care provider to prescribe less toxic medication for this patient.
2. The nurse is assigned to a patient who has been in the hospital for 24 hours with a
diagnosis of acute alcohol intoxication. The patient states, I am so stressed out in my
marriage and with my job. Drinking is the only way that I can relieve that stress. What
priority intervention would be beneficial for this patient?
A) Commit the patient to a state mental health facility.
B) Refer the patient to a comprehensive treatment program.
C) Make an appointment for the patient to see a psychiatrist.
D) Inform that there are other methods to relieve stress other than drinking alcohol.
3. An adolescent patient has arrived at the emergency department via ambulance after
friends reported that she ingested a large quantity of unknown pills after having an

om
argument with her boyfriend. The patient is combative and refuses to divulge the type of
pills that she ingested. What lab studies should be performed at this time? Select all that
apply.

.c
A) Electrolyte studies

ep
B) T3 and T4

pr
C) Serum osmolality test
D) Liver panel st
te
E) Lipid panel
ng

F) Acetaminophen level
4. The nurse administers morphine sulfate IV to a patient for complaints of abdominal pain.
si

The order was for 2 mg and the nurse realized that she gave the patient 10 mg. The
ur

patients respiratory rate is 10 and she is unresponsive. What is the nurses priority
intervention at this time?
yn

A) Call the charge nurse.


.m

B) Administer a stat dose of 50% dextrose.


C) Administer naloxone (Narcan).
w

D) Fill out a risk management incident report.


w

5. A patient is receiving chelation therapy. What would the nurse explain to the patient is
w

the purpose of chelation therapy?


A) Chelation therapy will remove carbon monoxide from the blood.
B) Chelation therapy will help to reverse the effects of narcotics.
C) Chelation therapy will help promote bowel movements so that the medication will
pass through the large intestine.
D) Chelation therapy will remove toxic levels of metals from the body.
6. A patient in the intensive care unit is in acute renal failure secondary to acute tubular
necrosis from a nephrotoxic medication and has an anion gap of 20 mEq/L. What does
this laboratory value indicate?
A) The patient has metabolic acidosis.
B) The patient has metabolic alkalosis.
C) The lab value is within normal range for this patient.
D) The patient is in end-stage renal failure.
7. A toddler is being discharged from the hospital after being monitored for 24 hours due to

om
a possible ingestion of household cleaner. What priority instruction should be provided to
the parents before discharge?

.c
A) The parents should be informed that if the toddler is brought to the hospital again,
child protective services will be called.

ep
B) Inform the parents that they were negligent in the care of their child and need to lock

pr
up the household cleaners.
C) Educate the parents on methods to child-proof the home so that the child will not
have access to various harmful materials.st
te
D) Tell the parents not to worry; many parents bring their children in for similar
ng

problems.
8. A patient has arrived at the emergency department via ambulance. The EMT states, She is
si

barely responsive. She took all of the Ativan in this bottle. What medication should the
nurse prepare to administer to the patient?
ur

A) Naloxone (Narcan)
yn

B) Physostigmine (Antilirium)
.m

C) Methylene blue
w

D) Flumazenil (Romazicon)
w

9. The patient has been taking amitriptyline (Elavil) for neuropathic pain in the lower
extremities. He comes into the emergency room stating, I took 15 of them right before I
w

got here. I was tired of it all but should never have done this. Someone help me! What
intervention can help prevent absorption of the medication?
A) Syrup of ipecac
B) Benzodiazepines
C) Gastric lavage
D) Hyperbaric oxygen therapy
10. The patient has been found to have a defective and leaking fentanyl (Sublimaze) patch
used for severe pain from recurring migraine headaches. What observations by the nurse
indicate complications from this defect?
A) Respirations 10 breaths per/minute
B) Blood pressure 96/68
C) Heart rate 106 beats/minute
D) Blood pressure 160/86
11. A patient is admitted to the emergency department after ingesting an unknown amount of

om
a mixed substance containing cocaine and an opioid. What nursing assessment finding
would indicate that the patients life is in immediate danger?

.c
A) Agitation and combativeness

ep
B) Tachycardia and elevated blood pressure
C) Depressed respiratory rate and volume

pr
D) Hypoxemia and metabolic acidosis
st
12. A patient is admitted to the emergency department after ingesting a large amount of an
te
unknown substance while at a beach party. The nurse finds delirium, dry and flushed
skin, dilated pupils, fever, decreased bowel sounds, and tachycardia. Urinary catheter
ng

insertion shows urinary retention. What ingested substance does the nurse most suspect?
si

A) Malathion
ur

B) Jimson weed
C) Opiates
yn

D) Cocaine
.m

13. During the initial treatment of a patient with a poisoning or overdose, treatments to
prevent absorption and to enhance elimination of the agent are of primary importance.
w

What substance does the nurse administer to enhance elimination of an orally ingested
w

alkaline caustic substance?


w

A) Mild acid
B) Antivenin
C) Emetic
D) Activated charcoal
14. A 2-year-old has been resuscitated after ingestion of a cleaning solution. As part of
discharge teaching, what is the most important information for the nurse to include?
A) Developmental characteristics of toddlers
B) Proper storage and labeling of poisons
C) Use of the poison control phone number
D) A list of most common household poisons
15. A patient has been admitted to the emergency department after ingesting an unknown
substance. The nurse finds agitation, tachycardia, hypertension, and episodes of tonic-
clonic muscle movements. Arterial blood gases reveal normal oxygenation and metabolic
acidosis. What ingested substance does the nurse most suspect?
A) Antihistamine cold medication

om
B) Carbamate insecticide
C) Lomotil

.c
D) Cocaine

ep
16. The patient has been admitted to the CCU after taking a large amount of cocaine
accompanied by large amounts of opioid. Initial detoxification was started in the

pr
emergency department. The patient is currently intubated and on mechanical ventilation.
To identify life-threatening complications from these substances, what is the nursing
priority? st
te
A) Continuous cardiac monitoring
ng

B) Assisting respirations with bag-mask device


si

C) Use of jaw thrust maneuver to protect airway


ur

D) Monitor serum drug levels of opioids


17. The nurse is providing initial care for a patient who has experienced an ocular splash
yn

injury of a toxic liquid. What is the priority of care?


.m

A) Irrigate eyes for 15 to 30 minutes with tap water.


B) Flush eyes with antidote to the substance.
w

C) Telephone the poison control center for instructions.


w

D) Obtain ophthalmology consult at bedside.


w

18. The patient has extensive dermal exposure to a dermal toxin. What is the nursing priority
of care?
A) Remove clothing and shower for 15 to 30 minutes.
B) Irrigate affected area with an alkaline solution.
C) Include use of soap in initial shower.
D) Apply a topical soothing and moisturizing agent.
19. The nurse is caring for a patient who has inhaled an airborne toxin. What is the priority of
care?
A) Protect the patients airway.
B) Administer supplemental oxygen.
C) Protect the airways of rescuers.
D) Administer antidote to toxin.
20. A child has ingested an unknown amount of household bleach, an alkaline substance. The
child is currently tachypneic and lethargic. What is the nursing priority treatment?

om
A) Have the child drink 8 ounces of water.
B) Insert a nasogastric tube for lavage.

.c
C) Administer a mild acid such as vinegar.

ep
D) Administer supplemental oxygen.

pr
21. A patient is admitted to the emergency department after ingesting a large number of
st
tablets. Gastric lavage is ordered. What is the best nursing action during this procedure?
te
A) Insert a small-bore nasogastric tube to minimize aspiration risks.
B) Position the patient in the left lateral decubitus position with his head down.
ng

C) Lavage the stomach with 200 mL of fluid until pill fragments are seen.
si

D) Initiate the lavage procedure no sooner than 2 hours after ingestion.


ur

22. A patient has swallowed a large amount of pills. Activated charcoal, an adsorbent, is
yn

ordered to reduce toxic absorption. For best use of this substance, what action does the
nurse initiate?
.m

A) Administer via nasogastric tube to increase the speed of administration.


B) Verify that the ingested pills will be adsorbed by activated charcoal.
w
w

C) Administer the charcoal 4 hours after the pills were ingested.


w

D) Combine the charcoal with an emetic to increase elimination of toxins.


23. The patient has experienced an accidental salicylate overdose. Urine alkalinization with
intravenous sodium bicarbonate solution is ordered. What symptom, if found by the
nurse, most indicates a complication of this therapy?
A) Increased urine pH
B) Increased urine salicylate levels
C) Compensated metabolic alkalosis
D) Altered level of consciousness

Answer Key

1. C
2. B
3. A, C, D, F
4. C

om
5. D
6. A

.c
7. C

ep
8. D

pr
9. C
10. A st
te
11. C
ng

12. B
13. D
si

14. B
ur

15. D
yn

16. A
.m

17. A
18. A
w
w

19. C
w

20. B
21. B
22. B
23. D

You might also like